You are on page 1of 149

Solution of 162 maths combinedly

(from 23/01/2018-23/02/2018 posted on BCB page)

-----------------------------------------------------------------------------
The file made,edited & completed by
RUBEL CHOWDHURY
Bsc.(Hons) in Botany
Msc in Botany,
Jagannath University

This process will continue………..till the completion of 1500 maths


Part – 01(23/01/2018-28/01/2018)

1.What percentage of numbers from 1 to 70 have 1 or 9 in the units


digit?
Ans with explanations:
From 1 to 70 unit digit 1 are 1,11,21,31,41,51,61,, total =7
From 1 to 70 unit digit 9 are 9,19,29,39,49,59,69,, total=7
From 1 to 70 unit digit 1 or 9 are 7+7=14
So,Required percentage =14×100/70=20%(ans)
Courtesy to Pintu Palit
2.Sequence of numbers are 1,11,21,1211,111221.What will be the
next one in the above sequence?
Ans with explanations:
To generate a member of the sequence from the previous member, read off
the digits of the previous member, counting the number of digits in groups of
the same digit.
For example:
1 is read off as "one 1" or 11.
11 is read off as "two 1s" or 21.
21 is read off as "one 2, then one 1" or 1211.
1211 is read off as "one 1, one 2, then two 1s" or 111221.
111221 is read off as "three 1s, two 2s, then one 1" or 312211(ans)
3.A man walked diagonally across a square lot. Approximately what
was the percent saved by not walking along the edges?
Ans with explanations:
let,
one side of the square=10 unit
we know that,Diagonal of the square is a√2
So, according to question,
the hypotenuse is 10√2
so he saves=10+10-10√2=20-10√2=10(2-√2)=10*0.6=6
Now,save in percentage(%) = 0.6*100/2=30%(ans)

Courtesy to Ayrin Hasan


4.How many pairs of letters in the word 'CHAIRS' have as many
letters between them in the word as in the alphabet?
Ans with explanations:
Letters in word-----C H A & R S
Letters in alphabet----C B A & R S
So,In the word “CHAIRS”, there are 2 pairs ,one is C A & the other is R S(ans)
5.A survey showed that 63% of Bengali people like chicken
whereas 76% like fish.If X% like both fish & chicken then what is
the range of X?
Ans with explanations:
Let the people who like both be x% and the people who like none be n% of the
total ; take the total as 100.

Thus,63 people like chicken , 76 people like fish, 'x' people like both and 'n'
people like none.

Now, Poeple who like only chicken = (63 - x) ; only fish = (76 - x).

Since total people = 100

A:T:Q;
(Only chicken) + (Only fish) + Both + None = 100.

Or, (63 - x) + (76 -x ) + (x) + (n) = 100.

Or, 139 -x + n = 100

Or x - n = 39
But x cannot exceed 63 since 63 people like chicken.
Hence, the range required :-
39 ≤ x ≤ 63 (ans)
6.Worker W produces N units in 5hrs, workers W & V working
independently but at the same time produces N units in 2hrs.How long
would it take V alone to produce N units?
Ans with explanations:
Workers W produce in 5 hrs N units
then,W produce in 2 hrs 2N/5 units
Again,Workers W & V produce N units in 2 hrs
so,The number of units produced by V alone in 2 hrs=(N-2N/5)
=3N/5
So,The number of units produced by V alone in 1hr =3N/5*2=3N/10
Now,3N/10 units V produce alone in 1hr
so,N units V produce alone in (10N/3N)hr
=10/3 hr
=3hr 20Min(ans)
Special thanks to KS Tom
7.In a group consisting of 40 people,24 were female & the rest were
male.Half of the members were smokers & the rest are non smokers.If 1/3
of the female members were smokers,how many of the male members are
non smokers?
Ans with explanations:
Total member= 40
Female member= 24
So, Male= 40-24= 16
Total smoker= 40 of 1/2= 20
None smoker= 40-20=20
Female smoker= 24 of 1/3 = 8
So, Male smoker= 20-8= 12
Male non smoker= 16-12= 4 ( ans)
Courtesy to Sohel Rana & Firoz Hossain Firoz
8.The angle of the elevation of the sun, when the length of the shadow of the
tree root 3 times the height of the tree is-
Ans with explanations:
Let,
The height of the shadow is h
so,the length is root 3h
we know,
tan(angle)=height/base
=>tan(angle)=h/root 3h
=>tan(angle)=1/root 3
=>tan(angle)=tan 30
so,angle= 30 degree(ans)
9.At 3.40 the hour hand & the minute hand of a clock form an angle of it?
Ans with explanations:
Angle traced by hour hand in 12 hrs = 360 degree
Angle traced by it in 11/3 hrs(3+40/60=3+2/3=11/3)=360*11/12*3=110 degree
Angle traced by minute hand in 60 min=360 degree
Angle traced by it in 40 min=360*40/60=240 degree
Now,required angle=(240-110)=130 degree(ans)
Shortcut:
[11m-60h]/2
=[11*40-60*3]
=[440-180]/2
=260/2
=130 degree(ans)
10.A sum of money lent at compound intrest for 2 yrs at 20% per annum
would fetch tk 482 more, if the interest was payable half yearly than if it was
payable annually.What will be the sum?
Original Question:
A sum of money is lent out at compound interest rate of 20 % per annum for 2
years. It would fetch tk 482 more if interest is compounded half-yearly. Find
the sum.
Ans with explanations:
Let,
Sum=x tk
C.I. when compounded annually
=x(1+20/100)^2-x=11x/25 tk
C.I. when compounded half yearly
=x(1+20/100*2)^4-x
=4641x/10000 tk
A:T:Q;
4641x/10000-11x/25=482
=>x=20000 tk(ans)
11.How many miles can a motor cyclist travel from 8:25am to 9:55am,at
a speed of 820 mph?
Ans with explanations:
8:25am to 9:55am=90 minutes
So,at 820 mph the distance covered=820*90/60=1230miles(ans)
12.A is 30% more efficient than B.How much time will they, working
together, take to complete a job which A alone could have done in
23dys?
Ans with explanations:
Let ,B takes x days to do the work
A:T:Q;
100:130=23:x
So,x=299/10
A's 1 day work=1/23p
B's 1 day work=10/299p
A+B do in 1 day=(1/23+10/299)=23/299=1/13p
A and B together can complete 1/13p in 1day
A and B together can complete 1p in 13days(ans)
13.a,b,c,d & e are 5 consecutive numbers in increasing order of
size.Deleting one of the five numbers from the set decreased the sum of
the remaining numbers in the set by 20%.Which of the numbers was
deleted from a,b,c,d & e?
Ans with explanations:
Let,
a=x
b=x+1
c=x+2
d=x+3
e=x+4
Sum of the five numbers=5x+10
After deleting one number the sum will be
=5x+10-(5x+10)*20/100
=4x+8
So,the deleted number
=(5x+10)-(4x+8)
=x+2
=c(ans)
14.During the next tree plantation week a company is considering
planting tree in line of its own rectangular piece of land which is 90 feet
long & 66 feet wide.The land surrounded by the boundary wall of 5.It was
also decided that the distance from one tree to another in both row &
column will be 4 feet.What is the maximum number of tree can bee
planted on the land?
Ans with explanations:
Given that,
The length of the land=90ft
& the width=66ft
As,the land surrounded with 5ft wall
Then,the possible length for plantation=90-5*2=80 ft
&the possible width=66-5*2=56ft
Again,the distance between two trees 4ft
So,In a row tree can be planted=56/4+1=15 trees
& in a column trees can be planted=80/4+1=21 trees
so,the total trees can be planted
=15*21=315 trees(ans)
15.A man & a boy together can complete a job in 81dys.Two man & three
boys together can complete the job in 36dys.In how many days can 4
men working together complete the job?
Ans with explanations:
Let,
1 man's 1 day's work=x
& 1 boy's 1day's work=y
A:T:Q;
x+y=1/81..................(1)
2x+3y=1/36..............(2)
from 1 equation we get,
x=1/81-y...................(3)
Now,after adding the value of x in equation (2) we get,
2(1/81-y)+3y=1/36
=>y=1/324
so,from equation (3) we get,
x=1/81-1/324
=>x=1/108
Now,
1 man does 1/108pwork in 1 day
4 men do 1p in108/4=27dys(ans)
16.Due to reduction of 6(1/4)% in the price of sugar a man is able to buy
1 kg more for tk 120.Find the original & reduced rate of sugar?
Ans with explanations:
Let,Previous price 100tk
At 6*1/4% ,the reduced price=(100-100*25/4*100)tk=375/4 tk
If current price 375/4tk then previous is 100tk
If current price 120tk the previous is 120*4*100/375 tk or 128tk
So,Original price for 1kg is=128-120=8 tk (ans)
Now,If original price is 100tk then reduced price is 375/4 tk
If original price is 8tk then reduced price is 375*8/4*100 tk or 7.5tk(ans)
17..After replacing an old member by a new member,it was found
that the avg age of 5 members of a club is the same as it was 3yrs
ago.What is the difference between the ages of the replaced & the
new member?
Ans with explanations:
Let,
The ages of the five members at present be a, b, c, d & e years.
And the age of the new member be f years.
So, the new average of five members' age
= (a + b + c + d + f)/5 ------- (1)
Now,
Their ages 3 years ago = (a-3), (b-3), (c-3), (d-3) & (e-3) years
So, their average age 3 years ago
From comparing the above equations we get,
(a + b + c + d + e - 15)/5=(a + b + c + d + f)/5
==> a + b + c + d + e - 15= a + b + c + d + f
==> e - f =15
So,difference between the old & the new member is 15 yrs(ans)
18.A mixture contains alcohol & water in the ratio 4:3.If 5 litres of
water is added to the miixture,the ratio beecomes 4:5.Find the
quantity of the alcohol in the given miixture?
Ans with explanations:
Let,
Alcohol & water ratio in the mixture 4x litre & 3x litre
A:T:Q;
4x/3x+5=4/5
=>x=5/2
so,quantity of alcohol in the mixture=4*5/2=10 litre(ans)
19.In a flight of 600km aircraft was slowed down due to bad
weather.Its avg speed for the trip was reduced by 30kmph & the
time of the flight increased by 1 hour.Findout the duration of the
flight?
Ans with explanations:
Let,Total distance=600 km
Actual time & speed is t hr & v kmph
Reduced speed (v-30)kmph & increased time (t+1)hr
we know that
speed,V=distance/time
A:T:Q;
600/t-600/(t+1)=30
=>600t+600-600t/t(t+1)=30
=>600=30(t^2+t)
=>20=t^2+t
=>t^2+t-20=0
=>(t-4)(t+5)=0
either,t=4 or t=-5(as neg value neglected)
So,here the actual time or duration of flight is,t=4 hour(ans)
20.A hall is 15m long & 12m broad.If the sum of the areas of the
floor & the ceiling is equal to the sum of the areas of the 4
walls,what is the volume of the wall?
Ans with explanations:
Given that,
The length,l=15m
& the breadth, b=12m
A:T:Q;
Area of the floor+ceiling=Area of the four wall
==>l*b+l*b=l*h+b*h+l*h+b*h
==>2*l*b=2*l*h+2*b*h
==>2*l*b=2h(l+b)
==>l*b=h(l+b)
==>h=l*b/l+b
==>h=15*12/15+12
So,h=20/3
Now,the volume of the wall
=l*b*h
=15*12*20/3
=1200 m^3(ans)
21.A hollow iron pipe is 21cm long & it's external diameter is 8cm.If
the thickness of the pipe is 1cm & iron weighs 8gm/cm^3,then what
is the weight of the pipe?
Ans with explanations:
Given that,
External radius,R=4 cm
Internal radius,r= 3 cm [because thickness of pipe is 1 cm]
We know that,
Volume of iron=πr^2h
= π(R^2-r^2)h
=[22/7×(4^2−3^2)×21]
= 462cm^3
So,Weight of iron
= (462 x 8)gm
= 3696 gm
= 3.696kg(ans)

22.The ratio between the length & the breadth of a rectangular park
is 3:2.If a man cycling along the boundary of the park at the speed
of 12kmph completes 1 round in 8min,then what is the area of the
park?
Ans with explanations:
Perimeter of the rectangular park
=distance covered in 8mins
=(12*1000*8/60)m
=1600m
Let,Length=3x & width=2x meter
ATQ,
2(Length+Breadth)= Perimeter
==>10x=1600metre
So,x=160m
Now,Length=3x=3x160=480m
breadth=2x=2x160=320m
So, Area of the rectangle=LengthXBreadth
=(480x320)m^2
=153600m^2(ans)
23.The diagonal of the floor of a rectangular closet is 7.5ft.The
shoter side of the closet is 4.5ft.What is the area of the closet in
square feet?
Ans with explanations:
Given that,
The shorter side of the closet,b=4.5ft=9/2 ft
We know,
The diagonal of the rectangular=root (a^2+b^2)
=root{a^2+(9/2)^2}
ATQ;
root(a^2+81/4)=7.5
=>a^2+81/4=(15/2)^2
=>a^2=225/4-81/4
=>a^2=144/4
=>a^2=36
So,a=6
Now,the area of the closet=(6*4.5)sqft=27sqft(ans)
24.A father divides his property between his two sons A & B.A
invest the amount at compound profit of 8% p.a. & B invest the
amount at 10% p.a. at simple profit.At the end of two years the
profit received by B is tk 1336 more than the profit received by
A.Find the share of A in the father's property of tk 25000?
Ans with explanations:
Suppose,
Share of A= x tk
share of B= (25000-x) tk
SI @ 10% p.a. for 2 yrs =B=(25000-x)*10%*2=5000 - 0.2x
CI @ 8% p.a. for 2 yrs=A =x(1+8%)^-x=0.1664x
ATQ,
=> 5000 - 0.2x - 0.1664x = 1336
=> 5000 - 0.3664x = 1336
=> 0.3664x = 3664
=> x = 3664/0.3664
=>x=10000
So,the share of A is 10000 tk(ans)
25.One day Wahid started 30mins late from home & reached his
office 50mins late,while driving 25% slower than his usual
speed.How much time in min does Wahid usually take to reach his
office from home?
Ans with explanations:
Let,
Actual time taken by Wahid= x min & his actual speed y km/min
We know,
distance=speed*time=xy km
Extra time taken=(50-30)min=20min
ATQ;
(x+20)*(y-y*25%)=xy
=>(x+20)*3y/4=xy
=>3(x+20)=4x
=>x=60
So,His actual time is 60min(ans)
26.A worker buys some shirts & some ties.Shirts cost tk 70 each &
ties cost tk 30 each.If he spends exactly tk 810 & buys the
maximum number of shirts possible under this conditions,the ratio
of the shirts to ties is?
Ans with explanations:
Let,
x and y be the number of Shirts and ties respectively.
ATQ,
70x + 30y = 810
Here we can make a trial & error process…...
Value (9,6) satisfied the above equations
such as 70*9+30*6=810
So,the ratio of the shirts to ties=9:6=3:2(ans)
27.The diagonal of a rectangle is root 41cm & it's area is
20sq.cm.What is the perimeter of the rectangle?
Ans with explanations:
We know,
The diagonal of rectangular=root(a^2+b^2) cm,
the area= ab sqcm,
& the perimeter =2(a+b) cm
ATQ,
root(a^2+b^2)=root 41
=>a^2 + b^2 = 41-----------------1
Again,
ab = 20--------------------------------2
From equation 1 we get,
a^2+b^2=41
=>(a+b)^2-2ab=41
=>(a+b)^2-2*20=41
=>(a+b)^2=41+40=81
=>a+b=9
=>2(a+b)=18
So,the perimeter is 18cm(ans)
28.A student multiplied a number by 3/5 instead of 5/3,what is the
percentage error in the calculation?
Ans with explanations:
Let,
The number x.
He should have multiplied by x by 5/3= 5x/3.
By mistake he multiplied x by 3/5 = 3x/5
Error = (5x/3 - 3x/5) = 16x/15
Error % = (error/True vaue) * 100
= [(16/15) * x/(5/3) * x] * 100 = 64 %(ans)

29.In a mixture of 60litres the ratio of milk & water is 2:1.If this ratio
is to be 1:2,then what is the quantity of water to be further added in
the miixture?
Ans with explanations:
Given that,
Total amount of mixture=60 litres
According to given ratio,
Milk=40 ltrs & Water=20 ltrs
Suppose,
X ltr water would be added in the mixture.
ATQ,
40/20+x=1/2
=>20+x=80
=>X=60
So,60 litre water would be added into the mixture(ans)
30.When 0.36 is written in simplest form,the sum of the numerator &
denominator is?
Ans with explanations:
Given decimal number=0.36
It's simplest form=0.36=36/100=9/25
The sum of the numerator & denominator= (9+25)=34(ans)
31.Two integers will be randomly drawn from the sets A={2,3,4,5} &
B={4,5,6,7,8}, one integer from set A & the other from set B.What is
the probability that the sum of the two integers will be 9?
Ans with explanations:
Total number of possibilities of choosing two numbers is
= 4 * 5 = 20 ways
No. of possibilities of choosing numbers such that their sum is 9 is:
(2,7),(3,6),(4,5),(5,4), thus 4 ways.
So,Probability= 4/20= 1/5(ans)
courtesy to Saiduzzaman Titu
32.In a certain shop notebooks that normally sell for BDT 1.50 each
are on sale at 2 for BDT 2.89.How much can be saved by
purchasing 10 of these notebooks at sale price?
Ans with explanations ::
While the price of each notebook is tk 1.50,then the price of 10
notebooks=10*1.50=15 tk
While the price of 2 notebooks is tk 2.89,then the price of 10
notebooks=10*2.89/2=14.45 tk
So, the price can be saved=15-14.45=0.55 tk(ans)
33.If a merchant offers a discount of 30% on the list price,then he
makes a loss of 16%.What % profit or loss will the merchant make if
he sells at a discount of 10% of the list price?
Ans with explanations ::
Let,
The list price is x tk & cost price is 100 tk.
At 30% discount,the selling price=(x-30*x/100)=0.7x tk
The merchant would have got =(100-100*16/100)=84 tk,when he offered a
discount of 30%.
ATQ,
0.7x=84
=>x=120
So,the list price is 120 tk
At 10% discount,he will sell the article at=(120-120*10/100)=108 tk
As the cost price was 100 tk & selling price was 108 tk
Then, the profit is =8%(ans)
34.Samad was planning on depositing a certain amount of money
each month into a bank.He then decided not to make any
contributions during May & June.To make the same annual
contribution take he had planned,by what percent should he
increase monthly deposit?
Ans with explanations:
Let,Samad planned to deposit 100 tk per month that would be 1200 tk per
year.
But,he decided not make any contributions for May & June
Then,his deposit for the rest of 10 month =1200 tk
So,his monthly payment increase to 1200/10=120 tk
Now,the percent increase of monthly payment=120-100=20%(ans)
35.A leak in the lower portion of a tank can empty the full tank in
9hr.An inlet pipe fills water at the rate of 10 litre. per min.When the
tank is full, the inlet is opened & due to leak, the tank is empty in
16hr. How many litres does the cistern hold?
Ans with explanations::
Part of tank empty in 1hr=1/9th
Given that,
in 1 min,tank filled=10 litres
so,in 1hr or 60 mins tank filled=600 litres
Let,
part of tank filled in x hour
ATQ;
1/9 - 1/x = 1/16
=>x=24hrs
Hence,
in 1 hr,part of tank filled= 7/144 th
ATQ,
7/144th part of tank contained 600 litres
So,1 part of tank contained 600*144/7 litres=12342.857 litre(ans)
36.A man rows to a place 48km distant & back in 14hr.He finds that
he can row 4km with the stream in the same time as 3km against
the stream.What is the rate of the stream?
Ans with explanations:
Let,
The man moves 4km downstream in x hrs & boat speed is a kmph & current
speed is b kmph
Then,the speed of downstream,
(a+b)=4/x kmph.........1
& the speed of upstream,
(a-b)=3/x kmph........2
ATQ,
48/(4/x)+48/(3/x)=14
=>x=1/2
Substract equation 1 & 2,we get
a+b-a+b=4/x-3/x
=>2b=(4-3)/x
=>2b=1/(1/2)
=>b=1
So,stream speed 1 kmph(ans)
37.A man rows a boat from point A against current for 10 mins &
then come back with the current for next 10 mins & reaches to a
point B.If distance between A & B is 1km,find the speed of the
current?
Ans with explanations:
Going with current & against current for the same time means speed of the
boat in still water is neutralized.
Only speed of current here moves the boat.
Boat is moved 1km by current in (10+10)=20 mins=20/60 hr=1/3 hr
Therefore,speed of current={1/(1/3)}=3 kmph(ans)

Alternate Solution:
Let,
Boat speed= x kmph
& current speed=y kmph
Against current effective rate
=(x-y) kmph
So,distance covered in 10 mins(10/60 hr)
=(x-y)*10/60=(x-y)/6 km
With current effective rate
=(x+y) kmph
So,distance covered in 10 mins(10/60 hr)
=(x+y)*10/60=(x+y)/6 km
The difference between this is the backward distance that the man has
traveled.
ATQ;
(x+y)/6 - (x-y)/6=1
==>y=3
So,speed of current 3 kmph(ans)
38.Two pipes P & Q can fill a cistern in 12 mins & 15 mins
respectively but 3rd pipe R can empty the full tank in 6min.P & Q
are kept open for 5min. in the beginning & then R is also opened.In
what time is the cistern emptied?
Ans with explanations:
Part filled in 5 min by P & Q=5(1/12+1/15)=3/4 portion
Part emptied in 1 min when all the pipes are opened together
={1/6-(1/12+1/15)} = 1/60 p
So,1/60 p emptied in 1 min
Now,3/4 p emptied in 3*60/4 min=45 mins(ans)
39.A milk vendor has 2 cans of milk.The 1st contains 25% water &
the rest milk.The 2nd contains 50% water.How much milk should he
mix from each of the containers so as to get 12litres of milk such
that the ratio of water to milk is 3:5?
Ans with explanations:
Let,
In can 1,
Total mixture be x litre
So,water=x/4 litre & milk=3x/4 litre
Again,In can 2,
Total mixture be y litre
So,water=y/2 litre & milk=y/2 litre
ATQ,
x/4+y/2:3x/4+y/2=3:2
=>x:y=1:1
So,milk should be added from each of the can =12*1/2=6 litre
Ans:6 litre & 6 litre.
...............Yousuf Ali
40.A can contains a mixture of two liquids A & B in the ratio 7:5.
When 9 litres of mixture are drawn off & the can is filled with B,the
ratio of A & B become 7:9.How many litres of liquid A was
contained by the can initially?
Ans with explanations:
Let,
The mixture be x litre
Given that,
A & B in the ratio=7:5
Here,A in the mixture=7x/12 litre
& B in the mixture=5x/12 litre
After,9 litres drawn off the remaining quantity in A=7(x-9)/12 litre
& in B =5(x-9)/12 litre
Again,9 litres replaced with B
ATQ,
7(x-9)/12:{5(x-9)/12}+9=7:9
=>x=36
So,initially A was contained=7*36/12=21 liitres(ans)
...........#YousufAli
41.Samad went to the stationers & bought things worth tk 25,out of which
30 paisa went on sales tax on taxable purchases.If the tax rate was 6%,then
what was the cost of the tax free items?
Ans with explanations:
Let, tk x is the total cost of the tax free items.
Given that,
Tax rate=6%
Tax amount=30 paisa=30/100 tk
Hence,
Cost of the taxable items*6%=30/100--------------1
Here,
Cost of the taxable item is
=(cost of all items-Tax-cost of tax free items)
=(25-30/100-x)
So,from equation 1 we get,
(25-30/100-x*6/100)=30/100
=>x=19.70
So,19.70 tk is the total cost of the tax free items(ans)
42.A bank offers 5% compound interest calculated on half yearly basis.A
customer deposits tk 1600 each on 1st january & 1st july of a year.At the
end of the year,what is the amount he would have gained by way of
interest?
Ans with explanations:
Amount after 1 year on tk 1600(deposited on 1st Jan) at 5% when interest calculated
half yearly
=p(1+(r/2)/100)^2t=1600(1+5/2*100)^2*1=1600(1+1/40)^2 tk
Amount after 1/2 year on tk 1600(deposited on 1st July) at 5% when interest
calculated half yearly=1600(1+5/2*100)^2*1/2=1600(1+1/40) tk
Total amount after 1 year
=1600(1+1/40)^2+1600(1+1/40)=3321
Compound interest=3321-3200=121 tk(ans)
43.A boat running upstream takes 8hrs 48mins to cover a certain
distance,while it takes 4hrs to cover the same distance running
downstream.What is the ratio between the speed of the boat & the water
current respectively?
Ans with explanations:
Assume,
The speed of boat & current respectively be x & y kmph
& distance d km.
8hrs 48 mins=8 hrs+48/60 hrs=44/5 hrs
As we all know that,speed=distance/time
Hence,
With stream,
x+y=d/(44/5)
=>x+y=5d/44
=>44(x+y)/5=d--------------------1
Against stream,
x-y=d/4
=>4(x-y)=d--------------------------2
Comparing the above equations,we get
44(x-y)/5=4(x+y)
=>x:y=8:3
So,the ratios between the speed of boat & stream are 8:3(ans)
44.How many kg of sugar costing tk 9 per kg must be mixed with 27 kg of
sugar costing tk 7 per kg so that there may be a gain of 10% by selling the
mixture at tk 9.24 per kg?
Ans with explanations:
At 10% gain,
While the selling price is 110tk cost price is 100tk
While the selling price is 9.24tk cost price is (9.24*100/110)tk=8.4 tk
Let,x be the quantity of sugar costing tk 9 per kg.
x*9+27*7=8.4(x+27)
=>9x+189=8.4x+226.80
=>0.6x=37.8
=>x=63
So,63kg is the quantity of sugar costing tk 9 per kg(ans)
45.A runs 5/3 times as fast as B.If A gives B a start of 80m,how far must the
winning post be so that both might reach it at the same time?
Ans with explanations :
Given that,
Ratio of speeds of A & B =5/3:1=5:3
Thus, in a race of 5m A gains 2m over B
Now,
2m are gained by A in a race of 5m
Then,80m will be gained by A in a race of (5*80/2)=200m
So,winning post is 200m away from the starting point(ans)
46.A boat can travel with a speed of 13kmph in still water.If the speed of
the stream is 4kmph,find the time taken by the boat to go 68km
downstream?
Ans with explanations:
Given that,
Speed of boat,x=13 kmph
Speed of stream,y=4 kmp
So,Downstream speed =x+y=13+4=17 kmph
Now,time taken to travel 68 km downstream= 68/17 hrs =4 hrs(ans)

47.A boat takes 90 mins less to travel 36 miles downstream than to travel the
same distance upstream.If the speed of the boat in still water is 10mph,then what
is the speed of the stream?
Ans with explanations :
Given that,
Speed of the boat,x=10 mph
Let,
Speed of the stream=x mph
Then,downstream speed=(10+x) mph
upstream speed=(10-x) mph
ATQ;
36/(10-x)- 36/(10+x)=90/60
=>(x+50)(x-2)=0
either,x= -50 or x=2
Since, Speed can not bee negative,So the value is 2 mph(ans)
48.Tea worth tk 126 per kg & tk 135 per kg is mixed with a 3rd variety in
the ratio 1:1:2.If the mixture is worth tk 153 per kg,what will be the price
of the 3rd variety per kg?
Ans with explanations:
Let,
we use 1kg of first two varieties and 2 kg of 3 rd variety & Total is 4 kg.
So the total price is = 153*4 = 612 tk
price paid for 1 st two varieties = (126+135) = 261 tk
price paid for 3 rd variety =612-261 =351 tk
So, Price per kg for 3 rd variety=351/2= 175.50 tk per kg.(ans)
49.Two friends M & N have 40% & 60% shares in a business,after
sometime a 3rd friend joined the business by investing tk 5 lac & acquired
20% shares in the business.What is the share of M in the business now?
Ans with explanations:
As,3rd investors 20% share in business=500000 tk

Then,

While Investing amount 20 tk ,total investment in business 100


While Investing amount 500000 tk ,total investment in business
=(500000*100/20) tk=2500000 tk
Thus, A & B"s share in business =2500000-500000=2000000 tk
so, M"s share = (2000000*40/100) tk= 800000 tk
So,M"s share% = 800000*100/2500000 =32%(ans)

50.According to car dealers sales report, 1/3rd of the cars sold during a
certain period were sedans & 1/5th of the others were station wagons.If N
station wagons were sold during that period,how many sedans in terms of
N sold?
Ans with explanations::
Let,
Total car sold be 15(LCM of denominator 3 & 5)
Then,Sedan sold=1/3 of 15=5
Remaining cars=15-5=10
Station wagons sold=1/5 of 10=2
Now,
2 is equivalent to N
Then,5 is equivalent to 5N/2(ans)
courtesy to Sarker Mamun
51.A & B entered into a partnership with capitals in the ratio 4:5.After 3
months A withdrew 1/4th of his capital & B withdrew 1/5th if his
capital.The gain at the end of 10 months was tk 760.What is A's share in
this profit?
Ans with explanations ::
Let,
According to the given ratios the capital of A & B is 4x & 5x tk
Given that,
The gain at the end of 10 months 760 tk
A:B=[4x*3+(4x-4x*1/4)*7] : [5x*3+(5x-5x*1/5)*7]
=(12x+21x) : (15x+28x)
=33x : 43xx
=33 : 43
So,A's share=(760*33/76)=330 tk(ans)
52.2 men & 3 boys can do a piece of work in 10 days while 3 men &
2 boys can do the same work in 8 days.In how many days can 2
men & 1 boy do the work?
Ans with explanations:
Let,
1 man's 1 day's work=x
& 1 boy's 1day's work=y
A:T:Q;
2x+3y=1/10..................(1)
3x+2y=1/8..............(2)
(1*3-2*2)=>
6x+9y-6x-4y=3/10-2/8
=>5y=6-5/20
=>y=1/100
from equation 1,we get
2x+3/100=1/10
=>x=7/200
For 2 man & 1 boy the equation is
2x+y=2*7/200+1/100=2/25
2 men & 1 boy do 2/25p in 1 day
so,1 portion in 12.5days(ans)

Part – 02(29/01/2018-04/02/2018)

53.A car goes 15km per litre of petrol if it is driven at a speed of


50kmph.When the car is driven at 60kmph,it can only go 80% as
far.How many litres of petrol is needed to travel 200km if half the
distance is traveled at 50kmph & the rest at 60kmph?
Ans with Explanations:
At 50 kmp per liter distance=15 km
At 60 kmp per liter distance=15*80%=12 km
As half of distance covered at 50kmp and 60kmp equally
So,total petrol need=100/15+100/12=15 liter(ans)
............Sohel Rana

54.A man takes twice as long to row a distance against the stream
as to row the same distance in favour of the stream.What is the
ratio of the boat & the stream?
Ans with Explanations:
Let,
The speed of boat & stream is respectively x kmph & y kmph
So,against the stream the rate=(x-y) kmph
& with the stream =(x+y) kmph
We know,distance=speed*time
ATQ;
(x+y)=(x-y)*2
=>x=3ty
=>x:y=3:1
So,the ratio of the boat & the stream 3:1(ans)
........Prosenjit Chandra Roy
55.4 persons can complete a job in 8days.After 4days 2 persons
have left the work.How many days would it take for the remaining
persons to complete the work?
Ans with Explanations:
Given that,
4 persons can complete in 8 days 1 part of work
4 persons can complete in 1day 1/8 parts of work
4 persons can complete in 4 day 4/8=1/2 parts
Now,remaining work =1-1/2 =1/2 part
& remain persons =4-2 =2
1 part work done by 4 m in 8day
1 part work done by 1m in =8*4d
1/2 part work done by 2m in=8*4/2*2 = 8days(ans)
............#ProsenjitChandraRoy
56.A motor covers 24 km upstream & 36 km downstream in 6
hours,while it covers 36 km upstream & 24km downstream in 6(1/2)
hours.What is the velocity of the stream?
Ans with explanations:
let,
upstream rate = x kmph
downstrem rate =y kmph
We have,
time= distance/velocity
ATQ,
24/x + 36/y =6.....(i)
36/x +24/y =13/2...(ii)
solving two equations,
x=8
y=12
So,Upstream rate = 8 kmph
& Downstream rate= 12 kmph
Now,Speed of stream= (12-8)/2=2 kmph(ans)
.........Samrat Roy Sondhan
57.2 pipes P & Q fill a cistern in 12 mins & 16 mins
respectively.Simultaneously both the pipes are opened
together,then after how much time Q should be closed so that the
tank is full in 9 mins?
Ans with explanations:
Given that,
In 12 minutes P can fill 1 cistern
In 9 minutes P can fill 9/12=3/4 parts
Remaining parts=(1-3/4)=1/4 parts
Q can 1 part of cistern fill in 16 minutes
Q can 1/4 parts fill in 16/4=4 minutes
So, Q close after 4 minutes (ans)
...........#PintuPulit
58.If 2 pipes function together, the cistern will be filled in 6hrs.One
pipe fills the cistern 5hrs faster than the other.How many hrs it take
the 2nd pipe to fill the cistern?
Ans with explanations:
Let,
first pipe fill in = x hr
second pipe fill =x+5hr
2 pipes fill in 1 hr =(1/x +1/x+5) portion
ATQ,
1/x + 1/x+5 =1/6
=>x =10
So, second pipe fill in =10+5=15 hr(ans)
..............#ProsenjitChandraRoy

59.2 numbers A & B are such that the sum of 5% of A & 4% of B is


2/3rd of the sum of 6% of A & 8% of B.Find the ratio of A : B?
Ans with explanations:
ATQ;
5A/100+4B/100=2/3[ (6A/100)+(8B/100)]
=>(5A+4B)3=2(6A+8B)
=> 15A+12B=12A+16B
=>3A=4B
=>A/B=4/3
So,the ratio of A:B=4:3(ans)
...........#MdKhaledSaifullah
60.If Salam makes a box in every 5mins & Kalam takes 7 mins to
make a box,what will be the ratio of the numbers of boxes
produced by Salam & Kalam if they work for 5hrs & 50mins?
Ans with explanations:
Given that,
5 hours 50 minutes =5*60+50=350 minutes
Moreover,
Salam produced in 5 minute =1 box
So,Salam produced in 350min =350/5=70 box
& Kamal produced in 7 minute =1 box
So,Kalam produced =350/7=50 box
Now,the ratio of the number of boxes produced by
salam:kalam=70:50=7:5(ans)
..........#SohelRana
61.A time study specialist has set the production rate for each
worker on a job at 22 units in every 3hrs.At this rate what is the
minimum numbers of workers that should be put on the job if at
least 90 units are to be produced per hour?
Ans with explanations:
Given that,
22 units in 3 hours done by 1 worker
∴ 22 units in 1 hour done by 1x3 "
∴ 90 units in 1 hour done by 3x90/22=12.27 worker
∴ Worker needed at least 13 person.(ans)
.............#HabibAhamed
62.A boat can row 3 km with the stream in the same time as 2 km
against the stream.Find the speed of the stream?
Ans with explanations:
Let,
In downstream 3 km goes in =1 hr
In upstream 2 km goes in =1 hour
So,speed of stream = (3-2)/2 =1/2 hr(ans)
...........#ProsenjitChandraRoy
63.A wire 2m in length is cut into two pieces by which a square & a
circle can be formed so that the circle may pass through the
corners of the square.Find the radius of the circle?
Ans with explanations:
Let,
One part of the wire= x m
Other part of the wire= 2-x m
Suppose,
radius of the circle = r m
Perimeter= 2*pi*r
ATQ,
2*pi*r= x..............1
Here,Diagonal of the square= a.root(2)
ATQ,
a*root(2)=2r
=>a=root(2)*r
Now,Perimeter of the square = 4*root(2)*a=4*root(2)*r
ATQ,
4*root(2)*r=x-2...........2
ATQ,
x+(2-x)=2
=>2*pi*r+4*root(2)*r=2 [putting the value]
=>r=2/((2*pi+4*root(2))
=>r=1/(3.1416+2.83)
=>r=1/5.9716
=>r=0.1675 meter
So,the radius of the circle is 0.1675m(ans)
............#SamratRoySandhan
64.A ladder is inclined to a wall making an angle of 30degree with
it.A man ascending the ladder at the rate of 2m/s.How fast is he
approaching the wall?
Ans with explanations:
ATQ,
Angle between ladder & wall= 30 deg
Angle between wall & ground= 90 deg
So,Angle between ladder & ground= 180-90-30= 60 deg
Ladder along hypotenuse whose value is = 2
ATQ,
Cos60=base/2
=> 1/2=base/2
=> base=1m/s
So,he is approaching the wall in 1mps(ans)..........#SamratRoySandhan
65.A taka received 1 year from now is worth 90 paisa today.If you
had tk 5 lac available for investment,what would be the minimum
rate at which you would have to invest inorder to keep the value of
your capital unchanged?What redemption value of your investment
would you expect if you wanted a real rerurn of 5% per annum?
Ans with explanations:
90 paisa in one year earn(100-90)=10 paisa
100 paisa in one year earn (10*100/90)=11.11 paisa
Minimum rate=11.11%
For real return of 5%
Expected return=(11.11+5)%=16.11%
So,redemption value=500000+500000*16.11/100=580550 tk(ans)
.............Rubel Chowdhury
66.If sales tax is payable at t% of gross sales value and you intend
to make a profit of r%,
what would be the gross sales price inclusive of tax of an article
which you bought for tk K?
(Basic Bank-2008)
Ans with Explanations:
Given that,
Cost price of article=k Tk
Sales price r% profit(Excluding tax)
=(k + k of r%)
=k{(100+r)/100}
Now,
Adding t% tax on sales ,the tax is
=k{(100+r)/100}*t%
So,
Gross sales price inclusive of tax is
=k{(100+r)/100} +k{(100+r)/100}*t%
=k{(100+r)/100} *{(100+t)/100}
Answer:k{(100+r)/100} *{(100+t)/100}
.........Yousuf Ali
67.The speed of railway engine is 42kmph when no compartment is
attached & the reduction in speed directly proportional to the
square root of the number of compartments attached.If the speed
of the train carried by this engine is 24kmph when 9 compartments
are attached,what is the maximum numbers of compartments that
can be carried by the engine?
Ans with explanations:
Let,
The speed is S kmph & number of comprtments be N
ATQ;
S = 42 - k*sqrt(N)......................1 (where k is a constant of the proportionality)
Given that,
N = 9 & S = 24 kmph
Then,from equation 1,we get
24 = 42 - k*sqrt(9) = 42 - 3k
=>3k = 18
=>k = 6
Now,we have to identify the value of N when S=o
So,from equation 1,
S = 42 - k*sqrt(N)
=>0 = 42 - 6*sqrt(N)
=>6*sqrt(N) = 42
=>sqrt(N) = 7
=>N= 7^2 = 49
If the compartment number is 49 then the speed will be o kmph.
So,the maximum number of compartments is 48 that can be carried by the
engine(ans)
.............#RubelChowdhury
68.If the beginning inventory is tk 60,cost is tk 380 & ending
inventory is tk 50,what will be the cost of goods sold ?
Ans with explanations:
We have,
cost of goods sold = beginning inventory + cost price - ending inventory
= 60+380-50
= 390 tk(ans)
..........#SaiduzzamanTitu
69.Let, N be the greatest number that will divide 1305,4665 &
6905,leaving the same remainder in each case.Then the sum of the
digits in N is?
Ans with explanations:
We have,
Divisor=N
& let remainder=R
We have,
Dividend = Divisor * Quotient + Remainder
We can write,
1305= N * x+R...... (1)
4665= N * y+R....... (2)
6905= N *z+ R....... (3)
Applying (2)-(1),
3360= N(y-x)....... (4)
Applying (3)-(2),
2240= N(z-y)....... (5)
Applying (3)-(1),
5600= N(z-x)....... (4)
Then accordingly we can say that N is the HCF of 3360,2240 & 5600.
By calculating Their HCF is 1120=N is the wanted divisor.
Sum of the digits of N is=1+1+2+0=4(ans)
.......... #SamratRoySondhan
70.3 taps A,B & C can fill a tank in 12,15 & 20 hrs respectively.If A is
open all the time and B & C are open for 1 hour each alternately,
what the time when the tank will be full in?
Ans with explanations:
ATQ;
(A+B) can fill in 1 hr = (1/12+1/15)= 9/60
(A+C) can fill in 1 hr = 1/12+1/20)=8/60
So, part fill in 2 hrs = 9/60+8/60=17/60
Now,part fill in 6 hrs= 17*6/60*2=51/60
Hence,Remaining part =1-51/60= 9/60
Now it is A and B's turn to fill the tank
ATQ; 9/60 part fill by A and B in 1 hr
So,total time = 6+1=7 hrs(ans)
.....#SaiduzzamanTitu
71..A man can row three-quarters of a km against the stream in
11(1/4) mins & down the stream in 7(1/2)mins.What is the speed of
the man in still water?
Ans with explanations :
Given that,
3 quarters of a kilometer=3*1000/4=750 meter
& 11(1/4) mins=45/4 mins=675 seconds
Let,the speed of man x kmph & the stream y kmph
With stream,
x+y=750/675=10/9.........1
Against stream
x-y=750/450=5/3............2
Applying (1+2)
x+y+x-y=10/9+5/3
=>x=25/18
=>x=25/18*18/5
=>x=5
So,the speed of man=5 kmph(ans)
......#RubelChowdhury
72.2 trains A & B start from stations X & Y towards Y & X
respectively. After passing each other they take 4hrs 48mins & 3hrs
20mins to reach Y & X respectively.If train A is moving at
45kmph,then what is the speed of train?
Ans with explanations:
Let,
B's speed = x kmph
Total distance covered by A after meeting each other =45*4(48/60)=216 km
Total distance covered by B after meeting each other = x*3(20/60)=10x/3 km
ATQ,
216/x=(10x/3)/45
=>x=54
So,the speed of train is 54kmph(ans)
.............#SaiduzzamanTitu
73.A train overtakes 2 persons who are walking in the same
direction in which the train is going, at the rate of 2kmph & 4kmph
& passes them completely in 9 & 10 seconds respectively. What is
the length of the train?
Ans with explanations:
Let,
speed of train = x km/hr
length of train =y m
Given that,
person 1's speed = 2 km/hr
person 2's speed = 4 km/hr
Relative speed = x-2 km/hr = (x-2) *5/18 m/s
Relative speed 2 = x-4 km/hr = (5x-20)/18 m/s
ATQ-1;
y/ (5x-10/18) = 9
=>5x-2y =10....(i)
ATQ-2;
y/ (5x-20/18) =10
=>25x-9y =100..... (ii)
Solving the equation, we get y = 50
So, length of train =50m(ans)
........#ProsenjitChandraRoy
74.A jar contains only red, yellow & orange marbles.If there are 3
red, 5 yellow & 4 orange marbles and if 3 marbles are choosen from
the jar randomly without replacing any of them,what is the
probability that 2 yellow,1 red & no orange marbles will be
choosen?
Ans with explanations:
Total marbles = 3+5+4= 12
Where, orange 4, yellow 5 & red 3.
3 marble can choose from 12 marble in
= 12C3 ways
= 12*11*10/3*2*1ways
= 220 ways
2 yellow marbles can choose from 5 yellow marbles in
= 5C2 ways
= 5*4/2*1
= 10 ways
1 red marble can choose from 3 red marbles in
= 3C1 ways
= 3/1= 3 ways
Required probability will be
= (favorable outcomes) / (total outcomes)
= 3*10/220
= 3/22(ans)
...........#SomratRoySandhan
75.There are 5 men & 5 women sitting in a waiting room for job
interview.If two of the applicants are selected at random,what is the
probability that both will be women?
Ans with explanations:
P(both selected people are women)
= P(1st selection is a woman & 2nd selection is a woman)
= P(1st selection is a woman) x P(2nd selection is a woman)
= 5/10 x 4/9
= 2/9(ans)
......#RubelChowdhury
76.1 year ago the ratio between A's & B's salary was 3:4,ratios of
their individual salaries between last year's & this year's salaries
are 4:5 & 2:3 respectively. At present the total of their salary is tk
4160,what is the salary of A?
Ans with explanations:
Given that,
1 years ago,
A's & B's salary ratio = 3:4
Again,A's last year salary & this year salary ratio = 4:5
& B's last year & this year salary ratio = 2:3
Let,
A's last year salary =4x & this year salary = 5x
Again,B's last year salary =2y & this year salary = 3y
ATQ-1,
4x:2y = 3:4
=>8x =3y..... (i)
ATQ-2,
5x+3y = 4160
=>5x +8x =4160 ( putting the value from equation 1)
=>x = 320
So,A's present salary = 320*5 =1600 tk.(ans)
..... #ProsenjitChandraRoy
77.In how many ways a coach can choose 1st a football team &
then a basketball team if 18 boys go out for either team?
Ans with explanations:
Football team=18c11= 31824
Remaining member =7
Basketball team formed by 5 member=7c5 = 21
Total chosen = 31824*21 = 668304(ans)
.............#PrasenjitChandraRoy
78.2 pipes A & B can fill a tank in 6hrs & 4hrs respectively. If they
are opened on alternate hours & if pipe A is opened first,in how
many hours the tank shall be full?
Ans with explanations:
Let ,
Total work = 12 units ( LCM of 6 & 4)
Efficiency of Pipe A = 12/6= 2 units per hour
Efficiency of Pipe B = 12/4 = 3 units per hour
2 pipes work alternatively,
So (1+1)= 2 hours two pipes work (2+3) = 5 units
So in 4 hour 2 pipes work in 5+5= 10 units
In next hour pipe A work in 2 units
In (4+1)= 5 hours total work done by two pipes (10+2) units = 12 units.(Ans)
..........#PintuPalit
79.Excluding stoppage,the speed of a bus is 54kmph & including
stoppage it is 45kmph.For how many minutes the bus stop per
hour?
Ans with explanations:
Given that,
Excluding stoppage , speed of bus = 54 kmph
Including stoppage, speed of bus = 45 kmph
Different = ( 54-45) kmph = 9 kmph
Bus stop per hour for( 9×60)/54= 10 minutes (Ans)
.............#PintuPalit
80..In a race of 300m A can beat B by 31m & C by 18m.In a race of
450m C will beat B by?
Ans with explanations:
When A goes 300 m
Then, B goes = (300-31)m= 269 m
& C goes = (300-18)m = 282 m
In 282 m race , C beat B in (282-269)m = 13 m
In 450 m race C beat B in (13×450)/282= 20.74 m(ans)
...........#PintuPalit
81.A man can row 30km upstream & 44km downstream in 10hr.He
can also row 40km upstream & 55km downstream in 13hr.Find the
rate of current & the speed of the man in still water?
Ans with explanations:
Let,
Downstream speed = x kmph & upstream speed =y kmph
1st condition,
44/x + 30/y = 10--------(1)
2nd condition,
55/x + 40/y = 13 --------(2)
After solving, x = 11 kmph & y = 5 kmph
Speed of current = (11-5)/2=3 kmph
Speed of man in still water = (11+5)/2=8 kmph(ans)
..............#PintuPalit
82.The list price of a commodity is the price after a 20% discount
on the retail price. The festival discount price on the commodity is
the price after a 30% discount on the list price. Customers
purchase commodities from stores at a festival discount price.What
is the effective discount offered by the stores on the commodity on
its retail price?
Ans with Explanations:
Let,
Retail price =100 tk
List Price = 100-100×20/100=80 tk
Festival price after discount 30% on list price = 80-80×30/100= 56 tk
Effective discount = (100-56)×100/100=44
Ans: 44%
...........#PintuPalit
83.Two people start jogging at the same point & time but in
opposite directions.If the rate of one jogger is 3mph faster than the
other & after 2 hours they are 30 miles apart,what is the rate of the
faster jogger?
Ans with explanations:
let,
One jogger speed = x kmph
2nd jogger speed = x+3 kmph
Effective speed = x+x+3=2x+3 kmph
Atq,
30/(2x+3)=2
=>x= 6
So,Faster jogger speed =6+3=9 kmph(ans)
...........#PintuPalit
84.In how many ways can the letters of the word 'Maxima' be
arranged such that all vowels are together & all consonants are
together?
Ans with explanations:
Here,
'Maxima' has the vowels are ( a, i, a ) & the consonants are (m, x, m)
Then, the following situations arise :-
(a) (vowels) then (consonants ); and
(b) (consonants) then (vowels).
So,The vowels can be arranged in
= 3!/2!
= 3*2*1/2*1
= 3 ways
& The consonants can be arranged in
= 3!/2!
= 3 ways
So situation (a) can be arranged in
= 3*3
= 9 ways
Again,situation (b) can be arranged in
= 3*3
= 9 ways
Therefore, required arrangements is
= 9+9
= 18 ways( Answer)
............#SamratRoySandhan
85.A bowl contains only marbles. If 1/4 th of the marbles were
removed,the bowl would be filled to 1/2 of it's capacity.If 100
marbles were added,the bowl would be full.How many marbles are
in the bowl?
Ans with explanations::
Let,
x number of marbles were at 1st
After removing we have=x-(x/4)=3x/4, which is half of the box.
So,full box contains doubles of these marbles=3x*2/4=3x/2
Again,100 more marbles will fill the box fully & we have x marbles in the box
now.
ATQ;
x+100=3x/2
=>x=200
So,200 marbles were initially there.(ans)
............#RubelChowdhury
86.If two cards are drawn at random from a deck of 52 cards,what is
the probability that both cards will be from same suit?
Ans with explanations:
a pack of cards has 4 suits. every suit has 13 cards.
when, the first card is picked, it could be one from any of 4 suits.
so, the probability of the first card picked is 13/52.
as the second card has to be from the same suit, it will be picked from the suit
first card has been picked from.
so, the suit from which first card has been picked has 12 cards remaining
now, the probability of the second card picked is 12/51
finally, the probability of picking two cards from the same suit is
(13/52)×(12/51)=1/17
but, this has been calculated for only one suit. there are 4 suits in total.
now, you have to multiply (1/17) by 4(answer)
..............#NilChad
87.If distinct numbers X,Y,Z & P are chosen from the numbers -
2,2,1/2,-1/3,what is the largest possible value of the expression
(x^2*y)/(z-p)?
Ans with explanations:
Given that,
x= -2
y= 2
z=1/2
p=-1/3
ATQ;
x^2*y = 4*2 = 8
& z-p = 1/2+1/3
= 5/6
So,the required answer = 8/(5/6)= 8*6/5= 48/5(ans)
..........#SamratRoySandhan
88.Tom & bill agreed to race across a 50 ft pool & back again.They
started together but Tom finished 10 ft ahead of Bill.If their rates
were constant & Tom finished the race in 27 seconds, how long did
Bill take to finish it?
Ans with explanations:
Given that,
Total distance covered by Tom in 27 seconds =50*2=100ft
so, total distance covered by Bill in 27 seconds =100-10=90 ft
Here,
Bill covered 90 ft in 27 sec
So, he covered 100 ft in 27*100/90=30 sec(ans)...........#SaiduzzamanTitu
89.A salesman's commission is 5% on all sales upto tk 10000 & 4%
on all sales exceeding this.He remits tk 31100 to his parent
company after deducting his commission,find the total sales?
Ans with explanations:
Let,
His total sales be x tk
ATQ;
Total sales-commission=31100
or,x-[5% of 10000+4%of(x-10000)]=31100
Or, x-[5/100(10000)+4/100(x-10000)=31100
Or, x-x/25=31200
Or, x = 32500
So,Total sales=32500 tk(ans)
..............#RubelChowdhury
90.Two cars race arround a circular track in opposite directions at
constant rates.They start at the same point & meet every 30
seconds.If they move in the same direction,they meet every 120
seconds.If the track is 1800 meter long,what is the speed of each
car?
Ans with explanations:
Let,
Speed of the fastest car and slowest car be x and y m/s respectively
ATQ;
1800/(x+y)=30
=> x+y = 60------------(i)
and 1800/(x-y)=120
=> x-y = 15--------------(ii)
By solving (i) and (ii)
x=37.5 m/s or 37.5*18/5= 135km/h
y=22.5 m/s or 22.5*18/5=81 km/h(ans)
..............#SaiduzzamanTitu
91.A rectangular park 60 m long & 40 m wide, has two concrete
crossroads running in the middle of the park & rest of the park has
been used as a lawn.If the area of the lawn is 2109 sqm,then what is
the wide of the road?
Ans with explanations:
Given that,
Length= 60 m
Width= 40 m
So, Area of the rectangular park is
= (60 x 40) m^2
= 2400 m^2
Given that,
Area of the lawn is= 2109 m^2
Then, Area of the crossroads is= (2400 - 2109) m^2= 291 m^2
Let, the width of the road be x metres.
Area of road along length= 60x m
Area of road along width= 40x m
Area of the road will be=( 60x + 40x - x^2) m^2
Accordingly,
60x+40x-x^2= 291
=> 100x - x^2 -291 = 0
=>x^2 - 100x + 291= 0
=> x^2 - 97x - 3x + 291=0
=> (x-97)(x-3)=0
X= 97 not acceptable because width of road can't greater than length.
Therefore, Length,x= 3 m(ans)
..............#SamratRoySandhan
92.Ayesha's father was 38 years of age when she was born.While
her mother was 36 years of age when her brother 4 years younger
to her was born.What is the difference between the ages of the
parents?
Ans with explanations:
Given that,
Ayesha's age at born time =0 yr
Father's age =38 yrs
Ayesha's brother age at born time = 0 yr.
Ayesha's age = 4 yr
Ayesha's mother age = 36 yrs
So, when Ayesha is 4, father is =38+4=42 yrs
Difference =42-36 =6 years(ans)
.................#ProsenjitChandraRoy
93.A can do a work in 20 days,B in 20 days & C in 60 days.From the
first day, every 3rd day B & every 4th day C helps A.In how many
days they will be done the work?
Ans with explanations:
ATQ;
A's 2 days work= 1/20+1/20=1/10
In 3rd days work by A & B will be= 1/20+1/20=1/10
In 4 th days work by A & C will be= 1/20+1/60= 4/60= 1/15
Now, In 4 days A, B & C can do the parts of work= (1/10+1/10+1/15)
= 8/30
Then, In 12 days parts of work by A, B & C will be=( 8/30+8/30+8/30)
= 4/5
In 14 days the parts of work will be=(4/5+1/10)=9/10
So,Remaining work= 1-27/30= 3/30= 1/10, which work can be done by (A+B)
in 1 day.
Now,Total days= 14+1= 15 days(ans)
..........#SamratRoySondhan
94.A train which travels at a uniform speed, due to some
mechanical fault after travelling for an hour goes at 3/5 th of the
original speed & reached the destination two hours late.If the fault
had occurred after travelling another 50 miles,the train would have
reached 40 mins earlier.What is original speed & the difference
between the two stations?
Ans with explanations:
Let,
The original speed be x mph & 3/5th of original speed be 3x/5 mph
As,we have,time=distance/speed
ATQ;
50/(3x/5)-50/x=40/60
=>x=50
So,3/5 th of 50 mph=30 mph & speed reduces by (50-30)=20 mph
Though,it is late by 2hrs & by this time it goes 30*2=60 miles
If the train goes with it's original speed,it took 60/20=3 hrs after the 1st
hour.That means total 4hrs.
Therefore,total distance covered 4*50=200 miles & speed is 50 mph(ans)
.........#RubelChowdhury
Alternate Solution:
Suppose,
The usual speed of the train is 5x mph
& The reduce speed of the train is 3x mph
For 50 miles,
ATQ;
50/3x - 50/5x =40/60=2/3
=>x = 10
So,the usual speed is 50 mph & reduce speed 30 mph
Let,
The distance is d miles
ATQ;
d/30 - d/50 = 2
=>d = 150
So,the distance between the two stations is 150+50=200 miles(ans)
..............Rubel Chowdhury
95.A train running at 36kmph passes another train completely in 12
seconds which is half of its length, running in the opposite
direction at 54kmph.If it also passes a railway platform in 1.5
mins.What is the length of the platform (in meter)?
Ans with explanations:
Let,
First train's length=x m
2nd train's length = x/2 m
Relative speed = (36+54) km/hr = 90 km/hr = 25 m/s
Total length of two train = x + x/2 = 3x/2 m
ATQ;
(3x/2)/25 = 12
=>x =200
So, first train's length 200 m
Again,
The length of platform = y m
First train speed = 36 km/hr = 10 m/ s
ATQ-2,
(y+200)/ 10 = 1.5*60
=>y = 700
So, The length of the platform = 700 m(ans).
.............#ProsenjitChandraRoy
96.A company has issued 2lacs ordinary shares of tk 10 each &
50000 debentures of tk 100 each.The debenture increased of 10%
per annum.In one year, after paying the interest of 10% on the
debentures & 9% dividend on the ordinary shares.Its profits are just
sufficient to put tk 2lacs to reserve.What are the profits for that
year?
Ans with explanations:
Total price of ordinary shares = (200000*10) tk = 2000000 tk
Total price of debentures = (50000*100) = 5000000tk
At 9% interest on ordinary share,makes interest =( 9*2000000/100)tk
= 180000tk
At 10% interest on debenture, makes interest =(10*5000000/100) tk
= 500000tk
Total profit of the company =(180000+500000+200000)= 880000tk
(Answer)
.............#ProsenjitChandraRoy
97.A sales person's monthly salary is tk 5000.He earns as
commission 5% on all sales between tk 20000 & tk 59999 and 8 %
on sales exceeding tk 60000.In adding he gets fixed bonus of tk
1000,if his monthly sales exceeds tk 10000.If in a month his total
income is tk 10400 what was his total sales?
Ans with explanations:
Given that,
Total income =10400tk
Monthly salary =5000tk
Fixed Bonus = 1000 tk
So, Income on commission = 10400- 5000-1000 = 4400tk
Let,
X tk sales above 60000 tk.
ATQ,
40000*5% + x*8% = 4400
=>x= 30000tk
Now, total sales = 20000+ 40000+30000 = 90000tk(ans)
............Sohel Rana
98.Kasem joined at a work 2 years ago.His starting gross income
was 1/2 of Selim's gross income at that time.Each year since then
Kasem has received a raise of 5% in his annual basic salary,which
is 70% of his gross income.His allowance which are 30% of his
gross income,have remain unchanged.Meanwhile Selim has
received a raise of 10% in his basic,which is 80% of his gross
income.His allowance which are 20% of his gross income,have
remain unchanged.Curently what % of Selim's gross income is
Kasem's current gross income?
Ans with explanations:
Let,
Kashem's Gross Income 2 yrs ago be x tk & Selim's Gross Income 2 yrs ago
be 2x tk
Here,
2 yrs ago Kashem's basic salary was = x*70/100 = 0.7x tk
At 5% raise,Kashem's annual basic salary now
= 0.7x*{1+(5/100)}^2=0.77175x tk
So,Kashem's current gross income
= 0.77175x+(x*30/100)=1.07175x tk
On the other hand,
2 yrs ago Selim's basic salary was = 2x*80/100 = 1.6x tk
At 10% raise, his basic salary now = 1.6x{1+(10/100)}^2=1.936x tk
So, Selim's current gross income = 1.936x +(2x*20/100) = 2.336x tk
ATQ;
(p/100)*2.336x = 1.07175x
=>p = 1.936*100/2.336
=>p = 45.879%(ans)
.........#RubelChowdhury
99.Suppose you deposit tk 10000 on January 01,2012 at 12.5%
interest rate for 1 year, on July 01,2013 tk 15000 at 12% intrest rate
for 6 months & on Oct 01,2013 tk 20000 at 11.5% interest rate for 3
months.Suppose you withdrew all deposits on Dec
31,2013.Calculate overall annual rate of interest you have received?
Ans with explanations:
Total interest Earned
=(10000*12.5/100)+(15000*12/100*2)+(20000*11.5/100*4)
=(1250+900+575) tk
=2725 Tk
Total deposited amount
=10000+(15000*6/12)+(20000*3/12)
=(10000+7500+5000) tk
=22500 Tk
Annual interest rate=2725*100/22500=12.11%(ans)
..............#YousufAli
100.A contract is to be completed in 46 days & 117 men were set to
work,each working 8 hours a day.After 33 days 4/7 of the work is
completed.How many additional men may be employed so that the
work may be completed in time.Each man now working 9 hours a
day?
Ans with explanations:
Remaining parts of work = (1-4/7) = 3/7 & Time = 46-33=13 days
33 days working 8 hours a day = 33×8hours= 264 hours
13 days working 9 hours a day = 13×9= 117 hours
So,In 264 hrs to complete 4/7 work by 117 men
In 117 hrs to complete 3/7 work by = (264×7×3)/(4×7)= 198 men
Additional men = 198-117=81 men (ans)
..........#PintuPalit
101.A,B & C start a business, each investing tk 20000.After 5
months A withdrew tk 5000,B withdrew tk 4000 & C invests tk 6000
more.At the end of the year a total profit of tk 69900 was
recorded.Find the share of each?
Ans with explanations:
A's 1 year investment =20000×5+(20000-5000)×7=205000 tk
B's 1 year investment = 20000×5+(20000-4000)×7= 212000 tk
C's 1 year investment = 20000×5+(20000+6000)×7= 282000 tk
A:B:C= 205000:212000:282000=205:212:282
Sum of the ratio = 205+212+282=699
So, A 's share = 69900×205/699=20500 tk
B's Share = 69900×212/699=21200 tk
C's Share = 69900×282/699=28200 tk
Ans : 20500tk. , 21200 tk & 28200 tk
..........#PintuPalit
102.Robi was 4 times as old as his son 8 years ago.After 8 years,Robi
will be twice as old as his son.What are their present age?
Ans with explanations:
Let,
8 years ago son's age = x years & Robi's age was = 4x years
Present age of son = x+8 years & present age of Robi = 4x+8
Atq,
4x+8+8=2(x+8+8)
or, 4x+16=2x=32
Or, x= 8
Present age of son = 8+8=16 years
Present age of Robi = 4×8+8=40 years (ans)
..........#PintuPalit
103..A dishonest businessman makes a profit of 10% both on
buying & selling of a commodity. If his profit is tk 1050,what is the
cost of the commodity?
Ans with explanations:
Let,
Cost Price of the article=100 Tk
10% profit on cost price then Market price=(100 + 100 of 10%) tk
=110 Tk
Again,
10% profits on market price then selling price=(110+110 of 10%) Tk
=121 Tk
Now,
Profit=(121-100)=21 Tk
When Profit 21 Tk then Cost Price=100 tk
When profit 1050 Tk then cost price =(1050*100/21) tk
=5000 tk(ans)
............#YousufAli
104.There are 200 questions in a 3 hour exam.Among these 50 are
mathmatics problems. It is suggested that twice as much time be
allowed for each mathmatics problems as for the other
questions.How many minutes should be spent on the mathematics
problems?
Ans with explanations:
Given that,
Total questions=200
Mathematics =50
Others=150
Let, other questions take = x min
ATQ,
2x*50+150x=180
=> x=18/25
Here,Total time, need to solve 50 mathmatics problems=2*18*50/25=72
mins(ans)
...............#RubelChowdhury
105.Two alloys A & B are composed of two basic elements.The
ratios of the compositions of the two basic elements in the two
alloys are 6:4 & 2:3 respectively. A new alloy X is formed by mixing
the two alloy A & B in the ratio 4:3,what is the ratio of the
composition of the two basic elements in alloy X?
Ans with explanations :
Let, Actual amount of A =4x
& Actual amount of B=3x
Since, A & B are in the ratio =4:3
So,in the X,
Amount of first elements = 6/10*4x + 2/5*3x = 18x/5
Amount of second elements = 4/10*4x. + 3/5*3x = 17x/5
So, Ratio of 2 elements = 18:17(ans)
..........Prosenjit Chandra Roy
106.The total income of Mr.Kamal in the years 2013,2014 & 2015
was tk 62400.His income increased by 20% each year.What was his
income in 2015?
Ans with explanations :
Let,
Total income of Mr. Kamal in the year of 2013 be tk.100x.
and In the year 2014 the income was tk.100x*120% = tk. 120x.
In the year 2015 it was tk. 120x*120% = tk. 144x
ATQ,
100x+120x+144x = 62400
=> 364x =62400
=> x = 171.42857
:. In the the year of 2015 the income was (171.42857*144) tk
= 24685.714 tk(ans)
..........#GikoGiko
107.Joan wishes to mix some tea worth $1.40 per pound with
another type of tea worth $2.25 per pound in order to make a 10
pound mixture worth $1.74 per pound.How many pounds of the
$1.40 tea should she use?
Ans with explanations:
Since the grocer is mixing the first two together to get the third, you can write
an equation and solve for x:
ATQ;
1.40x + 2.25(10-x) = 1.74*10
=>1.40x + 22.5-2.25x = 17.4
=>-0.85x = -5.1
=>x = 6
So, the grocer should use 6 pounds of $1.40 tea, she should use(ans).
.............#RubelChowdhury
108.In covering a distance of 30km Abhey takes two hours more
than Sameer.If Abhey doubles his speed,then he would take 1 hour
less than Sameer.What is the speed of Abhey?
Ans with explanations :
Given that,
Total distance=30km
Let,
Sameer takes =x hours & Abhey takes = (x+ 2) hours
Speed of Sameer =30/x kmph & Abhey=30/(x+2) kmph
If Abhey doubles his speed, the new speed will be 60/(x+2) kmph & time taken
= x-1 hours
As,we have,distance=speed*time
ATQ;
30= 60*(x-1)/(x+2)
=>x= 4
So,Abhey's speed = 30/(4+2) = 5 kmph(ans)
..................#RubelChowdhury & #NilufarYasmin
109.Masud can do a job in 40 days working alone.Karim can do the
same job in 60 days working alone.They start working together &
worked for 18 days,when Harun joined them.The three man worked
together & finished the job in 5days.In how many days could Harun
do the whole job working alone?
Ans with explanations:
Masud & Karim do the parts of job in 1 day=(1/40+1/60)=1/24
Masid & Karim do the parts of job in 18 days=18/24=3/4
Rest of the work = 1-3/4=1/4
All three together complete the 1/4 parts in 5 days
They complete 1 part in 5*4/1 =20 days
So,Harun complete the parts of work alone in 1 day
=(1/20-1/24)=1/120
Thus,Harun can complete the whole work in 120 days(ans)
..........#RubelChowdhury
110.Nafisa is 36 years old.She is twice as old as Shammi was when
Nafisa was as old as Shammi now.How old is Shammi now?
Ans with explanations:
Let Shammi's age be"a" years.
Given that,
Nafisa's age is 36 years , she was of Shammi's age (36 - a) years ago.
At that time, the age of Shammi was = current age - time difference
= a - (36 - a) = 2a - 36
Now, Nafisa is now twice as old as she was (36- a) years ago.
Thus, 36 = 2 (2a - 36)
Or, 36 = 4a - 72
Or, 4a = 108
Or, a = 27.
So, Shammi is 27 years old now.(ans)
...........#RubelChowdhury
Alternate Solution:
Let,
Shammi is x years old
and , Nafisa's age of y year ago = Shammi's present age
so, y year ago Nafisa was (36-y) years old
and Shammi was (x-y) years old
ATQ,
36 = 2(x-y)
=> (x-y) = 18.....(i)
Again,
(36 - y) = x
=> y = 36 - x.....(ii)
putting value of y in equation (i),
x - (36 - x) = 18
=> x - 36 + x = 18
=> 2x = 54
=> x = 27
So, Shammi is 27 years old now.(ans)
...........#AsmaBinteSalahuddin
111.A machine P can print 1lac books in 8hrs, Q can same in 10hrs
& R can print them in 12hrs.All the machines are started at
9am,while Machine P is closed at 11am & the remaining two
machines complete the work.Approximately at what time the work
will be finished?
Ans with explanations:
P, Q & R together do the parts of work in 1hr=(1/8+1/10+1/12)=37/120
As,they begin the work at 9 a.m. & P stops at 11 a.m.
So,they complete the parts of work in 2hrs=37*2/120=37/60
Rest of the work = {1-(37/60)}=23/60, which will complete by Q & R
Q & R complete parts of work in 1 hr =(1/10+1/12)=11/60
So,they complete 11/60 parts in 1 hr
Thus,They complete 23/60 parts in (60*23/11*60)=23/11 hr=2.09hrs
So,the total work will be finished in =11:00 a.m. + 2 hr=1:00 pm(ans)
................#RubelChowdhury

112.A father said to his son,"I was as old as you are at the time of
your birth".If the father's age is 38 years now,the son's age 5 years
back was?
Ans with explanations:
Let,
Son's age=0
Present age of Son= x yrs
Father age present=2x yrs
ATQ,
2x=38
=> x=19 yrs
So,Five yrs ago son age was=19-5=14 yrs(ans)
............#SumonShahidul
113.Each person in a group of 110 investors has investments in
either equities or securities or both.Exactly 25% of the investors in
equities have investments in securities & exactly 40% of the
investors in securities have investments in equities.How many
have investments in equities?
Ans with explanations:
The investors can be categorized into three groups:
(1) Those who have investments in equities only.
(2) Those who have investments in securities only.
(3) Those who have investments in both equities and securities.
Let ,
x, y, and z denote the number of people in the respective categories. Since
the total number of
investors is 110, we have
x + y + z = 110 ------------- (1)
Also,
The number of people with investments in equities is x + z
and
The number of people with investments in securities is y + z.
Since exactly 25% of the investors in equities have investments in securities,
we have the
equation
25% of (x+z)=z
x = 3z ------------------- (2)
Since exactly 40% of the investors in securities have investments in equities,
we have the
equation
40% of (y+z)=z
y = {3z}/2 -------------------- (3)
Substituting equations (2) and (3) into equation (1) yields
3z+(3z/2)+z=110
===============
Or, Z=20
Hence,
the number of people with investments in equities is:
=x+z
=3z+z
=3×20+20
=60+20
=80(Answer)
...................#YousufAli
114.A & B are two alloys of gold & copper prepared by mixing
metals in proportions 7:2 & 7:11 respectively.If equal quantities of
the alloys are melted to form a third alloy C,then what will be the
proportion of the gold & copper in C?
Ans with explanations:
In the first alloy A,
The amount of gold= 7/9
The amount of copper= 2/9
In the second alloy B,
The amount of gold= 7/18
The amount of copper= 11/18
When we mix both of them together the weight of gold
=(7/9+7/18)=21/18=7/6
The weight of copper=(2/9+11/18)=15/18=5/6
Since, two alloys A & B are melted to form the third alloy C,
So, weight of gold in C=7/6* 1/(1+1)=7/6*1/2=7/12
& weight of copper in C=5/6* 1/(1+1)=5/6*1/2=5/12
In alloy C the ratio of Gold & copper will be,
Gold:Copper= 7/12 : 5/12= 7:5(ans)
.........Samrat Roy Sondhan
115.Of the total number of oranges that arrive at a juice company,
20% by weight is rejected before processing. Of the oranges that
are processed,15% by weight rejected before being juiced.Of the
oranges that are juiced,25% are wasted as a result of juice
making.Of the oranges those arrive at the company,what % by
weight is being?
Ans with explanations:
Let,
Total number of oranges are 100
As 20% rejected before processing,
So the number of oranges being processed = (100-100*20/100)=80
As 15% processed oranges rejected before being juiced
So,the number of oranges being juiced =(80-80*15/100)=68
As 25% juiced oranges are wasted during juice making,
So,the rest of the number of oranges = (68-68*25/100)=51
Thus, 51% of the oranges are arrived at the company.(ans)
............#RubelChowdhury
116.In an acoustics class, 120 students are male & 100 students are
female.25% of the male students & 20% of the female students are
engineering students.20% of the male engineering students & 25%
of the female engineering students passed the final exam.What %
of engineering students passed the exam?
Ans with explanations:
Total Male Students=120
Male Engineer Students=25*120/100=30
Male Engineer Pass=20*30/100=6
Female Students=100
Female Engineer Students=20*100/100=20
Female Engineer pass=25*20/100=5
Total pass=6+5=11
Total Engineer Students =30+20=50
So,required % =11*100/50=22%(ans)
............#RubelChowdhury
117.When a producer allows 36% commission on the retail price of
his product, he earns a profit of 8.8%.What would be his profit % if
the commission is reduced by 24%?
Ans with explanations:
Let,
The marked price of the product be 100 tk
After allowing 36% commission, the retail price of the product will be=100-
36=64 tk
So,his selling price is 64 tk
At 8.8% profit,
If selling price is tk 108.8 then cost price will be tk 100
If selling price tk 64 then cost price will be=100*64/108.8
=58.8235 tk
After reducing commission by 24%,the actual commission will bee=(36-
24)%=12%
So,the selling price =(100-12)=88 tk
Profit=(88-58.8235)=29.1765 tk
So,profit %=29.1765*100/58.8235=49.6%(ans)
..........Rubel Chowdhury
118.A can do a piece of work in 10 days while B alone can do it in
15 days.They work together for 5 days & the rest of the work is
done by C in 2 days.If they get tk 4500 for the whole work, how
should they divide the money?
Ans with explanations:
A can complete in 10 days 1 part
So,A can complete in 5 days 5/10=1/2 part
Accordingly,B can complete in 5 days 5/15=1/3 part
Now,A & B can complete together in 5 days=(1/2+1/3)=5/6
Rest of the work = 1-5/6 = 1/6
ATQ;
C does 1/6 th portion of work in 2 days
Thus,A,B & C's ratios of parts of work = 1/2:1/3:1/6 = 3:2:1
Now,A gets = 3*4500/6= 2250 tk
B gets = 2*4500/6 = 1500 tk
& C gets = 1*4500/6 = 750 tk
So,they should divide the money among three's respectively 2250,1500 & 750
tk(ans)
............#RubelChowdhury
119.A boat sails M miles upstream at the rate of R mph.If the rate of
the stream is S mph. How long will it take the boat to return to it's
starting point?
Ans with explanations:
Let,
Boat speed be X mph
Given that,
Stream speed is S mph & total distance M miles
ATQ;
Upstream speed, X - S = R
=>X = S + R
Hence,Downstream(return) speed = (X + S) mph
As, we have, time = distance/speed
So,the boat takes time to return it's starting point
= M/(X+S)= M/(S+S+R)= M/(2S+R) (ans)
............#RubelChowdhury
120.Avg age of 18 men is decreased by 1 year when 1 man whose
age is 49 years is replaced by a new man.Find the age of the new
man?
Ans with explanations:
Let,
The age of the new man be x years.
After replacing new man total decrease in age is 18*1=18 years.
ATQ,
49-x = 18
=> x = 31
:. the age of new man is 31 years.(Ans)
...........#GikoGiko
Alternate Solution:
Let,
Average age=x yrs
New man's age=y yrs
ATQ;
18x+y=18(x-1)+49
or, y= 31 years
So,the age of new man is 31 years(ans)
..........Sohel Rana
121.If you add up the digits of a three digits number you get 18.The
tenth digits is twice the hundredth digits & the hundredth digit is
1/3rd the unit digits.What is the number?
Ans with explanations:
Let,
Unit digit= 3x
Tenth digit= 2x
Hundredth digit= x
ATQ,
3x+2x+x=18
=>x=3
Unit digit=3*3=9
Tenth digit=2*3=6
Hundredth digit=3
So,the number is=369(ans)
.........#RubelChowdhury
122.A man takes loan of 500 tk at 5% interest for some time.After
some period he again takes 400 tk loan at 3.5% interest rate.After 6
months of taking his 2nd loan, he paid the both loans with a total of
994.50 tk with interest.How many days after taking the first loan he
paid the loans?
Ans with explanations:
Given that,
Total capital=500+400=900
Total interest=994.5-900=94.5 tk
We have,Interest=Capital*Interest rate*time
So,Interest for 400 tk in 6 months=(400*3.5*1/100*2) tk =7 tk
Hence,Interest for 500 tk =(94.5-7) tk =87.5 tk
At 5% intetest rate,
ATQ;
87.5=500*5*n/100
=>n=3.5
Thus,After taking the 1st loan,he paid it in 3.5 years(ans)
..........#RubelChowdhury
123.A seller bought some pens @5tk for tk 100.,those were
separated into 2 stacks & one of which was sold @3tk for 100 tk &
the other @6tk for 100 tk.What was the ratio of pens number in 2
stacks if there is no loss no profit?
Ans with explanations:
Let,
Number of tk.3 type pen = a
& Number of tk.6 type pen = b
There is no loss no profit, average price=Tk.5
ATQ;
5 = (3a + 6b)/(a+b)
=> 2a = b
=> a/b = 1/2
So, the ratio is 1 : 2 (Ans)

............#AsmaBinteSalahuddin

124.Fresh grapes contains 90% water by weight while dried grapes


contains 20% water by weight.What is the weight of dry grapes
available from 20kg of fresh grapes?
Ans with explanations:
Let,
x kg dry grapes available from 20 kg of fresh grapes.
Fresh grapes contain (100-90) = 10% solid grapes
& Dry grapes contain (100-20) = 80% solid grapes.
Then, solid part in fresh grapes = solid part in dry grapes.
ATQ,
20*10% = x*80%
=>x = 2.5
So, the weight of dry grapes available from 20kg of fresh grapes is 2.5kg.
(ans)
..............#GikoGiko
125.A brand of paints costs tk 14 per litre & 1 litre of paint will cover
an area of 150 sqft.What is the minimum cost of paint needed to
cover the 4 walls of a rectangular room that is 12 ft wide,16 ft long
& 8 ft high?
Ans with explanations:
Area of 4 wall = 2*(wide+length)*height= 2*(12+16)*8=448 sqft
Liter of paint is needed = 448/150=2.98 litre = 3 liter(approximate)
So, minimum cost of paint needed to cover the 4 walls
= 3*14= 42 tk(ans)
...........Saiduzzaman Titu
126.Danish & Sam who live 10 miles apart,meet at a cafe that is
directly north of Danish's house & directly east of Sam's house.If
the cafe is 2 miles closer to Danish's house than to Sam's
house,how many miles is the cafe from Sam's house?
Ans with explanations:
Let,
The cafe is x miles from Danish's house.
So, it is (x+2) miles from Sam's house
As the cafe is directly north and east from Danish's and Sam's house, So, it
forms a RIGHT angle .
So, x^2 + (x+2)^2 = 10^2
=> (x+8) (x-6) = 0
=> x = 6
So, the cafe is (6+2)=8 miles from Sam's house. (Ans)
.........#AsmaBinteSalahuddin
127.In a cricket tournament, Team Dhaka has won 30 games out of
50 played.It has 46 more games to play.In order to maintain an avg
of 75% victory rate,then Dhaka has to win how many more games?
Ans with explanations:
Given that,
Already played=50 games
Has to play more=46 games
So,total games=50+46=96
At 75% victory rate,
In 100 games One has to win in total 75 games
So,in 96 games One has to win in total 96*75/100=72 games
Dhaka has won 30 games, so it has to win more
=72-30=42 games(ans)
..........#RubelChowdhury
128.Avg age of 7 members of a family is 29 year.If present age of
the youngest member is 5 year,find avg age of the remaining
members at the time of birth of youngest member?
Ans with explanations:
Given that,
Total age=7*29=203 years
5 years ago total age=203-(5*7)=168 years
Avg age of remaining member at the time of birth of youngest
member=168/6=28 years(ans)
..........Sohel Rana
129.The avg marks obtained by some students in an exam is 54.If
20% of the students got a mean score of 90 marks & the 30% of the
students got a mean score of 20.Find the avg marks of the
remaining students?
Ans with explanations:
Let,
Total students =x
Total marks=54x
Total marks of 20% students =0.20x*90=18x
Total marks of 30% students =0.30x*20=6x
Remaining marks =54x-18x-6x=30x
Remaining students =x-.20x-.30x=0.50x
Avg marks of remaining students=30x/0.50x=60(ans)
............#SohelRana
130.The elevator in an 11 storied office building travels at the rate
of 1 floor per 1/4 mins,which allows time for picking up & letting
down passenger.At the main floor & the top floor, the operator
stops for 1 minute.How many complete(with return) trips will the
operator make during a 7hr period?
Ans with explanations:
Here in 11 storried building lift traveled for 10 floor and stay 1 min in groung
floor. so total time need for 1 round with return
= (1+1/4*10)*2= 7min
Total time = 7*60=420 min
Required trip= 420/7= 60 trip(ans)
............Saiduzzaman Titu
31.The water in a fish tank measuring 15 inches by 8 inches is 7
inches high.The water is poured into another tank measuring 40
inches by 20 inches.What will be the height of water in inches in the
2nd tank?
Ans with explanations:
Given that,
The length,width & height of 1st water tank are 15, 8 & 7 inches respectively.
& the length and the width of 2nd water tank are 40 & 20 inches
Let, the height of 2nd water tank is h inches
ATQ;
15*8*7 = 40*20*h
=>h = 1.05
So,the height of the 2nd water tank is 1.05 inches(ans)
..........Ayrin Hasan
132.A & B together can do a piece of work in 12 days,which B & C
together can do in 16 days.After A has been working at it for 5 days
& B for 7 days C finishes in 13 days.In how many days C alone will
do the work?
Ans with explanations:
(A+B) together do for 5 days [as A do the work for 5 days]
(B+C) together do for (7-5)=2 daya [ as B already work with A for 5 days]
C alone do the work for 13-2= 11 days [ as C already work with B for 2 days)
so (A+B)'s 5 days work=5/12
(B+C)'s 2 days work= 2/16
Let Ccan do the work in x days
so C's 11 days work = 11/x
ATQ, 5/12+2/16+11/x=1
=>x=24
So,C alone can do the work in 24 days(ans)
........Saiduzzaman Titu
133.Two merchants sell, each an article for tk 1000.If merchant A
computes his profit on cost & merchant B computes on selling
price,they end up making profits of 25% respectively. By how much
profit made by merchant B greater than that of merchant A?
Ans with explanations:
Profit for Merchant A= 1000*25/125= 200 [ as profit computes on cost price]
Profit of Merchant B= 1000*25/100= 250 [as profit computes on selling price]
So,The Difference between the profit of Merchant A & B =250-200=50(ans)
..........Saiduzzaman Titu
134.A light system can be emitted by the intensity of candle.The
intensity of light is equivalent to 64 candles when the candles are 2
meter apart.According to optical physics, the intensity is inversely
proportional to the square of the distance.So when candles are 8
meter apart,find the intensity equivalent to the numbers of
candles?
Ans with explanations:
In 2 meters intensity equivalent to 64 candles
so in 8 meters intensity equivalent to 64*8/2= 256 candles(ans)
.......Saiduzzaman Titu
135.Rita produced two products plain cake & chocolate cake.Of the
cakes produced last month 3/5 th was plain cake & rests were
chocolate cakes.It takes 7/5 th as many hrs to make a chocolate
cake as it does to make a plain cake.If last month Rita worked for a
total of 58hrs,how many hrs did she spend for making chocolate
cakes?
Ans with explanations:
Let,
The numbers of plain cake = 3x
& the numbers of chocolate cake = 2x
Time required for making a plain cake = y hrs
time required for making a chocolate cake = 7y/5 hrs
ATQ,
3xy+2x*7y/5=58
=>xy=10
so she spend for making chocolate cake
= 2x*7y/5= 2*7*10/5= 28 hrs(ans)
........Saiduzzaman Titu
136.A 20 litre mixture of milk & water contains in the ratio of 3:2.10
litres of the mixture is removed & replaced with pure milk and the
operations is repeated once more.At the end of the two removals &
replacement,what is the ratio of milk & water in the resultant
mixture?
Ans with explanations:
According to the given ratio,
Amount of water in the mixture = 20*3/5 = 12 litres
Amount of milk in the mixture = 20*2/5 = 8 litres
After removal of 10 litres from mixture,
The amount of milk will be = 10*3/2 =15 litres
& the amount of water will be = 10*2/5 = 4 litres
After pouring 10 litre pure milk into the mixture,
The amount of milk in the new mixture will be =10+6=16 litres
& the amount of water will be remain unchanged & that is 4 litres.
Now,the ratio of water & milk in the new mixture will be = 4:16 =1:4
After removal again 10 litres from mixture,
The amount of milk will be = 10*4/5 = 8 litres
& the amount of water will be = 10*1/5 = 2 litres
Now, after addition of 10 litres pure milk in the mixture,
The new amount of milk will be = 8+10 = 18 litres
& the water remain unchanged,which is 2 litres
So,the ratio of milk & water in the resultant mixture is 18:2=9:1(ans)
........#RubelChowdhury

137.A contract is to be completed in 56 days if 104 persons work,


each working at 8 hours a day. After 30 days, 2/5 of the work is
completed. How many additional persons should be deployed so
that the work will be completed in the scheduled time,each persons
now working 9 hours a day?
Ans with explanations:
Persons worked (M1) = 104
Number of hours each person worked per day (H1) = 8
Number of days they worked (D1) = 30
Work completed (W1)= 2/5
Remaining days (D2)= 56 - 30 = 26
Remaining Work to be completed (W2)=1 - 2/5 = 3/5
Let,
The total number of persons who do the remaining work (M2) = x
Number of hours each person needs to be work per day (H2) = 9
ATQ;
M1xD1xH1/W1=M2xD2xH2/W2
⇒104×30×8/(2/5)=x×26×9/(3/5)
⇒104×30×8/2=x×26×9/3
⇒52×30×8=x×26×3
⇒2×30×8=3x
⇒x=2×10×8=160
So, Number of additional persons required = 160 - 104 = 56(Ans)
.........#GikoGiko
Alternate Solution:
Given that,
Persons worked=104
Number of hrs each person worked per day=8
Number of days they worked=30
Remaining days=56-30=26
Work completed=2/5 th
Remaining work to be completed=1 - 2/5 = 3/5
ATQ-1;
Amount of work 104 persons did in 30 days working 8hrs a day = 2/5
Amount of work 1 person did in 1day, working 1 hr a day =2/5*104*30*8 =
1/62400
Let,Total number of persons who do the remainig work = x
Given,Number of hrs each person needs to be work per day=9 hrs
Now,
Amt of work x persons should do in 26 days,working 9hrs a day 3/5
Amt of work 1 persons should do in 1 day,working 1 hr a day
=3*x/5*26*9 = 1/390x
ATQ-2;
1/390x = 1/62400
=> x = 160
So,number of additional persons required = 160 - 104 = 56 (ans)
...........#RubelChowdhury
138.In an organization 30% of all employees live over 10 miles away
from the place of work & 60% of workers who live over 10 miles use
company transport.If 40% of the employees of the company use
company transport,what % of the employees live 10 miles or less
from work & use company transport?
Ans with explanations:
Let total employee = 100
Employee who live over 10 miles distance= 30
Employee who live over 10 miles and use co. transport= 30*60%= 18
Total employee use co. transport =100*40%= 40
So,% of employee who live 10 miles or less distance and use co. transport =
40-18= 22%(Ans)
........Saiduzzaman Titu
139.The ratio of income of Yousuf & Rubel is 3:4 and their ratio of
expenditure is 4:5.Find the ratio of their savings if the savings of
Yousuf is 1/4th of his income?
Ans with explanations:
Let, Yousuf and Rubel's income be 3x & 4x respectively
and their expenditure be 4y & 5 y
So Yousuf's savings = 3x-4y
& Rubel's savings = 4x-5y
ATQ;
3x-4y = 3x/4
=>y= 19x/16
So, Rubel's savings = 4x - 5*9x/16=19x/16
Required ratio = 3x/4:19x/16=12:19(ans)
.........Saiduzzaman Titu

140.Mr. A invested an amount of tk 13900 divided into two different


schemes A & B at a simple interest rate of 14% per annum & 11%
per annum respectively. If the total amount of simple interest
earned in 2 years be tk 3508,what was the amount invested in
scheme B?
Ans with explanations:
Let,
The amount invested in scheme A = X tk
& the amount invested in scheme B= 13900-x tk.
ATQ;
3508 = x. 14%. 2 +( 13900-x) *11%*2
=>x= 7500
So,Investment in scheme B = 13900-7500= 6400tk(ans)
.........Prosenjit Chandra Roy
141.A,B & C can do a piece of work in 16,32 & 48 days respectively.
They started working together but C left after working for 4 days
and B, 2 days before the completion of work.How many days it took
to complete the work?
Ans with explanations:
Let, Total work will be complete x days.
A can do1/16 part in 1 day
B can do 1/32 part in 1 day.
C can do 1/48 part in 1 day.
A's x days work =x/16
C's 4 days work = 4/48 = 1/12
B's (x-2) days work = x-2/32
ATQ;
x/16 + x-2/32 +1/12 = 1
=>x = 94/9
So,the work will be finished in 94/9 or 10.44 days.(ans)
.............Prosenjit Chandra Roy
142.A train went 300 miles from X to city Y at an avg speed of
80mph.At what speed did it travel on the way back if it's avg speed
for the whole trip is 100 mph?
Ans with explanations:
let, the train back Y to X at p mph.
so,
300/80 + 300/p = 300*2/100
or, 1/80 + 1/p= 2/100
or, p = 133.33
So,the train back Y to X at 133.33 mph(ans)
...........Prosenjit Chandra Roy
143.Two workers A & B are engaged to do a work.A working alone
takes 8 hours more to complete the job than they both working
together.If B worked alone he would need 4.5 more hrs to complete
the job than they both working together.What time would they take
to do the work together?
Ans with explanations:
Let A works at the rate of 'a' units/hr and B works at the rate of 'b' units/hr.
A and B complete the job in x hrs.
A working alone takes 8 hours more to complete the job than if both worked
together.
=> a(x+8) = (a+b)x
=> ax + 8a = ax + bx
=> 8a = bx or a = bx/8 ---(1)
If B worked alone, he would need 4.5 hours more to complete the job than
they both working together
=> b(x+4.5) = (a+b)x
=> bx +4.5b = ax + bx
=> 4.5b = ax or a = 4.5b/x ---(2)
From (1) and (2), we have
bx/8=4.5b/x
⇒x^2=4.5∗8
⇒x=6
So, They take 6 hrs to do the work together. (Ans)
...........#GikoGiko
Alternate Solution:
Let, A & B together take x hours to complete the work.
Then,A alone takes (x+8)l hrs & B alone takes (x+4.5) hrs
ATQ;
1/(x+8) + 1/(x+4.5) = 1/x
=>x=6
So,A & B take 6 hours to complete the work together. (ans)
........#RubelChowdhury

144.3 numbers which are co-prime to each other are such that the
product of the first two is 551 & the last two is 1073.What is the
sum of the 3 numbers?
Ans with explanations:
Since the numbers are co- prime, they contain only 1 as the common factor.
Also, the given two products have the middle number in common.
So, Middle number = H.C.F of 551 and 1073=29
First number = 551/29 =19
& Third number =1073/29 = 37
So,Required sum= 19+29+37= 85(ans)
.......Prosenjit Chandra Roy
145.Sterling silver is 92.5% of pure silver.How many grams of
sterling silver must be mixed to a 90% silver alloy to obtain a 500g
of a 91% silver alloy?
Ans with explanations:
Let,
X is the number of grams of 92.5% Sterling silver
Then,(500-x) is the number of grams of 90% Silver alloy
ATQ;
92.5*x/100 + 90*(500-x)/100 = 91*500/100
=>0.925x + 450 - 0.90x = 455
=>0.025x = 5
=>x = 200
So, 200 gm of Sterling silver is needed to make 91% silver alloy(ans)
.........#RubelChowdhury
Alternate Solution:
Let,
x = grams of 92.5% silver alloy
Then, since the total weight is 500g, we have
(500-x) = grams of 90% silver
We now write an equation relating the amounts of pure silver in the
two "input" mixtures and in the "output" mixture:
"x" grams at 92.5% silver + (500-x) grams at 90% = 500g at 91%
(x)(0.925) + (500-x)(0.90) = (500)(0.91)
0.925x + 450 - 0.90x = 455
5 = 0.025x
5000 = 25x
5000/25 = x
200 = x
So, to make 500g of an alloy of 91% silver, It need to mix 200g of
92.5% alloy and (500-200)=300g of pure silver. (Ans)
.......#GikoGiko
146.You have a rifle with only 3 bullets by which you have to
shootdown a helicopter.You will shoot the bullets one after
another.The probability of the 1st bullet to hit it,is 90% & those of
2nd and 3rd one is 80% & 70% respectively. What is the probability
that the helicopter will shootdown?
Ans with explanations:
Let,
probability of shootdown for 1st bullet P(1) = 90/100
probability of shootdown for 2nd bullet P(2) = 80/100
probability of shootdown for 3rd bullet P(3) = 70/100
so, probability of not shootdown for 1st bullet P'(1) = 10/100
probability of not shootdown for 2nd bullet P'(2) = 20/100
probability of not shootdown for 3rd bullet P'(3) = 30/100
so, probability of not shootdown the helicopter P' = P'(1)xP'(2)xP'(3)
= 6/1000
so, probability of shootdown the helicopter P = 1- 6/1000
= 994/1000
= 0.994 (Ans)
.............#AsmaBinteSalahuddin
147.Of the 300 subjects who participated in an experiment using
vertual-reality therapy to reduce their fear of heights,40%
experienced sweaty palms,30% experienced bomiting & 75%
experienced dizziness. If all of the subjects experienced at least
one of these effects & 35% of the subjects experienced exactly two
of these effects,how many of the subjects experienced only one of
these effects?
Ans with explanations:
Let,
Swety palms= a,Bomiting= b & Dizziness= c
Suppose,
Subjects experiencing Exactly two of these effects are
(a+b) = p, (b+c) =q & (c+a)=r
Subjects experiencing all 3 effects =x
& Only one of these effects = (a+b+c)
ATQ-1;
a+b+c+p+q+r+x=300..............(1)
ATQ-2;
a+p+x+q=120(as 40% of 300)........(2)
b+p+x+r=90(as 30% of 300)...........(3)
c+q+x+r=225(as 75% of 300)..........(4)
ATQ-3;
p + q+ r = 105(as 35% of 300)..........(5)
Adding equation (2),(3) & (4),we get
a+b+c+2(p+q+r+x)+x=435................(6)
Substract equation (1) from above equation,we get
p+q+r+2x=135
=>105+2x=135 [putting the value from equation (5)]
=>x=15
Now,from equation (1), we get
a+b+c+105+15=300
=>a+b+c=180(ans)
..........#RubelChowdhury
148.A project has been started by a man on the 1st day.Each
subsequent day a new person joined the work force & it is known
that the total work got completed on 11th day.If from the starting
day 6 men working on the project & no new men added later,in how
many days the work got completed?
Ans with explanations:
Let,
One person can do 'x' unit of work in one day
So,In 2nd day= 2x, 3rd day = 3x.........., 11th day = 11x unit work is done.
ATQ,
x+2x+3x+......+11x=1
=> x(1+2+3+......+11)=1
=> x*66=1
=> x=1/66
Hence, 1 man' one day's work =1/66
So, 6 men 1 day's work = 6/66=1/11
Here 1/11 work done by 6 man in 1 day
So, 1 work done by 6 man in 11 days (ans)
.........Saiduzzaman Titu
149.A library has an avg of 510 visitors on Sunday & 240 on other
days.The avg numbers of the visitors per day in a month of 30 days
beginning with a Sunday is?
Ans with explanations:
As Sunday is the first day so total 5 Sunday in a 30 day month
remaining day =30-5= 25 days
Average no. of visitors = (510*5+25*240)/30= 285(ans)
........Saiduzzaman Titu
150.A hollow garden roller 63cm wide with a girth of 440cm is made
of iron 4cm thick.What is the volume of the iron used?
Ans with explanations:
Let,
External radius = R
Internal radius = r
Here, Girth of roller = circumference of roller =440cm
Thickness of iron = 4cm & height =63 cm
ATQ,
2πR=440
=> R =70
So, Internal radius r= 70-4=66
Volume of iron = πR^2h-πr^2h
=πh(R^2-r^2)
=22/7*63*(70^2-66^2)
=107712
..........Saiduzzaman Titu
151.In a factory where toys are manufactured,machines A,B & C
produce 25%,35% & 40% of the total toys respectively. Of their
output 5%,4% & 2% respectively are defective toys.If a toy drawn at
random is found to be defective,what is the probability that it is
manufactured on Machine B?
Ans with Explanations:
Let,
The total toys produced be 100
A's production is 25
B's production is 35
C's production is 40
Defective:
A=25×5%=25/20
B=35×4%=35/25
C=40×2%=4/5
Total defective s=(25/20+35/25+4/5)=345/100
So, probability=(35/25)÷(345/100)=28/69=0.405(ans)
.........#NilChad
152.A cricket team won 3 matches more than they lost.If a win gives
them 2 points & loss -1 point,how many matches,in all have they
played if their score is 23?
Ans with explanations:
Let,
Lost = x matches
So,Win =x+3 matches
ATQ,
2(x+3) + x(-1)= 23
=>x=17
Total played =x+x+3 = 37(ans)
...........#NilufarYasmin
153.A shopkeeper combined X litres of a solution containing 10%
certain material Q,with 10 litre of another solution that contained
2% that material Q,to produce Z litres of a solution that was 5%
Q.What is the value of X?
Ans with explanations:
Given that,
Material Q in X litres = X*10/100 =X/10 litres
Material Q in 10 litres = 10*2/100 = 1/5 litres
Material Q in Z litres = (X+10)*5/100 = (X+10)/20 litres
ATQ;
X/10 + 1/5 = (X+10)/20
=> x = 6
So,the value of X is 6 litres(ans)
......#RubelChowdhury
154.P starts jogging from point X to Y.A half hour later R who jogs
1mph slower than twice P's rate starts from the same path.If R
overtakes P in 2hrs.How many miles will R have covered?
Ans with explanations:
Let,
P's speed = x mph & R's speed = (2x - 1) mph
ATQ;
2(1/2)*x = (2x - 1)*2
=>2.5x = 4x -2
=>x = 3.5
=>x = 1.33
So,Distance covered by R =( 2*1.33-1)=3.33miles(ans)
...........#RubelChowdhury
155.A cask initially contains pure alcohol upto the brim.The cask
can be emptied by removing exactly 5 litres at a time.Each time this
is done,the cask must be filled back to the brim with water.The
capacity of the cask is 15 litres.When the cask is completely
emptied & filled back to the brim two times,what is the ratio of
Alcohol to Water in the cask?
Ans with explanations:
Given that,
The capacity of the cask is 15 litres
ATQ;
1st time, 5 litres of alcohol is removed and 5 litres of water is added.
So, the ratio of alcohol and water in the cask is 10:5=2:1
In case of 2nd time,
5 litres is removed which contain both alcohol and water.
So, remaining alcohol = 10-(5×2/3)=20/3 litres
remaining water=5-(5×1/3)=10/3 litres
Again 5 litres of water is added.So, the amount of water is increased.
Now,Total water=(10/3+5)=25/3 litres
Thus, the ratio of Alcohol to water =(20/3):(25/3)=4:5(ans)
...........#NilChad
156.A store currently charges the same price for each towel that it
sells.If the current price of each towel were to be increased by tk
1,10 fewer of the towels could be bought for tk 120,excluding sales
tax.What is the current price of each towel?
Ans with explanations:
Let,
Total no. of towel be x &
Price of each towel be y tk
ATQ-1;
xy=120…(1), ;
=>x = 120/y
ATQ-2;
(x-10)(y+1) = 120…(2)
From 2,
xy+x-10y-10=120
=>120*y/y + 120/y -10y = 130 (Plugging from 1)
=>120/y -10y = 10
=>y^2+y-12 = 0
=>(y+4)(y-3)=0
Hence, y=3 (As price cannot be negative)
So,the current price is 3 tk(ans)
...........#RubelChowdhury
157.Car A is 20 miles behind Car B,which is travelling in the same
direction along the same route as Car A.Car A is travelling at a
constant speed of 58mph and Car B is traveling at a constant speed
of 50 miles per hour.How many hours,it will take for Car A to
overtake & drive 8 miles ahead of Car B?
Ans with explanations:
Given that,
A has (58-50) = 8 miles per hour more speed than car B
ATQ,
Car A needs to travel (20+8) = 28 more miles than car B
So, the required time = distance/ speed = 28/8 h = 3.5 h = 3h 30min (Ans.)
..........#AsmaBinteSalahuddin
158.1/3 rd of Vindo's mark in mathematics exceeds half of his
marks in social studies by 30.If he got 240 marks in 2 subjects
together,how many marks did he get in social studies?
Ans with explanations:
Let, Marks in Mathematics and Social Studies be M and S
ATQ,
M+S=240.......(i)
=>M=240-S
Again,
M*1/3-30 = S*1/2
=> 2M -3S = 180
=> 2(240-S) -3S=180
=> 480 - 2S -3S = 180
=> S = 60
So,He get 60 marks in Social Studies (Ans.)
.........#AsmaBinteSalahuddin
159.Coins are to be put into 7 pockets so that each pocket contains
at least 1 coin, at most 3 of the pockets are to contain the same
number of coins & no two of the remaining pockets are to contain
an equal number of coins.What is the least possible number of
coins needed for the pockets?
Ans with explanations:
Since at most 3 of the pockets are to contain the same number of coins then
minimize of coins in each, so let each contain just 1 coin;
Next, we are told that no two of the remaining 4 pockets should contain an
equal number of coins, so they should contain 2, 3, 4, and 5 coins each (also
minimum possible);
Total: 1+1+1+2+3+4+5=17(ans)
......#RubelChowdhury
160.1/3 rd,1/4 th,1/5 th & 1/7th of the human population of Island
X,which has fewer than 5000 human inhabitants,are all whole
numbers & their sum is exactly the population of Island Y.What is
the population of Island Y?
Ans with explanations:
Since,
1/3rd, 1/4th, 1/5th & 1/7th of the population are all whole numbers, Hence,
The total population must be a multiple of the LCM of 3, 4, 5 & 7.
LCM of 3, 4, 5, 7 = 3x4x5x7 = 420
Now, 5000/420 11(20/21).
So, the largest number less than 5000, that is also a multiple of
420 is 420x11 = 4620
:. Population of Y
= 4620 x (1/3 + 1/4 + 1/5 + 1/7)
= 4620{(105+140+84+62)/420}
= 11x389 = 4279
So, the population of Y Island is 4279 (Ans)
.........#GikoGiko
161.The cost of diamond varies directly as the square of it's
weight.Once this diamond broke into 4 pieces with weights in the
ratio 1:2:3:4.When the pieces were sold,the merchant got tk 70000
less.Find the original price of the diamond?
Ans with explanations:
We are given that cost=(weight)^2-directly as the square of the weight and
total piece was broken into four pieces
first let us find the total weight of initial piece
1k+2k+3k+4k=10k where k is +ve integer
So, cost of intial piece=(10k)^2
Then, let us find the cost of other pieces
p1. with weight k
cost=(k)^2
p2, with weight 2k
cost=(2k)^2=4k^2
p3. with weight 3k
cost=(3k)^2=9k^2
p4, with weight 4k
cost=(4k)^2=16k^2
total cost of pieces=k^2+4k^2+9k^2+16k^2=30k^2
and we are given that the value of broken pieces is 70000 less than initial total
one
30k^2=100k^2-70000
70k^2=70000
k^2=1000
to find value of initial diamond simply insert k^2=1000 in expression fo its
value
cost=100k^2=100*1000=100000 tk
..............#GikoGiko
Alternate Solution:
Let,
Price of diamond as kx^2 where k is a contant
Total price for 4 pieces =kx^2[1+4+9+16]=30kx^2
Price of original diamond=100kx^2
Difference 70kx^2=$70000 or kx^2=1000
Original price of the diamond=100*1000=100000 tk(ans)
...........#RubelChowdhury
162.2 cogged wheels of which 1 has 32 cogs & other 54 cogs,work
into each other.If the latter turns 80 times in three-quaters of a
minute,how often does the other turn in 8 seconds?
Ans with explanations:
Cogs turned per second =80×54/0.75×60=96
Turns of the first wheel in 8 seconds =96×8/32=24(ans)

.........#GikoGiko

Part – 03(06/02/2018-23/02/2018)

163.A box of 54 oranges is sold for tk 162 & a box of 27 is sold for
tk 132.A customer needs only 54 oranges but he had to buy 2
boxes as one with 54 oranges was not available.Approximately,
how much was the excess money that the customer has to pay for
each orange?
Ans with explanations:
To buy 54 oranges have to paid excess=(132*2)-162=102 tk
So,to buy 1 orange has to pay=102/54=1.89 tk(ans)
164.How many ounces of water must be added to a 30 ounce
solution that is 40% alcohol to dilute the solution to 25% alcohol?
Ans with explanations:
In 30 ounce solution Alcohol = 30*40%=12 ounce
& Water = 30 -12 =18 ounce
Let, x ounce water must be added to this solution
ATQ,
12/(18+x)=25/75 [as in new solution alcohol is 25%]
=> x=18
So,18 ounces water must be added to dilute the solution(ans)
...........Saiduzzaman Titu
165.A shopper spends $25 to purchase floppy disks at 50¢
each.The next day, the disks go on sale for 30¢ each & the shopper
spends $45 to purchase more disks?What was the avg price per
disk purchased?
Ans with explanations:
On the 1st day number of disks purchase=$25/50¢=2500/50=50¢
On the next day number of disks purchase
=$45/30¢=4500/30=150¢
So,the avg price per disks
=(2500+4500)/(50+150)=35¢(ans)
165..After driving to a riverfront parking lot,Ansar plans to run
south along the river,turn around & return to the parking
lot,running north along the same path.After running 3.25 miles
south,he decides to run for only 50 minutes more.If Ansar runs at a
constant rate of 8 minutes per mile,how many miles further south
can run & still be able to return the parking lot in 50 minutes?
Ans with explanations:
Given that,
Ansar runs at a constant rate of 8 minutes per mile
Thus,in 50 mins he can cover 50/8=6.25 miles
Therefore the total round trip distance Ansar covers=(3.25+6.25)=9.50 miles
Half of that distance he runs south,
So,he runs 9.50/2=4.75 miles south
Since,he already runs 3.25 miles south,so he can run (4.75-3.25)=1.50 miles
farther to south(ans)
166.The total cost for company X to produce a batch of tools is tk
10000 plus tk 3 per tool.Each tool sells for tk 8.The gross profit
earned from producing & selling these tools is the total income
from sales minus the total production cost.If a batch of 20000 tools
is produced & sold,then what is company X's gross profit per tool?
Ans with explanations:
The total production cost of 20000 tools=10000+3*20000=70000 tk
The total income from sales of 20000 tools=8*20000=160000tk
So,the gross profit earn=160000-70000=90000 tk
Hence,the gross profit per tool=90000/20000=4.50 tk(ans)
167.A club collected exactly tk 599 from it's member.If each
member contributed at least tk 12,what is the greatest number of
members the club could have?
Ans with explanations:
Let's,
First divide 599 by 12(the minimum amount that each member can contribute)
& then we have to use the remainder to solve this.
599/12=49.91
That mean,49*12+1*11=599 tk
So,the greatest number of the club members is 49 who could contribute at
least 12 tk each to make it 599 in total with another member who could
contribute 11 tk(ans)
168.A tank contains 10000 gallons of solution that is 5% NaCl by
volume.If 2500 gallons of water evaporate from the tank,the
remaining solution will be approximately what % of NaCl?
Ans with explanations:
Here NaCI is 10000*5%=500 gallons
Water = 10000-500=9500
After evaporating 2500 gallons of water remaining water is = 9500-2500=7000
Required percentage =
500*100/(7000+500)= 6.67%(ans)
........Saiduzzaman Titu
169.The ratio by volume,of soap to alcohol to water in a certain
solution is 2:50:100.The solution will be altered so that the ratio of
soap to alcohol is doubled while the ratio of soap to water is
halved.If the altered solution will contain 100 cubic centimeters of
alcohol,how many cubic centimeters of water will it contain?
Ans with explanations:
Given that,
Initial ratio of soap to alcohol to water=2:50:100
Here,soap:alcohol=2:50
Therefore,the ratio while double,it would be=2*2/50=4:50
Again,from the initial ratio we get,soap:water=2:100
Therefore,halved the ratio of soap:water=1*2/2*100=1:100
=4:400
Thus,new ratio of soap:alcohol:water=4:50:400
After mixing 100 cm^3 alcohol,the ratio will be changed into=8:100:800(as
4:50:400)
So,it will contain 800 cm^3 of water (ans)
Alternate Solution:
In Initial solution soap to alcohol=2/50=2:50
In Final solution soap to alcohol=4/50=2/25=2:25
In Initial solution soap to water=2/100
& In Final solution soap to alcohol=2/200=2:200
So,In the final solution the ratio of soap to alcohol to water=2:25:200
Suppose,
In final solution soap is 2x cm^3,alcohol is 25x cm^3 & water is 200x cm^3
ATQ,
25x=100
=>x=4
So,the amount of water in final solution is 200*4=800 cm^3(ans)
170.Two friend A & B invest tk 40000 & tk 60000 respectively in
order to setup a business as the managing partner,A receives a
remuneration of tk 12000 out of total prrofits & B receives no
salary.After A's salary has been paid of 10% of the profits are set
aside each year to purchase new equipments.The remainder of the
profit is then divided between the friends in the ratio of their capital
investment.The total profit for the 1st year was tk 210000.Calculate
the total amount received by A & B individually at the end of the 1st
year?
Ans with explanations:
Ratio of the capital of A and B = 40000:60000=2:3
Given that,
Total profit = 210000 tk
Salary of A. = 12000 tk
So,Remaining profit = 210000-12000= 198000 tk
After purchasing equipment,
Now, the dividable profit =198000 - 198000*10/100 = 178200 tk
A receive from profit=178200*2/(2+3)= 71280 tk
B receive from profit= 178200*3/(2+3)= 106920 tk
So A's total amount = 71280+12000=83280 tk
B's total amount = 106920 tk(ans)
.....Saiduzzaman Titu
171.A pharmaceutical company received tk 3lacs in royalties on the
1st tk 20lacs in sales of the generic equivalent of 1 of it's products
& then tk 9lacs in royalties on the next tk 108lacs in sales.By
approximately what % did the ratio of royalties to sales decrease
from the 1st tk 20lacs in sales to the next tk 108lacs?
Ans with explanations:
In 20 lacs sales royalties 3 lacs
in 108 lacs sale royalties 16.2 lacs
royalties decrease 16.2-9=7.2 lacs
so,decrease %=7.2*100/16.2=44.44%(ans)
..........Rasel Arahan
172.A ladder of a fire track is elevated to an angle of 60 degree &
extended to a length of 70ft.If the base is of the ladder is 7ft above
the ground,how many feet above the ground does the ladder
reach?
Ans with explanations:
Given that,
Elevation angle(উন্নতি ক োণ)=60 degree
Hypotenuse(as extended length)=70 ft
& the base is of the ladder is 7ft above the ground.
So,total length of the ladder,h=x+7, which is the value of height.
We have,
Sin 60 degree=height/hypitenuse
=>(sqrt 3)/2=x/70
=>x=35 sqrt 3
So,Ladder could reach =(7+35sqrt 3)=67.622 ft above the ground. (ans)
173.A tea merchant blends 2 kinds of tea,putting in 3 parts of a tk
25 per kg grade to 1 part of a tk 15 per kg.If the mixture is changed
to 1 part of the tk 25 per kg grade to 3 parts of less expensive
grade,how much will the shop lose or save in blending 200 kg?
Ans with explanations:
Given that,
Total quantity of tea is 200 kg
ATQ-1;
Quantity of 25 tk kg tea=3*200/4=150 kg &
Quantity of 15 tk kg tea=1*200/4=50 kg
So,total price of 200kg tea
= (150*25+50*15)= 4500 tk
ATQ-2;
Total price of 200 kg tea
=(50*25+150*15)=3500 tk
Thus,save from 2nd case=4500-3500=1000 tk(ans)
174.A boy multiplied 423 by a number & obtained 65589 as his
answer.If both the fives in the answer are wrong & all other figures
are correct,then what is the correct answer?
Ans with explanations:
Observe Last digit is 9, so Last digit of the number to be multiplied must be 9,
observing that first digit is 6, the first 2 digit of the number to multiplied can be
15,16 ,17{ SINCE 15*4= 60, 16*4= 64,17*4 = 68}, so numbers can be
153,163,173 , but all these values do not give 89 as last digit , so now check
for 143 { since 18*4 = 72 , and first digit will be greater than 7 } , so checking
for 143 , u get 423*143 = 60489
Courtesy to Joy Shaha
Alternate Solution:
65589/423=155
unit digit must be 3. 3*3=9
so, number can be less than or more than..155
53*23=19
43*23=89
we have found last two digit 89.
143*423=60489(ans)
.........#WaresKhan
175.A cube of side 5 cm is painted on all it's side.If it is sliced into 1
cubic centimeter cubes,how many 1 cubic centimeter cubes will
have exactly one of their sides painted?
Ans with explanations:
It will have 5*5=25 squares per unit
The border will always be 2 side painted.
There calculation per side=(5-2)*(5-2)=9
(As two rows & columns are to be subtracted)
So,total 6 sides=6*9=54(ans)
176.If 80 lamps can be lighted 5 hrs per day for 10 days for tk
21.25,then the number of lamps which can be lighted 4 hrs daily for
30 days for tk 76.50 is?
Ans with explanations:
Given that,Per day 5 hrs
Then,in 10 days 10*5=50 hrs
Again,4 hrs daily,so in 30 days=120 hrs
In 50hrs need 21.25tk to get light from 80 lamps
In 120 hrs need 76.50tk to get light from=(80*50*76.50)/120*21.25=120
lamps(ans)
177.Machine A produces bolts at a uniform rate of 120 in every 40
seconds & Machine B produces bolts at a uniform rate of 100 in
every 20 seconds.If the two machines run simultaneously, how
many seconds will it take for them to produce a total of 200 bolts?
Ans with explanations:
Machine A produces in every second= 120/40=3 bolts
Machine B produces in every second 100/20=5 bolts
Together,A & B can produce in every second 3+5=8 bolts
So,They together can produce 200 bolts in = 200/8=25 seconds(ans)
178.A rectangular garden is to be twice as long as it is wide.If 360
yards of fencing including the gate,will completely enclose the
garden,what will be the length of the garden in yards?
Ans with explanations:
Let,
Width of the garden is x yards
and length is 2x yard
ATQ;
2(2x+x)=360
x=60
length=2*60=120 yards(ans)
........Rasel Arahan
179.If a right angled isosceles triangle is inscribed in a circle,what
is the ratio of the area of the circle to the area of the triangle?
Ans with explanations:
Let,
Radius of the circle x
so,side of the triangle x
Ratio of the area of the circle and triangle is =22/7*x^2:1/2*x^2=44:7
(ans)
.......Rasel Arahan
180.In a group of students,80% of the students weigh less than
60kg & a total of 32 students weigh less than 50kg.If 60% of the
students weigh at least 50kg,how many of the students weigh at
least 50kg but less than 60kg?
Ans with explanations:
Let,
Total students be x
ATQ;
Students weigh at least 50 kg + students weigh less than 50 kg=total students
=>60%x+32=x
=>x=80
So,students above 60 kg=(100-80)%*80=20%*80=16
& Given that students less than 50kg=32
So,remaining=80-16-32=32 are the number of students who weigh at least 50
kg but less than 60kg(ans)
181.Jack has two quarts of a 30% acid solution & three pints of a
20% solution.If he mixes them, what will be the concentration to the
nearest of the resulting solution?
Ans with explanations:
We have,1 quarts= 2 pints
So,2 quarts=2*2=4 pints
Given,
2 quarts of 30% solution contain=4*30/100=1.2 pints of acid.
Thus,3 pints of 20% solution contain=3*20/100=0.6 pints of acid
The concentration of the resultant solution is
=(1.2+0.6)/4+3=26%(approximately) (ans)
182.A certain cake recipe states that the cake shoul be baked in a
pan 8 inches in diameter. If Salma wants to use the recipe to make
a cake of the same depth but 12 inches in diameter,by what factor
should she multiply the recipe ingredients?
Ans with explanations:
Round pan means it is cylindrical and by increasing diameter we are
increasing the volume of the cylinder.
As we have,volume of cylinder =πr^2h
Here, radius are =12/2=6 inches & 8/2=4 inches
So,the factor by which recipe must be increased
=π*6^2*h/π*4^2*h (as depth is same in both the case)
= 36/16
= 9/4(ans)
183.The savings of an employee equals income minus
expenditure.If their income ratio is 1:2:3 & their expenses ratio is
3:2:1,then what is the order of the employees A,B & C in the
increasing order of the size of their savings?
Ans with explanations:(As MCq Type question)
As the the ratio of their incomes are in ascending order, and their expenses
are in descending order, their savings also in their incomes order.
So savings order = A < B < C(Ans)
184.How many 4 letter words with or without meaning, can be
formed out of the letters of the word 'LOGARITHMS' , if repetition of
the letters not allowed?
Ans with explanations:
LOGARITHMS contains 10 different letters.
So,the required number of words = 10p4 = 5040(ans)
185.From a group of 5 managers(Joon,Kendra,Lee,Marlie,Noomi),2
people are randomly selected to attend a conference, what is the
probability that Marlie & Noomi are both selected?
Ans with explanations:
The total ways of choosing two people from a group of 5 managers is 5c2 =
10
If we need to find the probability that Marlie & Noomi are choosen,that is just 1
of the 10 combinations possible.
Hence,the required probability = 1/10(ans)
186.If x is an even number,what is the difference between the
smallest even number greater than (5x+6) & the largest even
number less than (3x+9)?
Ans with explanations:
As,x is given an even number,
Then 5x+6 = odd*even+even=even+even=even
So,the smallest even number greater than (5x+6) is =5x+6+2=5x+8
Again,3x+9=odd*even+odd=even+odd=odd
So,the largest even number less than (3x+9) is =3x+9-1=3x+8
Now,the difference is=(5x+8)-(3x+8)=2x(ans)
187.A new word is to be formed by randomly rearranging the letters
of the word 'ALGEBRA'.What is the probability is the new word has
consonants occupying only the positions currently occupied by
consonants in the word 'ALGEBRA'?
Ans with explanations:
The number of words that can be formed from the word 'ALGEBRA'
=7!/2! (as A repeats for 2 times)= 2520
If the positions of the consonants in the new word occupy the same positions
of the consonants as the word 'ALGEBRA'(like. V,C,C,V,C,C,V where V for
vowels & C for consonants)
Hence,the consonants can be filled in 4! = 24 ways
& the vowels can be filled in 3!/2! = 3 ways (as A repeats for 2 times)
So,the arrangement for new word will be =24*3=72 ways
The required probability is = 72/2520 = 1/35 (ans)
188.You are planning to deposit tk 12500(the principal) in a bank
account at the beginning of the year.The interest rate per annum is
8.5%.You are taxed 25% each year on your interest income,not on
the principal amount.The remainder of your interest income earns
further interest over the next year & well as the principal.What will
be your total income at the end of 3 yrs?
Ans with explanations:
At the end of 1st year, interest will be=12500*.085=1062.5
after deducting 25% tax interest will added to principal=1062.5*.75=796.88
at the end of 2nd year, interest will be=(12500+796.88)*.085 =1130.23
after deducting tax interest will add to principal =1130.23*.75=847.67
at the end of 3rd year interest will be=(12500+796.88+847.67)*.085=1202.29
after deducting tax interest income=1202.29*.75=901.72
Total income =796.88+847.67+901.72=2546.27 (approximately) (ans)
......Joy Shaha
189.There are 58 balls in a jar.Each ball is painted with at least one
of two colors,red or green.It is observed that 2/7 of the ball that
have red color also have green color,while 3/7 of the balls that have
green color also have red color.What is the probability that a ball
randomly picked from the jar will have both red & green colors?
Ans with explanations:
Let,T be the total number of balls, R be the number of Red balls, G be the
number of Green balls & B be the number of balls with both Red & Green.
So, the number of balls having oonly Red is R-B and
The number of balls having only Green is G-B
So,the total number of balls,T =(R-B)+(G-B)+B=R+G-B............1
Given that,
2/7 of the balls that are red have also green,
So B = R*2/7=>R=7B/2
Again, 3/7 of the balls that are green have also red,
So B=G*3/7=>G=7B/3
From equation 1,we get
T=7B/2 + 7B/3 - B = 29B/6
=>B = 6T/29
Hence, 6T/29 of the balls contain both colors.
So, the probability of selecting such a ball is the fraction
= (6T/29)/T = 6/29 (Ans)
........Thnx to #GikoGiko
190.A manager has a budget of tk 60000 for giving salary increment
of 4 full-time & 2 part time employees.Each of the full time
employees receives the same increment,which is twice the
increment that each of the part time employees receives.What is
the amount of increment that each full-time employees receives?
Ans with explanations:
Let,
Each Part time employee receives x tk
& Each Full time employee receives 2x tk
ATQ;
4*2x+2*x=60000
=>x=6000
Then, each full time employee receives 2*6000=12000 tk(ans)
191.A ferry can travel twice as fast when empty as when it is full.It
travels 20 miles with full load, spends 1 hour for unloading &
returns to it's original port empty.It takes 11 hours to complete the
journey.What is the speed when the ferry was empty?
Ans with explanations:
Given that,
Spends 1 hr for unloading
So,total time spent without the unloading time = 11-1=10 hrs
ATQ,
Speed without cargo = 2*speed with cargo
So,Time taken without cargo=(1/2)*time taken with cargo(as speed is
inversely proportional to time).
Let,time taken with cargo= t hrs & time taken without cargo =t/2 hrs
So,t+t/2=10
=>t=20/3
Thus,time taken without cargo =20/6=10/3 hrs
Hence,
speed without cargo=20/(10/3)=6mph= 6mph(ans)
Alternate Solution:
Given that,
Spends 1 hr for unloading
So,Total time spent without the unloading time=(11-1)=10 hrs
Let,
The ferry takes 2x hrs to go 20 miles with loading & the ferry takes x hrs to
return 20 miles without loading
ATQ;
x+2x=10
=>x=10/3
So,speed without loading = 20/(10/3)=6 mph(ans)
.........Giko Giko
192.Arif & Mukul start running at the same time & from the same
point around the circle of 216 meters circumference. If Arif can
complete one round in 40 seconds & Mukul in 50 seconds,how
many seconds will it be before they both reach the starting point
simultaneously & how many rounds they will complete by this
time?
Ans with explanations:
Arif reaches starting point in every 40, 80,120....... sec,
Mukul 50; 100; 150..... sec
So, LCM of 40, 50 =200 sec
speed ratio = (1/40:1/50)= 5:4 ( total rounds) (ans)
........Joy Shaha
193.How many circles, each with a 4 inch radius, can be cut from a
rectangular sheet of paper,measuring 16 inches*24 inches?
Ans with explanations:
Since the circle of the radius is given 4 inch then the diameter is 8 inch.
Given that,
the length & the breadth of the rectangular are 24 inches & 16 inches
respectively.
So,Along the length it can be cut in 24/8=3 rows
& along the breadth it can be cut in 16/8=2 rows
So,total 3*2=6 circles can be cut from a rectangular sheet of paper(ans)
............Thnx to Joy Shaha
194.A square is inscribed in a circle & another circle is inscribed in
a square.If the area of the smaller circle is π, what is the area of the
larger circle?
Ans with explanations:
Let,radius of the smaller circle be r & radius of the larger circle be R
ATQ-1;
π*r^2 = π
=>r = 1
As we know,when a circle is inscribed in a square, the diameter of the circle is
equal to the side length of the square.
ATQ-2;
One side of square=1+1=2 unit
So,it's diagonal = 2root2
Hence,the diameter of the larger circle is 2root2
Now,the area of the larger circle is=π*(2root2/2)^2=2π (ans)
.............Thnx Rasel Arahan & Saiduzzaman Titu
195.A popular website requires users to create a password
consisting of digits only.If no digit may be replaced & each
password must be at least 9 digits long,how many passwords are
possible?
Ans with explanations:
Total number of digit
= 10
9 digits password taken from 10 in
= 10P9
= 10!/(10-9)!
= 10!
10 digits taken from 10 numbers in
= 10P10
= 10!/(10-10)!
= 10!/0!
= 10!/1
= 10!
So, sum= 10!+10!=2*10! (Ans)
........Samrat Roy Sondhan
196.When n is divided by 6, the remainder is 3 & the quotient is q.
3n+2 is divided by 6, the remainder is 5, then the quotient in terms
of q is?
Ans with explanations:
Here, divisor= 6
Quotient=q
remainder=3
ATQ,
n=6q+3......... (i)
Let, new quotient is x
So, 6x+5=3n+2
=> 6x+5=3(6q+3)+2[from equation(i)]
=>x=3q+1(ans)
..........Saiduzzaman Titu
197.An old man distributed all the gold coins he had to his two
sons into two different numbers such that the difference between
the squares of the two numbers is 36 times the difference between
the two numbers.How many coins did the old man have?
Ans with explanations:
Let,the no of coins two sons got be x & y
ATQ;
x^2-y^2 = 36(x-y)
=>x+y = 36
So,the old man has 36 coins (ans)
198.A survey of n people in the town of Eros found that 50% of
them prefer Brand A.Another survey of 100 people in the town of
Angle found that 60% prefer Brand A.In total 55% of all the people
surveyed together prefer Brand A.What is the total number of
people surveyed?
Ans with explanations:
ATQ,
n*50/100+100*60/100=55/100(n+100)
=>(50n+6000)/100=(55n+5500)/100
=>n=100
so total number of people surveyed= 100+100=200(Ans)
.......Saiduzzaman Titu
199.In a garden there are 10 rows & 12 columns of trees.The
distance between any two trees is 2m & a distance of 1m is left
from all sides of the boundary of the garden.What is the length of
the garden?
Ans with explanations:
Each row contains 12 plants.
There are 11 gapes between the two corner trees.
So, the required length {(11*2)+(2*1)} metres. (1 metre contains each left
side= 2 sides)
Therefore Length = (22 + 2) m = 24 m.(Ans)
......Giko Giko
200.A room is half as long again as it is broad.The cost of carpeting
the room at tk 5 per sq.m is tk 270 & the cost of papering the 4
walls at tk 10 per sq.m is tk 1720.If a door & 2 windows occupied 8
sq.m.Then find the dimensions of the room?
Ans with explanations:
Let breadth = x metres, length = 3x metres, height = H metres.
Area of the floor=(Total cost of carpeting)/(Rate) = (270/5) sq.m = 54 sq.m
Now,
x*3x/2= 54
=> x^2 = 54×2/3
=> x = 6
So, breadth = 6 m and length =3/2*6 = 9 m.
Now, papered area = Total expense/Cost = (1720/10) = 172 sq.m
Given, Area of 1 door and 2 windows = 8 sq.m
Now, Total area of 4 walls = (172 + 8) sq.m = 180 sq.m
Or, 2(Length+Breath)Height = 180 m
Or, 2(9+6)H=180
Or, H = 6
So, The dimension of the room are Length = 9 meter, Breadth = 6 meter &
Height = 6 meter.(Ans)
..........Giko Giko
201.The population of a country doubled every 10 years from 1980
to 2010.What was the percent increase in population during the
period?
Ans with explanations:
Let, the population in 1980= x
ATQ,
1990...... 2x
2000....... 2*2x
2010.….... 2*2*2x
The population in 2010
= 2*2*2x
= 8x
Increased population from 1980 to 2010
= 8x-x
=7x
% increased in population=7x/x*100=700%(Ans)
.............Samrat Roy Sondhan
202.How many different 3 digits number are greater than 299 & do
not contain the digits 1, 6 or 8?
Ans with explanations:
3 digits number greater than 299 = 999-299 = 700
Three places to be filled to make desired 3 digit number = - - -
Total ways to fill the left most place = 5 choices {3, 4, 5, 7, 9},It means
hundredth digit has 5 options
Total ways to fill the Middle place = 7 choices {All digits except 1, 6 and 8},It
means tenth digit has 7 options.
Total ways to fill the Middle place = 7 choices {All digits except 1, 6 and 8},It
means unit digit has 7 options.
So,Total Desired numbers = 5*7*7 = 245 (ans)
203.In a room filled with 7 people,4 people have exactly 1 sibling in
the room & 3 people have exactly 2 siblings in the room.If two
individuals are selected from the room at random,what is the
probability that those two individuals are not siblings?
Ans with explanations:
The sequence will be {1,1,1,1,2,2,2} for the vertices let's say {A,B,C,D,E,F,G}
Since, first 4 people have exactly 1 sibling So,
A is the sibling of B and vice versa,
C is the sibling of D and vice versa,
Three people have exactly 2 siblings, So,
E, F, and G are siblings to each other.
The probability of two individual persons are sibling
= (2C2+2C2+3C2)/7C2
= (1+1+3*2/2*1)/(7*6/2*1)
=(1+1+3)/21 = 5/21
Required probability = 1-(5/21)=16/21 (Ans)
.........#SamratRoySondhan
204.In a bag there are 6 identical green marbles, 4 identical blue
marbles & 2 identical red marbles.If the marbles are picked at
random from the 12 in the bag,how many distinct sets of 3 can be
selected?
Ans with explanations:
There are 7 identical group.
total set = all uncommon or 3 uncommon and 2 common or 2 uncommon and
3 common or 1 uncommon and 2 common from 2 group or 3 common and 2
common
= 7C5 + 6C3 x 2 + 6C2 x 1 + 5C1 x 1 x 1 + 1 [***]
= 21 + 20x2 + 15x1 + 5x1x1 + 1
= 82 (Ans.)
NB : *** 6C3 x 2 [2 common possible from green group & red group]
6C2 x 1 [3 common possible from green group only]
5C1 x 1 x 1 [2 & 2 common possible from green group & red group]
................#AsmaBinteSalahuddin
205.A man cheats while buying as well as while selling.While
buying he takes 10% more than what he pays for & while selling he
gives 20% less than what he claims to.Find the profit percent,if he
sells at 9.09% below the cost price of the claimed weight?
Ans with explanations:
Suppose,
He buys 100kg for 100 tk.
Price per kg=100/100=Tk.1
Step- Buying:
Pays for =100 kg but gets =110 kg[10% more]
Step- Selling:
Charges for = 100 kg but gives=80 kg[20% less]
Step-Profit:
At 9.09% less, cost per kg =1*90.91= Tk. 0.9091
Cost price of 100 kg;
At 0.9091 per kg=100*0.9091=Tk.90.91
At 1 per kg=100*1=Tk.100
When Cost price is tk. 90.91, he sells 80 kg
When Cost price is tk. 1, he sells 80/90.91 kg
When Cost price is tk. 100, he sells 80*100/90.91=87.99=88 kg (Approx)
He buys 110 kg for Tk. 100 and sold 88 kg for Tk.100
Gain=100/88-100/110=20/88;
Required %=(20*11/88*10)100=25%(ans)
............. Tom KS
206.The volume of a wall 5 times as high as it is broad & 8 times as
long as it is high, is 12.8 m^3, the breadth of the wall is?
Ans with explanations:
Let,
Breadth= x m
Hight= 5x m
Lehgth= 8*5x = 40x m
ATQ,
x*5x*40x = 12.8
=>x^3= 128/10*5*40
=>x^3= 8*16/5*2*5*5*8
=>x^3= 2*2*2/5*5*5
=>x= 2/5
So,the breadth of the wall is 2*100/5=40 cm(Ans.)
.............Samrat Roy Sondhan
207.In what ratio must a person mix 3 kinds of tea costing tk 60 per
kg, tk 75 per kg & tk 100 per kg so that the resultant mixture when
sold at tk 96 per kg yields a profit of 20%?
Ans with explanations:
CP of the mixture =96/1.2=80
Suppose,
Ratio of mixture of three kinds of tea are x,y and z respectively
Now 60x+100z=80(x+z)
Or x:z=1:1
Again 75y+100z=80(x+y)
Or 5y=20z
Or y:z=4:1
So required ratio 1:4:2(1+1)(Ans)
..........#KSTom
208.A school has raised 75% of the amount it needs for a new
building by receiving a avg donation of tk 60 from the people
already solicited.The people aleady solicited represent 60% of the
people,the college will ask for donations.If the college is to raise
exactly the amount needed for the new building,how much must
the remaining people donate per person?
Ans with explanations:
Suppose,
The number of people be x who has been asked for the donations.
People already solicited = 60% of x = 0.6x
Remaining people = 40% of x = 0.4x
Amount collected from the people solicited,
= 60 *0.6x = 36x tk
36x = 75% of the amount collected.
Remaining amount = 25% = 12x tk(Ans)
..........#TomKS
209.A metallic sheet is of rectangular shape with dimensions.
40m*60m.From each of it's corners, a square is cut off so as to
make an open box.If the length of the square is 8, what is the
volume of the box?
Ans with explanations:
Length =40-16= 24 m, [because 8+8 = 16]
Breadth = (60-16) = 44m,
Height = 8 m.
Volume of the box = (24 x 44 x 8) m cube = 8448 m^3.(Ans)
..........#GikoGiko
210.3 pipes A, B & C can fill a tank from empty to full in 30 mins, 20
mins & 10 mins respectively. When the tank is empty, all the 3
pipes are opened.A, B & C discharge chemical solution P, Q & R
respectively.What is the proportion of the solution R in the liquid of
tank after 3 mins?
Ans with explanations:
After 3 minutes total solution =(3/30+3/20+3/10)=33/60
After 3 minutes chemical R =3/10
so, proportion of solution R=3/10*60/33=6/11
.......#RaselArahan
211.At a dinner party every two guests shared a bowl of rice
between them,every 3 guests shared a bowl of vegetable between
them & every 4 guests shared a bowl of meat between them.There
were altogether 65 bowls.How many guests were present at the
party?
Ans with explanations:
For Rice =(1,2) (3,4) (5,6) (7,8) (9,10) (11,12) [total of 6 dishes].
For Vegetables (1,2,3) (4,5,6) (7,8,9) (10,11,12) [total of 4 dishes].
For Meat (1,2,3,4) (5,6,7,8) (9,10,11,12) [total of 3 dishes].
So 12 guests used 13 dishes,
So 13 dishes by 12 guests,
65 dishes by x guests,
13 : 12 :: 65 : x.
=>x = 60(Ans)
212.A farmer build a fence around his square plot.He used 27 fence
poles on each side of the square.How many poles did he need
altogether?
Ans with explanations:
Since each pole at the corner of the plot is common to its two sides, So we
have :
Total number of poles needed = 27 x 4 - 4 = 108 - 4 = 104(ans)
............#TomKS
213.A dept store received a shipment of 1000 shirts, for which it
pays tk 9000.The store sells the shirts at a price 80% above the cost
for 1 month,after which it reduces the price of the shirts 20% above
the cost.The store sells 75% of the shirts during first month & 50%
of the remaining shirts afterward.How much gross income did sales
of the shirts generate?
Ans with explanations:
Shipment=1,000 shirts
Pays=Tk.9,000
The cost per shirt was 9 tk.
Selling price for that month =1.8 x 9 = Tk.16.20
The number of shirts was selling= 0.75 x 1,000 = 750 shirts
The revenue earned is 750 x 16.2 = Tk.12,150.
After that month, 50% of the remaining shirts or 0.5 x 250 = 125 shirts
Sold at a cost of 1.2 x 9 = Tk.10.80.
The revenue earned from the sale of these shirts was= 125 x 10.8 = Tk.1,350.
So, the gross income earned was 12,150 + 1,350 = Tk.13,500(Ans)
..................#TomKS
214.Mr.Yousuf decided to walk down the escalator of a tube
station.He found that if he walks down 26 steps, he requires 30
seconds to reach the bottom.However if he steps down 34 stairs he
would only require 18 seconds to get the bottom.If the time is
measured from the moment the top steps begin to descend to the
time he steps off the last step of the bottom,find out the height of
the stair way in steps?
Ans with explanations:
Suppose,
the speed of escalator be x steps/sec
total steps in escalator is 26 + 30x
In the same way, the second case the total no of steps is 34 + 18x
ATQ,
26 + 30x = 34 + 18x
x = 2/3
So, steps 26 + 30*2/3 = 46 steps(Ans)
.......#TomKS
215.A railway half ticket cost half the full fare & the reservation
charge is the same on half ticket as on ticket.One reserved 1st
class ticket from Jessore to Rajshahi costs tk 216 and one full &
one half reserved 1st class tickets cost tk 327.What is the cost of
the 1st class full fare & the reservation charge?
Ans with explanations:
Let,
The reserved charge be x tk
& The 1st class charge for full ticket be y tk&
ATQ-1;
x + y = 216..................(1)
ATQ-2;
x+y+(x/2)+y=327
=>3x+4y = 654.............(2)
=>3*(216-y)+ 4y =654
=>y=6
From equation-1,we get
x+6=216
=>x=210
So,The cost of the 1st class full fare is 210 tk &
The reservation charge is 6 tk(ans)
...... Giko Giko
216.What is the radius of the in-circle of the triangle whose side
measures 5,12 & 13 units?
Ans with explanations:
5, 12 and 13 is a Pythagorean triplet. So, the triangle is a right triangle.
Area of a triangle = 1/2*b*h, where 'b' is the base and 'h' is the height of the
triangle.
In this right triangle, if the base is 12, the height will be 5 or vice versa.
In either case, area = 1/2*5*12 = 30 sq units.
Area of a triangle = r * s, where 'r' is the radius of the inscribed circle and 's' is
the semi perimeter of the triangle.
Semi perimeter = a + b + c/2=5+12+13/2= 15
Now,
30 = r * 15
Or r = 2 units.(Ans)
..........#GikoGiko
Alternate solution
Here,
Triangle with sides 5,12 ,13 is a right angled triangle.
Raius of incircle = 2(area of triangle)/(sum of sides) = 2(1*5*12/2) / (5+12+13)
=60/30 = 2 units(ans)
Another Alternate solution in following image

217.Before budget,a businessman has increased the price of his


product by 12%.In the budget a 10% sales tax(to be paid by seller)
was imposed on that product.As a result, the profit ultimately
increased by tk 64.What was the original selling price?
Ans with explanations:
Suppose,
OP= original selling price =100tk
At 12% increased, PP=112
After deducting 10% sales tax, pp=112*90/100=100.8
Profit increased=100.8-100=0.8
When profit increased tk. 0.8, OP is tk. 100
When profit increased tk. 1, OP is tk. 100/0.8
When profit increased tk. 64, OP is tk. 100*64/0.8= tk.8000(Ans)
..........#TomKS
218.The number of flights leaving a certain airport doubles during
every 1 hr period between its 9am opening and noon,after noon, the
no of flights leaving from the airport doubles during every 2 hr
period if 4 flights left from the airport between 9am & 10am,how
many flights left airport between 2pm & 4pm?
Ans with explanations:
Between 9am-10am number of flights left 4
so,between 10am-11am number of left flights 8
between 11am-12pm numbers of flight left 16
between 12pm-2pm numbers of flights left 32
between 2pm-4pm number of flights left 64(Ans)
.......#RaselArahan
219.1/5 th of the products made by a company are defective.4/5th of
the defectives were rejected &1/20th of the products were rejected
by mistake.What percent of the products sold by the company is
defective?
Ans with explanations:
Let,total product =100
Defective product =(100/5)=20
Good product 100-20=80
Defective product reject 20*4/5=16
Defective product sold 20-16=4
Good product rejected 80/20=4
Good product sold 80-4=76
Total sold 76+4=80
Defective sold 4*100/80=5%(Ans)
......#RaselArahan
220.How many number of digits are used in numbering of a book
having 366 pages?
Ans with explanations:
In total there are 366 pages! out of which First 9 pages having 1 digit nos
(1x9=9 digits).
Next 90 pages having 2 digit nos (2 x 90=180 digits).
Remaining 267 pages having 3 digit nos (267x3 = 801 digits) so total will be
9+180+801 = 990 digits(Ans)
.......#GikoGiko
221.If the degree measures of two angle of an isosceles triangle are
in the ratio 1:3,what is the degree measure of the largest angle if it
is not a base angle?
Ans with explanations:
Let the base angle be x. Then the other angle is also x.
Ratio given is 1:3, so the largest angle is 3x.
Now,We have, sum of three angles of a triangle =180 degree
So,x + x + 3x=180
=>5x = 180
=>x = 36
So, the largest angle = 3*36 = 108 degrees(ans)
.......#AsmaBinteSalahuddin
222.When 3 numbers are divided by 5,the remainders are 0,3,4
respectively.What could be the sum of the 3 numbers?
Ans with explanations:
may be the numbers are 5, 8, 9 or 5, 13, 9 or 5, 8, 14 or 10, 13, 9 etc.
so the sum = 22 or 27 or 27 or 32 etc.
= 5*4+2 or 5*5+2 or 5*5+2 or 5*6+2
= 5*(1+3)+2 or 5*(2+3)+2 or 5*(2+3)+2 or 5*(3+3)+2
= 5*(n+3) +2 [n=natural number]
= 5n+17 (Ans.)
..........#AsmaBinteSalahuddin
223.In a bag, there are 6 identical green marbles, 4 identical blue
marbles & 2 identical red marbles.If the marbles are picked at
random from the 12 in the bag, how many distinct sets of 3 can be
selected?
Ans with explanations:
there r 3 identical group.
total set = all uncommon or 1 uncommon and 2 common or 3 common
= 3C3 + 2C1 x 3 + 3c0 x 2 [***]
=1+3x2+1x2
=9 (Ans)
.........#AsmaBinteSalahuddin
NB : *** 3C3 + 2C1 x 3 [2 common can take from all 3 groups] + 3c0 x 2 [3
cant be common from red]
224. A man has to take a hen, a fox & some corn across a river.He
can take only one thing across at a time.Unless the man is present
the fox will eat the hen & the hen eats the corn.How is it done?
Ans with explanations:
The man and the chicken cross the river, (the fox and corn are safe together),
he leaves the chicken on the other side and goes back across.
The man then takes the fox across the river, and since he can't leave the fox
and chicken together, he brings the chicken back.
Again, since the chicken and corn can't be left together, he leaves the chicken
and he takes the corn across and leaves it with the fox.
He then returns to pick up the chicken and heads across the river one last
time.
.....Giko Giko
225.A contractor employed 30 men to do a piece of work in 38
days.After 25 days,he employed 5 men more & the work was
finished one day earlier.How many days he would have been
behind, if he had not employed additional men?
Ans with explanations:
Given that,
Total number of days=38
Number of days completed =25
Number of man=30
Number of man increased =35
Remaining number of Days=(38-25)=13
ATQ;
After 25 days 35 men complete the work in (13-1)= 12 days
Hence,30 men complete the work in (35*12/30) days =14 days,
which is 1 day behind (ans)
226.Among 120 students of a class, numbered 1 to 120, all even
numbered students opt for Bangla, whose numbers are divisible by
5 opt for English & those numbers are divisible by 7 oot for
math.How many opt for none of these three subjects?
Ans with explanations:
Let,
The set of No. of students who took at least one of the 3 subjects
=n(A∪B∪C),where A is the set of students who took Physics, B is the set of
students who took Chemistry and C is the set of students who opted for Math.
Set A(2): 2...120 ==> (120-2)/2 + 1 = 60
Set: B(5): 5...120 ==> (120-5)/5 + 1 = 24
Set C(7): 7...119 ==> (119-7)/7 + 1 = 17
Both(2,5): 10...120 ==> (120-10)/10 + 1 = 12
Both(2,7): 14...112 ==> (112-14)/14 + 1 = 8
Both(5,7): 35...105 ==>(105-35)/35 + 1 = 3
All three(2,5,7): 70 ==>1
As,we have
n(A∪B∪C)= n(A) + n(B) + n(C) - {n(A ∩ B) + n(B ∩ C) + n(C ∩ A)} + n(A∩B∩
C)
=60+24+17-(12+8+3)+1
=79
So,Number of students who opted for none of the three subjects
= 120 - 79 = 41(Ans)
227.3 math classes X, Y & Z take an algebra test.The avg score in
class X is 83.The avg score in class Y is 76 & the avg score in class
Z is 85.The avg score of all students in classes X & Y together is
79.The avg score of all students of the classes of Y & Z together is
81.What is avgvg of all these three classes?
Ans with explanations:
According to the given condition
(83x+76y)/(x+y) = 79
on solving x = 3y/4
similarly (76y+85z)/(y+z) = 81
on solving z = 5y/4
average score for all the three classes taken together = 83x+76y+85z/x+y+z
on solving avg. of the all three will be 978y/12y=81.5 (Ans)
......Giko Giko
228.A man deposited tk 50000 at a certain interest rate for 1
year.After 1 year, he received tk 55280 as both principal &
interest,after deduction of tk 120 as govt levy & 10% on interest as
govt tax.What was the interest rate in percentage?
Ans with explanations:
Interest after deducting Tax and Levy => 55280-50000 = Tk.5280
Interest Before deducting levy => 5280+120 = Tk.5400
Now, 90% ≡ 5400; then 100% ≡ 5400*100/90 = Tk.6000
That is, Interest before deducting Tax and Levy = Tk.6000
So, Interest Rate = (6000/50000)*100 = 12% (Ans.)
.......Rasel Arahan
229.A certain club has 10 members including Harry.One of the 10
members is to be chosen at random to be the president.One of the
9 members is to be chosen at random to be the secretary & one of
the remainig 8 members is to be chosen at random to be the
treasurer.What is the probability that Harry will be either the
member chosen to be the secretary or the member chosen to be
the treasurer?
Ans with explanations:
When Harry is chosen as secretary the probability is= 9/10 * 1/9 * 8/8 = 1/10
(9 available for post of president out of 10, then Harry has to be chosen out of
9, and finally out of 8 anyone can be treasurer)
Again,
When Harry is chosen as treasurer the probability is
= 9/10 * 8/9 * 1/8
= 1/10
(9 available for post of president out of 10, then out of 8 anyone can be
secretary, and finally Harry has to be treasurer)
So, the probability that Harry will be either the member chosen to be the
secretary or the member chosen to be the treasurer
= 1/10 + 1/10
=1/5(ans)
230.Ronaldo & Messi have two children.The probability that the 1st
child is a girl is 50%.The probability that the 2nd child is a girl is
also 50%.Ronaldo & Messi tell you that they have a daughter.What
is the probability that their other child is also a girl?
Ans with explanations:
1st child is a girl 50% that is it can be boy also 50%.
& 2nd child is a girl 50% that is it can be boy also 50%.
Therefore, the child can be boy or girl, its possibilities can be shown in the
following figure:-
........... (B) ............(G)
.
(B) ..... BB...............BG
.
(G) ......GB...............GG
Total outcomes
= {BB, BG, GB, GG}
BB is not acceptabe because according to question 50% girl.
So total outcomes reduces to {BG, GB, GG}
that is 3 points
Two childs can be daughter or girl such favorable outcome only one point
which is {GG}
So,
Probability= favourable outcome/total outcome= 1/3(ans)
...............#SamratRoySandhan
231.A clock is set right at 8 a.m.The clock gains 10 mins in 24
hour.What will be the true time when the clock indicates 1 p.m. on
the following day?
Ans with explanations:
Total Time from 8 a.m. on a day to 1 p.m. on the following day = 29 hours.
24 hours 10 min. of this clock = 24(1/6)=145/6 hours of the correct clock.
145/6 hrs of this clock = 24 hours of the correct clock.
29 hours of this clock = 24*6*29/6hrs of the correct clock
= 28 hrs 48 min of the correct clock.
Therefore, the correct time is 28 hrs 48 min. after 8 a.m. or 12 mins before
1:00 pm
=12:48 pm(ans)
232.A dog is tied to a post in a corner where two fences meet at an
angle of 120 degree.If the length of the rope is 8 feet over how
many square feet can the dog roam?
Ans with explanations:
The dog covered=120/360
=1/3 area of the circle
Area of the circle=πr^2
=3.1416*64
=201.06
So, The dog covered=201.06/3
=67.02 (Ans.)
.........Sarker Mamun
233.A fires 5 shots to B's 3 but A kills only once in 3 shots while B
kills once in 2 shots.When B has missed 27 times, then A has
killed---
Ans with explanations:
B missed 1 in 2 shots.
B missed 27 in 54 shots.
when B fires 3, A done 5 shots
when, B fires 54, A done=(5*54)/3=90 shots
in 3 shots A killed 1
in 90 shots, A killed =90/3=30(Ans)
........#JayantaKumarBarman
234.An organization decided to raise tk 6lac by collecting equal
contribution from each of its employees.If each of them contributed
tk 60 extra,the contribution would have been tk 6.24lac.How many
employees are there in that organization?
Ans with explanations:
Let,
Each employee contribute x tk and number of employees be y
then xy = 600000.........(1)
If each of them had contributed Rs. 60 extra, the contribution would have
been Rs. 624000
ATQ-2;
y(x+60) =624000.........(2)
Applying (2)-(1)
Hence, y(x+60) - xy = (6240000 - 600000)
=> 60y=24000
=> y=400
Total no. of employees are 400(ans)
235.Free notebooks were distributed equally among the childrens
of a class.The no. of notebooks each child got was 1/8 th of the no.
of children.Had the no. of children been half,each child would have
got 16 notebooks.Total how many notebooks were distributed?
Ans with explanations:
Let,total no. of childrens be x
ATQ;
x*1*x/8=x*16/2
=>x=64
So,total notebooks were distributed=64**16/2=512(ans)
236.In a square the four unshaped part are quarter circles each
having the same radius.If the side of the square is 16,what is the
area of the shaped part?
Ans with explanations:
Area of square is 16*16=256
Radius of each quarter
circle is 16/2=8
area of each quarter circle is
π8^2/4=16π
Area of upshaped part is 16π*4=201.14
Area of shaped part =256-201.14=54.86(Ans)
..........Rasel Arahan
237.A brand of paint costs tk 14 per litres & 1 litre of paint will cover
an area of 150 square feet.What is the minimum cost of paint
needed to cover the 4 walls of a rectangular room that is 12 feet
wide,16 feet long & 8 feet high?
Ans with explanations:
Total area of the 4 wall=2*(l+b)*h
=2*(12+16)*8=448 sqft
ATQ; 14 tk spend in covering 1 litre paint
So,for covering 150 sqft need 14 tk
for covering 448 sqft need 448*14/150=41.81 tk=42 tk(approximately) (ans)
238.A rectangular carpet covers half of a rectangular floor with the
dimensions 9 feet by 12 feet.If the ratio of their dimensions is
same,then what is the length of the carpet?
Ans with explanations:
Given that,
Length of the floor,a= 12 ft & width,b= 9 ft
So,Area of floor =a*b= 12 * 9 = 108
ATQ-1;
Area of carpet is half that: 54
As,the dimensions of the carpets are in the same ratio as the floors
ATQ-2;
a:b=12:9
=>b=3a/4
Now, carpet area= ab sqft
So,ab=54
=>3a*a/4=54
=>a^2=72=36*2
=>a=6sqrt2
Thus,the length of the carpet=6sqrt2(ans)
239.A man buys tk 20 shares paying 9% dividend. The man wants
to have an interest of 12% on his money.What is the market value
of each share?
Ans with explanations:
Dividend on tk 20 = 9%of 20=9/5 tk
ATQ;tk 12 is an income on tk. 100
So, tk 9/5 is an income on tk. 100*9/12*5 = 15 tk (ans)
240.There is a road covering the outside of a square garden of 11
yard broad.If the area of the road is 4 acre.What is the perimeter of
the garden?(in metric unit)
Ans with explanations:
Let,each side of garden x yard
Each side of garden with road x+11*2=x+22
ATQ,(x+22)^2-x^2=4*4840
=>x=429
perimeter=4*429=1716 yard or 5148 feet or 156911.04 cm or 1569.11
meters(Ans)
Note:1 acre=4840 yards
1 yard=3 feet
1 feet=30.48 cm
.....Rasel Arahan
241.A man bought a house for tk 10lacs & rents it.He puts 12.5% of
each month's rent aside for repairs,pays tk 1660 as annual taxes &
realizes 10% on his investment thereafter.What is the monthly rent
of the house?
Ans with explanations:
Given that,
Total investment 1000000 tk
Since,10% on his investment
He realizes after one year
=(10*1000000/100) tk
=100000 tk
Let him rents it at Rs. x per month.
So,In 12 months he receives Rs. 12x tk
Out of this 12x he sets out
=12x*(12.5/100) tk
= 1.5x tk for repair
Pays tax =1660 tk
Hence,12x=100000+1.5x+1660
=>10.5x = 101660
=>x = 9681.90
So,the monthly rent is 9681.90tk(ans)
242.A 3 digit number 4a3 is added to another 3 digit number 984 to
give a 4 digit number 13b7, which is divisible by 11.Then a+b =?
Ans with explanations:
ATQ;
4a3
984
----------
13 b 7
=> a + 8 = b -----------(Equation 1)
Again,
13b7 is divisible by 11
=> (1 + b) - (3 + 7) is 0 or divisible by 11
=> (b - 9) is 0 or divisible by 11 -----------(Equation 2)
Assume that (b - 9) = 0
=> b = 9
Substituting the value of b in Equation 1,
a+8=b
a+8=9
=> a = 9 - 8 = 1
If a = 1 and b= 9,
(a + b) = 1 + 9 = 10 (ans)
243.The cost in tk of manufacturing x microwaves is 6000+300x
tk.The amount received when selling these x microwaves is 700x
tk.What is the least no. of microwaves that must be manufactured &
sold so that net profit is tk 2400?
Ans with explanations:
selling price-cost price= profit
so, 700x-(6000+300x)=2400
=>x=21(ans)
.........Saiduzzaman Titu
244.A bank has a parking lot of 70 parking spaces.Each row in the
parking lot contains the same no. of parking spaces.The bank has
purchased additional land for expansion of the office space.When
the expansion is made,two parking spaces will be lost from each
row,however 4 more rows added to the parking lot.After the
expansion is made the capacity of parking lot will remain same &
each row contain same no. of parking spaces.How many rows were
there in the parking lot before the expansion was made?
Ans with explanations:
Let,Total rows be x
Parking spaces per no. of rows
= 70/x
ATQ;
(70/x)-2=70/(x+4)
=>x=10, -14(not accepted as neg value)
So,total rows 10 in the parking lot(ans)
245.Given 2E+2F=6G,2E-2F=2G & F-G=0,how many unique solution
are there for G?
Ans with explanations:
Given that,
F= G......(1)
2E-2F= 2G
=> E= F+G..... (2)
& 2F+2E= 6G
=>2F+2(F+G)= 6G
[Using (2)]
=>4F= 4G
=>4G= 4G [using(1)]
From here we can't find out the value of 'G'.
So, it has infinite solution i.e.
it has no any unique solution.
.......#SamratRoySandhan
246.The interest charge on loan is tk p per tk 1000 for the 1st
quarter & q per tk 1000 for each month after the 1st quarter.How
much interest will be charged for the 1st year on tk 1000?
Ans with explanations:
Given that,
Loan interest charge for first quarter
=100*p/1000=p/10 tk
Loan amount=1000 tk
Loan interest charge for the later months of the year=q/10 tk
Interest for first quarter=1000*p*1/10*4=25p tk
Interest for the later months of that year=1000*3*q/10*4=75q tk
So,total interest earned on that year=(25p+75q) tk (ans)
247.Marium has n chocolates where n is an integer such that
20<n<50.If Marium divides the chocolates equally among 5
children,she will have 2 chocolates remaining. If she divides the
chocolates among 6 children, she will have 1 chocolates remaining.
How many chocolates will remain if she divides chocolates among
7 children?
Ans with explanations:
Let N be the total candies.
ATQ;
N = 5a + 2............1
N = 6b + 1.............2
Applying (2)-(1)
6b - 5a = 1 ( 5a can be 25,30,35,40,45)
So,we have to find a multiple of 6 which is 1 more than multiple of 5,
Hence,
Only 36,35 satisfy this condition.
So total = 37 candies, 37/7 -> remainder = 2 (ans)
248.Mr. Karim deposited a certain amount of money for a fixed
period of time. On maturity, he received a total of Tk. 45000 when
the ratio of interest and investment became 2:5. If the interest rate
was 3.6%, calculate the time period for which the money was
invested?
Ans with explanations:
ATQ,
Interest = 45000×2/7
Investment = 45000×5/7
Let, time period = n
So, 45000*5/7*n*3.6/100=45000*2/7
=> n = 45000*2/7*100/3.6*7/(5*45000)
=> n=11(1/9)yrs (ans)
.........Asma Binte Salahuddin
249.After being dropped a certain ball always bounces back to 2/5
of the height of its previous bounce. After the first bounce it
reaches a height of 125 inches. How high (in inches) will it reach
after its fourth bounce?
Ans with explanations:
1st bounce = 125
2nd bounce = 2/5*125 = 50
3rd Bounce = 2/5*50 = 20
4th bounce = 2/5*20 = 8 (Ans)
.....Giko Giko
250.A man started a business investing TK. 70,000. Rakhi joined
him after six months with an amount of TK. 1,05,000 and Sagar
joined them with TK. 1,40,000 after another six months. The amount
of profit earned should be distributed in what ratio among Aman,
Rakhi and Sagar respectively, 3 years after Aman started the
business?
Ans with explanations:
Aman : Rakhi : Sagar = (70,000 x 36) : (1,05,000 x 30) : (1,40,000 x 24)
= 12 : 15 : 16 (Ans)
......Giko Giko
251.A team of 3 men and 5 women completed one-fourth of the job
in 3 days. After 3 days, another man joined and they took 2 days to
complete another one-fourth of job. How many men can complete
the whole job in 5 days?
Ans with explanations:
Given that,
3 men + 5 women can do 1/4 parts of work in 3 days
So,......................................1 part of work in (3*4) = 12 days
Again,(4 men + 5 women) can do 1/4 parts of work in 2 days
So,...............................................1 part of work in 8 days
We get,
(3 men + 5 women) can do in 1 day = 1/8 parts
& (4 men + 5 women) can do in 1 day = 1/12 parts
So, 1 man can do in 1 day = (1/8 - 1/12) = 1/24 parts
Hence, 1/24 parts can do by 1 man in 1 day
Then, 1 part can do by 1 man in 24 days
By, 24 days needed 1 man
So, 1 day needed (24*1)=24 men
So, 5 day needed 24/5 = 4.8 men = 5 men (ans)
252.A computer company offered to sell a city agency new machine
at a 15% discount from the price list and to allow the agency
Tk.8500 for each of two old computers being traded in the list price
of the new machine is Tk.62500. If the agency accepts the offer,
how much money will it have to pay for the 4 new machines?
Ans with explanations:
Total selling Price for 4 PCs is tk 62500*4= Tk 250000
After deducing 15% discount,
Final selling price is Tk 250000*85% = Tk 212500.
For 2 old computer the agency get Tk 8500*2 = Tk 17000
Hence, Total payable amount is Tk (212500-17000) = Tk 195500.(ans)
.......Giko Giko
253.During transportation 5% merchandise was stolen and another
5% was damaged. At what profit (in percentage) should the goods
be sold so as to make an overall profit of 20%?
Ans with explanations:
Total damaged 10%
Remaining value=100-10 =Tk.90
At 20% profit, Sp=Tk.120
Then, profit=120-90=Tk.30
So, profit %=30*100/90=33.33% (Ans.)
Sarker Mamun
254.A jar contains marbles of 4 different colour. The number of blue
marbles is three times as many as the yellow ones. The number of
red marbles is half that of the yellow ones. The number of orange
marbles is equal to the number of red ones. What percent of the
marbles are orange colored?
Ans with explanations:
Let, Yellow marbles be x
Blue marbles be 3x,
Red marbles be x/2
Orange marbles x/2
% of orange colour =(x/2)*100/5x=10%(ans)
255.The average price of a new car increased by 20% from 1990 to
1995 and again increased by 20% from 1995 to 2000. If the average
price of a new car was tk. 450000 in 1995, what is the difference in
average price of a car between year 1990 and 2000?
Ans with explanations:
1990 increased by 100%
1995 """""" 120%
2000 """""""""""" 144%
Now, 120% is............. 450000
Then, 44% is..44*450000/120
=165000 (Ans.)
........Sarker Mamun
256.Rina and Shila entered into a partnership with their capitals in
the ratio 5:6. At the end of 8 months, Rina withdraws her capital. If
they receive the profit in the ratio of 5:6. At the
end of 8 months, Rina withdraws her capital. If they receive the
profit in the ratio of 5:9, find how long shila’s capital was used?
Ans with explanations:
Let A invested for x months.
Ratio of profit A : B = 5*x: 6*8
ATQ;
5x:48 = 3:2
=>x = 14(2/5) months(ans)
257.A train having a length of 1/4 mile, is traveling at a speed of 75
mph. It enters a tunnel 3 ½ miles long. How long does it take the
train to pass through the tunnel from the moment the front enters
to the moment the rear emerges?
Ans with explanations:
Total distance covered = ( 7/2+ 1/4)miles = 15 /4 miles.
Therefore Time taken
={15/(4 x 75)} hrs=1/20 hrs=60/20=3mins(ans)
258.The distance between two stations, Dhaka and Chittagong is
450km.A train stars at 5 pm from Dhaka and moves towards
Chittagong at an average speed of 60km/hr. Another train starts
from Chittagong at 4.20 pm and moves toward Dhaka at an average
speed of 80km/hr. At what time will the two trains meet?
Ans with explanations:
Given that,
Total distance = 450 km
(4:20pm-5:00pm)=40/60=2/3 hrs
Let,the two trains met in x hrs after 5:00pm
ATQ;
60t+80(t+2/3)=450
=>t=17/6
=>t=17*60/6
=>t=170 mins
=>t= 2 hrs 50 mins
So,the two trains will meet at 5:00 pm + 2 hrs 50 mins =7:50 pm(ans)
259.At a college football game, 4/5 of the seats in the lower deck of
the stadium were sold. If one fourth of all the seating in the stadium
is located in the lower deck and if 2/3 of all the seats in the stadium
were sold, what fraction of the unsold seats in the stadium was in
the lower deck?
Ans with explanations:
Let,Total seats be x
Then, total sold = 2x/3
So,total unsold seats = x - 2x/3 = x/3
Total seats in the lower deck = x/4
& Total seats sold in the lower deck = 4x/4*5 = x/5
So,Total unsold seats in lower deck = x/4 - x/5 = x/20
The required function = x/20 / x/3 = 3/20 (ans)
260.In a cricket match, five batsman A,B, C,D and E scored an
average of 36 runs .D scored 5 more than E; E scored 8 fewer than
A; B scored as many as B and E combined and B and C jointly
scored 107. How many runs did E score?
Ans with explanations:
Given that,
Five batsmen total = 5*36 = 180
Let, E = x, D = x +5, A = x + 8 & B + C = 107
ATQ;
A + B + C + D + E = 180
=>(x + 8) + 107 + (x + 5) + x = 180
=>x = 20
So, E score 20 runs (ans)
261.In a family, each daughter has the same number of brothers as
she has sisters and each son has twice as many sisters as he has
brotheTK. How many sons are there in the family?
Ans with explanations:
Let,daughter =x
son=y
thus,x-1=y......i
again,(y-1)2=x......ii
Equation i-ii
x-y=1
x-2y=-2
--------------
=>y=3
Number of sons =3 (Ans)
262.A train can travel 50% faster than a car. Both start from point A
at the same time and reach point B 75 kms away from A at the same
time. On the way, however, the train lost about 12.5 minutes while
stopping at the stations. What is the speed of the car?
Ans with explanations:
Let,speed of car=x km
speed of train =1.5x
ATQ,
75/x-75/1.5x=12.5/60
=>x=120
Speed of the car=120 km/h (Ans)
.....Rasel Arahan
263.A sum of tk. 725 is lent in the beginning of a year at a certain
rate of interest. After 8 months, a sum of tk. 362.50 more is lent but
at the rate twice the former. At the end of the year, tk. 33.50 is
earned as interest from both the loans. What was the original rate
of interest?
Ans with explanations:
Let the original rate be R%. Then, new rate = (2R)%.
Here, original rate is for 1 year(s); the new rate is for only 4 months or 1/3
years
ATQ;
(725*R/100*1) + (362.50*2R/100*1/3) = 33.50
=>R = 3.46
Now,Original rate is 3.46% (ans)
264.Two stations A and B are 110 km apart on a straight line. One
train starts from A at 7 a.m. and travels towards B at 20 kmph.
Another train starts from B at 8 a.m. and travels towards A ata
speed of 25 kmph. At what time will they meet?
Ans with explanations:
Let,they will meet x hour later after starting the first train
ATQ,
20x+25(x-1)=110
=>x=3
so,they will meet at 10 a.m(Ans)
265.The captain of a cricket team of 11 members is 26 years old and
the wicket keeper is 3 years older. If the ages of these two are
excluded, the average age of the remaining players is one year less
than the average age of the whole team. What is the average age of
the team?
Ans with explanations:
Let,average age of team =x
Average age of team without captain and wicket keeper =(x-1)
ATQ,
11x-(26+29)=9(x-1)
=>x=23
Average age of the team =23(ans)
266.A, B and C jointly thought of engaging themselves in a
business venture. It was agreed that A would invest tk. 6500 for 6
months, B tk. 8400 for 5 months and C tk. 10,000 for 3 months. A
wants to be the working member for which, he was to receive 5% of
the profits. The profit earned was tk. 7400. Calculate the share of B
in the profit.
Ans with explanations:
For managing, A received = 5% of Rs. 7400 = Rs. 370.
Balance = Rs. (7400 - 370) = Rs. 7030.
Ratio of their investments = (6500 x 6) : (8400 x 5) : (10000 x 3)
= 39000 : 42000 : 30000
= 13 : 14 : 10
So,B's share in the profit =7030 x 14 /37 = 2660 tk (ans)
267.The sum of three numbers is 68. If the ratio between first and
second be 2:3 and that between second and third be 5:3, then the
second number is?
Ans with explanations:
1st : 2nd = 2:3 = 10:15
2nd : 3rd = 5:3 = 15:9
1st : 2nd : 3rd = 10:15:9
So, the 2nd number is= 68*15/34 = 30 (ans)
268.If the first 200 numbers are written down and those divisible by
2 are deleted and again those divisible by 5 are deleted, how many
numbers are left out?
Ans with explanations:
Total numbers divisible by 2 = 100
Total numbers divisible by 5 = 40
But there is double counting. So we have to subtract Total numbers which are
divisible by both 2 and 5 i.e, 200 / 10 = 20
So Total numbers which are divisible by either 2 or 5
=100 + 40 - 20 = 120
Number of numbers which are not divisible by any of those
= 200 - 120 = 80(ans)
269.Gold is 19 times as heavy as water and copper 9 times as
heavy as water. The ratio in which these two metals be mixed so
that the mixture is 15 times as heavy as water is?
Ans with explanations:
Given that,
G = 19W (As gold is 19 times as heavy as water)
C = 9W (copper is 9 times as heavy as water)
Mixing A pounds of gold with B pounds of copper:
A(G) + B(C) = 15W(A + B)
=>A(19W) + B(9W) = 15W(A + B)
Divide both sides by W
=>19A + 9B = 15(A + B)
=>19A + 9B = 15A + 15B
=>4A = 6B
=>A = (3/2)*B
For each part of Gold, there needs to be 1.5 parts Copper
So,The Ratio Gold to copper: 1:1.5 (ans)
270.Drum X is 1/2 full of oil and drum Y which is twice the capacity
of drum X, is 2/3 full of oil. If all the oil in drum X is poured into
drum Y, then drum Y will be filled to what fraction of its capacity?
Ans with explanations:
Drum X capacity 3 units
Drum Y capacity 6 units
Drum X contains =3*1/2=1.5 units.
Drum Y contains =2*6/3 = 4 units.
Drum Y gets filled 5.5 after pouring 1.5 from A to B.
Hence, Drum Y will filled to 5.5/6 or 11/12 of its capacity.(ans)
271.The price of a shirt and a trouser is Tk.525. If the prices of a
shirt and a trouser are increased by 5% and 10% respectively, then
to buy those it costs Tk.568.75. What is the cost price of each of a
shirt and a trouser?
Ans with explanations:
Let the price of shirt = x
and the price of trouser is = y
x+y= 525
=> x = 525-y. ....... (i)
1.05x + 1.1y = 568
=> 1.1y = 568 - 1.05x
=> 1.1y = 568 - 1.05 (525-y)
=> 1.1y = 568 - 551.25 + 1.05y
=> .05y = 16.75
=> y = 335
So, x = 525- 335 = 190
the cost price of each of a shirt and a trouser = tk. 190 & 335(ans)
Asma Binte Salahuddin
272.The length of a pool is 60m and its breadth is 40m. If the width
of the bank of the pool is 3m, then what is the area of the bank of
the pool?
Ans with explanations:
Given that,
The length is 60m & breadth is 40m
So,the area is 60*40=2400 sqm
Given,the width of the bank of the pool is 3m
Length without pool = 60+3*2 = 66m
Breadth without pool = 40+3*2 =46m
So,Area of the bank of the pool = 66*46-2400= 636 sqm (ans )
Asma Binte Salahuddin
273.The length, breadth and height of a rectangular copper solid
are 11m, 10m and 5m respectively. It is melted and re-casted into a
number of spheres of diameter 50cm. Find the number of spheres.
Ans with explanations:
Volume of the copper solid
= (11×100)×(10×100)×(5×100) cm^3 (1m =100cm)
= 550000000 cm^3
Volume of a sphere = 4/3* π* (50/2)^3= 4/3* 22/7 * 25^3
number of spheres = 550000000 ÷ 4/3* 22/7 * 25^3= 8400 (ans )
Asma Binte Salahuddin
274.A 19 liter mixture consists by volume of 1 part juice to 18 parts
water. If z liters of juice and y liters of water are added to this
mixture to make a 54 liter mixture consisting by volume of 1 part
juice to 2 parts water, then what is the value of x( means how much
juice to be added)?
Ans with explanations:
In 19 L; J = 1L, W = 18L
In 54L; J:W = 1:2; So, J = 18L, W = 36 L
So, Juice to be added in 54 litre mixture = 18-1 = 17L (Value of x)
(ans)
........Joy Shaha
275.Two tanks X and Y are filled to capacity with fuel. Tank X holds
600 gallons more than tank Y. If 100 gallons of fuel were to be
pumped from each tank, tank X would contain 3 times as much fuel
as tank Y. What is the capacity of each of the fuel tanks?
Ans with explanations:
say, tank Y contains p litres fuel
hence, tank X contains=p+600
after pumping 100 litres from each, tank X & Y have (p+500) & (p-100)
respectively.
atq, 3(p-100)=p+500
=> p=400
so capacity of Y =400 litres
capacity of X =400+600=1000 litres.(ans)
......Jayanta Kumar Barman
276.The average weight of 3 men A, B, C is 84 kg. Another man D
joins the group and the average now becomes 80 kg. If another
man E, whose weight is 3 kg more than that of D, replaces A then
the average weight of B, C, D and E becomes 79 kg. Find out the
weight of A?
Ans with explanations:
A+B+C=84*3=252....(1)
A+B+C+D=80*4=320....(2)
(2)-(1)=>D=68
E=68+3=71
Again, B+C+D+E=78*4=316
B+C+68+71=316
=>B+C=316-(68+71)=177
From (1), A=252-(B+C)=252-177=75 kg (ans)
......Jayanta Kumar Barman
277.Maria got married 6 years ago. Today her age is 1¼ times her
age at the time of marriage. Her son’s age is 1/10 times her age.
Find out her son’s age.
Ans with explanations:
Let, Maria's age at the time of marriage = x yrs
& her present age=x+6
atq, x+6=5x/4
x=24
so, present age=24+6=30 yrs
Her son's age=30/10=3 yrs (ans)
......Jayanta Kumar Barman
278.In a two-digit number, the digit in the unit`s place is more than
twice the digit in ten`s place by 1. If the digits in the unit`s place
and the ten`s place are interchanged, difference between the newly
formed number and the original number is less than the original
number by 1. What is the original number?
Ans with explanations:
Let, Tenth digit=x
Unit digit=2x+1
The number is =10x+2x+1=12x+1
ATQ;
{10(2x+1)+x} - (12x+1)=12x+1-1
=>x=3
So, the original number =12*3+1=37( Ans)
279.Helpers are required to prepare cakes. Each helper can make
either 2 large cakes or 35 small cakes per hour. The kitchen is
available for 3 hours and 20 large & 700 small cakes are required to
be prepared. What is the minimum number of helpers are required
to complete the task?
Ans with explanations:
35 small cake =2 large cakes
So, 700 small cakes = 40 large cakes
So, total large cake is 20+40=60 .
1 helper make in one hour 2 large cakes
So, 1 helper make in 3 hours 6 cakes
Now,men needed for making 60 large cakes 60/6=10(ans)
280.At a game of billiards, A can give B 15 points in 60 and A can
give C to 20 points in 60. How many points can B give C in a game
of 90?
Ans with explanations:
When A=60, B=45
When A=60, C=40
Difference of B & C is 5.
in 45, B can give C (45-40)= 5
in 90, B can give C = (5*90)/45=10 (ans)
281.The sum of ages of 5 children born at the intervals of 3 years
each is 50 year. What is the age of the youngest child?
Ans with explanations:
Let, Age of 5 childrens at the 3 yrs intervals are
x,(x+3),(x+6),(x+9),(x+12) respectively.
ATQ;
x+x+3+x+6+x+9+x+12=50
=>5x+30=50
=>x=4
So, Age of youngest 4yrs (ans)
282.Present ages of Sameer and Anand are in the ratio of 5:4
respectively. Three years hence, the ratio of their ages will become
11:9 respectively. What is Anand's present age in years?
Ans with explanations:
Let,
present age of Sameer & Anand are 5x & 4x respectively.
ATQ;
(5x+3)/(4x+3)=11/9
=>45x+27=44x+33
=>x=6
So,present age of Anand=4*6=24 yrs (ans)
283.The average of 20 numbers is zero. Of them, at the most, how many may
be greater than zero?
Ans with explanations:
Average of 20 numbers = 0.
Sum of 20 numbers (0 x 20) = 0.
It is quite possible that 19 of these numbers may be positive and if their sum is
a then 20th number is (-a). (Ans-19)
284.When positive integer x is divided by 5, the remainder is 3; and
when x is divided by 7, the remainder is 4. When positive integer y
is divided by 5, the remainder is 3; and when y is divided by 7, the
remainder is 4. If x > y, which of the following must be a factor of x-
y?
Ans with explanations:
When the positive integer x is divided by 5 and 7, the remainder is 3 and 4,
respectively: x=5q+3 (x could be 3, 8, 13, 18, 23, ...) and x=7p+4(x could be 4,
11, 18, 25, ...).
Divisor will be the least common multiple of above two divisors 5 and 7, hence
35.
Remainder will be the first common integer in above two patterns, hence 18 --
>
So, to satisfy both this conditions x must be of a type x=35m+18 (18, 53, 88,
...);
The same for y (as the same info is given about y): y = 35n+18;
x−y=(35m+18)−(35n+18)=35(m−n) -->
thus x-y must be a multiple of 35 (Ans)
285.Workers are grouped by their areas of expertise, and are placed
on at least one team. 20 are on the marketing team, 30 are on the
Sales team, and 40 are on the Vision team. 5 workers are on both
the Marketing and Sales teams, 6 workers are on both the Sales
and Vision teams, 9 workers are on both the Marketing and Vision
teams, and 4 workers are on all three teams. How many workers are
there in total?
Ans with explanations:
Total workers are = Marketing + sales + vision - (marketing and sales
+marketing and vision + vision and sales) + (in all three)
= 20+30+40 - (5+6+9) + 4 = 74 (answer)
286.In a consumer survey, 85% of those surveyed liked at least one
of three products: 1, 2, and 3. 50% of those asked liked product 1,
30% liked product 2, and 20% liked product 3. If 5% of the people in
the survey liked all three of the products, what percentage of the
survey participants liked more than one of the three products?
Ans with explanations:
Let, Liked exactly two be x
ATQ,
85 = 50 + 30 + 20 - x - 2(5) + 0
x=5
Then Liked more than 1 includes those who liked exactly 2 and those who
liked exactly 3.
Liked exactly 2 ---> From x = 5
Liked exactly 3----> Given 5
Total 5+5 =10 (Ans)
287.An aircraft uses 1680 gallons of fuel, the left tank uses 45%, the
right tank uses 32.5%, how much was used by the centre tank?
Ans with explanations:
Centre tank use=100-(45+32.5)=22.5%
So, 22.5% of 1680=(22.5*1680)/100=378 lires (ans)
288.A certain right circular cylinder has a radius of 5 inches. There
is oil filled in this cylinder to the height of 9 inches. If the oil is
poured completely into a second right cylinder, then it will fill the
second cylinder to a height of 4 inches. What is the radius of the
second cylinder, in inches?
Ans with explanations:
The volume of the liquid is constant.
Initial volume = π∗5^2∗9
New volume = π∗r^2∗4
ATQ; π∗5^2∗9=π∗r^2∗4 So,r=(5∗3)/2=7.5 inches(ans)
289.A man travels from A to B at a speed of x km/hr. He then rests
at B for x hour. He then travels from B to C at a speed of 2x km/hr
and rests for 2x hour. He moves further to D at a speed twice as
that between B and C. He thus reaches D in 16 hr. If distances A-B,
B-C, C-D are all equal to 12 km, then find the time for which he
rested at B.
Ans with explanations:
Total time taken by the man to travel from A to D = 16 hr and total distance
traveled =(12*3)= 36 km.
The time that he would have taken had he not rested in between will be (16-x-
2x) = (16-3x).
ATQ,
12/x + 12/2x + 12/4x = 16-3x
=> 21/x = 16-3x
:.x = 3 or 7/3 (Ans)
290.In a class of 40 students, the number of students who passed
the math exam is equal to half the students who passed the science
exam. Each students in the class passed at least one of the two
exams, if 5 students passed in both exam. Then how many
students passed in math exam?
Ans with explanations:
Let, number of students pass in science =2 x
math = x
both = 5
we know,
total = all single - both + none.
40 = 2x +x -5
x = 15
So, number of students pass in math =15.(Ans)
291.Diba has a wheat business. She purchases wheat from a local
wholesaler at a particular cost per pound. The price of the wheat at
her stores is Tk 3 per pound. Her faulty spring balance reads 0.9
pound for a pound. Also in the festival season, she gives a 10%
discount on the wheat. She found that she made neither a profit nor
a loss in the festival season. At what price did Diba purchase the
wheat from the wholesaler?
Ans with explanations:
"Faulty spring balance reads 0.9 pounds for a pound" means that she sells 1
pound for the price of 0.9 pounds, so she looses 10% of the price because of
the faulty spring balance;
She looses another 10% because of the discount.
So, she actually sells 1 pound for Tk 3*0.9*0.9=Tk 2.43 and since at that price
she made neither a profit nor a loss, then Katrina purchase the wheat from the
wholesaler for Tk 2.43. (Ans)
292.A, B and C can do a piece of work in 20, 30, 60 days
respectively. In how many days can A do the work if he is assisted
by B and C on every third day?
Ans with explanations:
A's 3 day's work = 1/20 * 3 = 3/20 part
(A + B + C)'s 3 day's work = 3/20 + 1/30 + 1/60 = 1/5 part
Now, 1/5 part work is done in 3 days.
Whole work will be done in (3 x 5) = 15 days. (Ans)
293.A room is 12 feet long, 8 feet wide and 10 feet high. Find the
cost of white washing the four walls inside the room, leaving out
two doors each measuring 7 feet high and 4 feet wide and four
windows each measuring 4 feet high and 3 feet wide if the rate is
Tk. 12 per square feet.
Ans with explanations:
Area of two walls in longside=12*10*2=240 sq. feet.
Area of two walls in wideside =8*10*2=160 sq. feet
Total Area of the room=240+160=400 sq. feet
Again, Area of two doors=7*4*2=56 sq feet
Area of four windows =4*3*4=48 sq. feet
Now, Area be white washing =400-(56+28)=296 sq. feet
Cost of white washing =296*12=3552 tk (ans)
294.A man is standing on a railway bridge which is 180 meters
long. He finds that a train crosses the bridge in 20 seconds and
crosses him in 8 seconds. Find the length of the train and its
speed.
Ans with explanations:
Let, Speed of train be x m/s
Length of train=8x m
Total distance =(180+8x)m
ATQ;
(180+8x)/x=20
=>20x=180+8x
=>12x=>180
=>x=180/12=15
So, speed 15m/s or (15*18/5)=54kmh & length of train=8*15=120m
(ans)
295.Price of sugar increases by 32%. A family reduces its
consumption so that the expenditure of the sugar is up by 10%
only. If the total consumption of sugar before the price rise was
10kg per month, then the consumption of sugar per month at
present is?
Ans with explanations:
Let,
previous price=10 tk/perkg
previous use of sugar =10 kg
total price =100 tk
present price =10*132/100 =13.2 tk
present expenditure = 110 tk
so, family bought = 110*10/132 = 25/3 kg
296.A trader mixes 26 kg of rice at TK. 20 per kg with 30 kg of rice
of other variety at TK. 36 per kg and sells the mixture at TK. 30 per
kg. His profit percent is?
Ans with explanations:
Total rice = 26+30 = 56 kg
Total cost price = 26*20+ 30*36 = 520+1080=1600 tk
Total sp = 56*30 = 1680
Profit = 1680-1600 =80
Profit % = 80*100/1600 = 5% (ans)
297.A garrison of 500 men had provisions for 27 days. After 3 days
a reinforcement of 300 men arrived. For how many more days will
the remaining food last now?
Ans with explanations:
Remaining day = 27-3 = 24 days
Now, number of men = ( 500+300) = 800
500 m had food for 24 days
1..........24*500 days
800.....= 24*500/800d = 15 days. (ans)
298.Among 50 junior officers of a bank, 22 have MBIM degree, 15
have banking diploma and 14 are M.Com in banking. Nine of the
employees have two of the degrees and one has taken all the three
of the degrees. How many of 60 officers have none of the degrees?
Ans with explanations:
Given that,
MBIM Degree : 22
Banking Diploma: 15
M.Com: 14
Which would add up to 51 "different" attendands, which is not possible.
Now 9 have taken exactly 2 courses, which means that there are 9 less
"different" attendants = 51-9= 42
1 Person has taken all three courses. As above, we subtract him from the
number of "different" attendant . Since this time the person took all three
courses, we have to substract him two times.
42-2= 40.
So, 50-40=10 employees who didn't take any courses (ans)
299.A tank is fitted with 8 pipes, some of them that fill the tank and
others that are waste pipe meant to empty the tank. Each of the
pipes that fill the tank can fill it in 8 hours, while each of those that
empty the tank can empty it in 6 hour. If all the pipes kept open
when the tank is full, it will take exactly 6 hours the tank to empty,
how many of these are fill pipes?
Ans with explanations:
Let, filled pipe = x
empty pipe = 8-x
so,
1 pipe fill in 1 h 1/8 p
x.................1h..x/8 part.
1pipe empty in 1h 1/6 parts
8-x............. 1 h..(8-x)/6 p
ATQ;
(8-x)/6 - x/8 =1/6
=>x = 4
So, filled pipe =4 (ans)
Alternate solutions:
Let, the no. of fill pipes be n
Therefore, there will be (8−n) no of waste pipes.
Each of the fill pipes can fill the tank in 8 hours. Therefore, each of the fill
pipes will fill (1/8)parts of the tank in an hour.
Hence, n fill pipes will fill (n/8)th of the tank in an hour.
Similarly, each of the waste pipes will drain the full tank in 6 hours. That is,
each of the waste pipes will drain (1/6)th of the tank in an hr
Therefore, (8−n) waste pipes will drain {(8−n)/6}th of the tank in an hr
Between the fill pipes and the waste pipes, they drain the tank in 6 hours. That
is, when all 8 of them are opened, (1/6)th of the tank gets drained in an hour.
ATQ;
=>n/8−(8−n)/6=−1/6
=>n = 4
So, the no. of fill pipes is 4 (ans)
300.At 1 PM, ship A leaves port heading due west at 20 miles per
hour. Two hours later, ship B is 100 miles due south of the same
port and heading due north at 25 miles per hour. At 5 PM, how far
apart are the ships?
Ans with explanations:
According to figure,
A goes 80 miles at 20 mph in 4 hrs (1:00-5:00=4)
& B goes 50 miles at 25 mph in 2 hrs(3:00-5:00=2)
Let,hypotenuse be x & height and base are 80 miles & 50 miles respectively.
So,according to phythagorian law,
hypotenuse^2 = height^2 + base^2
=>x^2 = (80)^2 - (50)^2
=>x = 94.34
So,the ships are 94.34 miles apart (ans)
301.A red light flashes 3 times per minute and a green light flashes 5 times
in two minutes at regular intervals. It both lights start flashing at the same
time, how many times do they flash together in each hour?
Ans with explanations:
Red light flashes 3 times/minute
that means 3times in 60 seconds
So, 1time in 20 seconds
Similarly, green light flashes 1 time in 24 seconds
LCM of 20 and 24=120
so in every two minutes they flash together for 1 time
So in 1 hour/ 60 mins they will flash 60/2=30 times together. (ans)
.........#RezaRahman
302.How many 3 digit numbers can be formed from the digits 2, 3, 5, 6, 7 &
9 which are divisible by 5 and none of the digits is repeated?
Ans with explanations:
Since each desired number is divisible by 5,
So, we must have 5 at the unit place.So, there is 1 way of doing it.
The tenth place can now be filled by any of the remaining 5 digits (2, 3, 6, 7, 9). So,
there are 5 ways of filling the tenth place.
The hundredth place can now be filled by any of the remaining 4 digits. So, there are
4 ways of filling it.
Required number of numbers = (1 x 5 x 4) = 20 (ans)
303.A jar contains 4 red tooth picks, 10 blue toothpicks and 6 yellow
toothpicks. It three toothpicks are removed from the bag at random and no
toothpick is returned to the bag after removal, what is the probability that
all three toothpicks will be blue?
Ans with explanations:
Probability that all three toothpicks will be blue
= 10/20 * 9/19 * 8/18 = 2/19 (ans)
304.In a city, 40% of the people are illiterate and 60% are poor. Among the
rich, 10% are illiterate. what percentage of the poor population is
illiterate?
Ans with explanations:
Let,
Total people=100
Illiterate=40
Literate=60
Poor =60
Rich =40
Rich & illiterate=40(10/100)=4
Poor &illiterate =40-4=36
Percentage poor & illiterate=36*100/60=60%( ans)
........Rubel Hasan
305.A and B ran, at their respective constant rates, a race of 480 m. In the
first heat, A gives B a head start of 48 m and beats him by 1/10th of a
minute. In the second heat, A gives B a head start of 144 m and is beaten by
1/30th of a minute. What is B’s speed in m/s?
Ans with explanations:
Let,
velocity of A=x m/s
& velocity of B= y m/s
First case:1
A goes 480 m & B goes 480-48=432 m
ATQ;
432/y -480/x =60/10=6 __(1)
Second case:
A goes 480 m & B goes 480-144=336 m
ATQ;
480/x -336/y =60/30=2 ___(2)
Applying (1)+ (2)
432/y -480/x =6
480/x -336/ y =2
_________________
432/y -336/y=8
=> y =12 m/s
So, B's speed is 12 mps (ans)
.............Sumon Sahidul
306.A draining pipe can empty a pool in 4 hour. On a rainy day, when the
pool is full, the draining pipe is opened and the pool is emptied in 6 hour. If
rain inflow into the pool is 3 liters per hour, what is the capacity of the
pool?
Ans with explanations:
Let,
Raining water can fill the pool in x hrs
ATQ,
1/4 -1/x =1/6
=> x=12 hrs
Now,
In 1 hr fill ____3 L
...12 hr _____36 L ( ans)
........#SumonSahidul
307.Usually Holly leaves home to school at 9:00, however today she left
home 20 minutes later. In order to be at school on time she increased her
usual speed by 20% and still was at school 15 minutes later than usual.
What is her usual time from home to school?
Ans with explanations:
Let,
Usual time = t hrs
at 20% increase=120/100=6/5
Let,
Previous speed = 5x km/hr
Present speed =6x km/hr
As distance is equal for both the case.
ATQ,
5x*(t+ 20/60)=6x*(t+ 15/60)
=> t=1/6 hr =10 min( ans)
......#SumonSahidul
308.A contractor estimated that his 10-man crew could complete the
construction in 110 days if there was no rain. (Assume the crew does not
work on any rainy day and rain is the only factor that can deter the crew
from working). However, on the 61-st day, after 5 days of rain, he hired 6
more people and finished the project early. If the job was done in 100 days,
how many days after day 60 had rain?
Ans with explanations:
Total men needed = 10 X 110 = 1100
Men worked before 61st day = (60-5=)55*10 = 550
Men worked after 61st day = (10+6=)16*(100-60-x=40-x),
where x is the number of days it rained after day 60.
ATQ;
1100 = 550 + 16*(40-x)
=>16x = 90
=>x = 5.6
So, number of days it rained after day 60 is 5 days (ans)
309.Of the applicants passes a certain test, 15 applied to both college X and
Y. If 20 % of the applicants who applied college X and 25% of the
applicants who applied college Y applied both college X and Y, how many
applicants applied only college X or college Y?
Ans with explanations:
Let,
A = Applicants applied to only College X & B = Applicants applied to only College
Y.
Then, 0.20(A+15) = 15. (20% of applicant applied in college X)
& 0.25(B+15) = 15. (25% of applicant applied in college Y)
By solving this, we get
A = 60 and B = 45
So X + Y = 105 applicants (ans)
Alternate Solution:
Given that,
20% of X = 15
Thus, X = 75
So, the applicants who applied to only college X = 75-15=60
Again, 25% of Y = 15
Thus, Y = 60
So, the applicants who applied to only college Y = 60-15=45
Thus, the applicants who applied to only college X or college Y
= 60+45=105 (ans)
310.This year Henry will save a certain amount of his income, and he will
spend the rest. Next year Henry will have no income, but for each dollar
that he saves this year, he will have 1 + r dollars available to spend. In
terms of r, what fraction of his income should Henry save this year so that
next year the amount he was available to spend will be equal to half the
amount that he spends this year?
Ans with explanations:
Let's say that his income is I. So he saves S and spends E.
I = E + S..........(1)
For each dollar he saves today he'll have 1+r dollars to spend next year, so if he saves
S, he'll have S(1+r) dollars to spend next year.
ATQ;
S(1+r)=E/2
=>S = E /2(1+r)
So, S + E = E/2(1+r) + E = E*(3+2r)/(2+2r)
Thus,Savings as a fraction of his income is:
S/I = S/(S + E) = E/(2+2r) / E*(3 + 2r)/(2 + 2r) = 1 / (3 + 2r) (ans)
Alternate solution:
Saving this year = S
Spending this year = E
Next year spend = S (1+r)
Given, S(1+r) = E/2
=>2 + 2r = E/S
=>2 + 2r + 1 = E/S + 1
=>3+2r = (E+S)/S
=>S/(E+S) = 1/(3+2r) (ans)
311.In a club the ratio of men and women is 16:7. The ratio of married men
and married women is 4:3. Calculate the percentage of married men if 60%
of the women are married.
Ans with explanations:
Let,
m = number of men
w = number of women
M = number of married men
W = number of married women
Given conditions:
m:w = 16:7 =>m/w = 16/7 =>m = 16w/7........1
M:W = 4:3 =>M/W = 4/3 =>M = 4W/3............2
W:w = 60:100 = 3:5 = 3/5
Applying 2/1
M/m = (4W/3)/(16w/7)=(7/12)(W/w)=(7/12)(3/5)=7/20=35% (ans)
312.A train travels from New York to Chicago, a distance of approximately
840 miles, at an average rate of 60 miles per hour and arrives in Chicago at
6:00 in the evening Chicago time . At what hour in the morning ,New York
city time ,did the train depart for Chicago?( note- Chicago time is one hour
earlier than New York city time)
Ans with explanations:
Given that,
Distance=840 km & Speed=60 km/hr
So,Time taken = 840/60 hours = 14 hours.
ATQ;
6:00 in evening in Chicago = 7:00 in evening in New York. So, the train was in
Chicago 7:00 in the evening, New York time.
From NYC's point of view, the train reached at 7:00 in the evening so it must have
started at 5:00 in the morning. (ans)
313.Two varieties of sugar are mixed in a certain ratio. The cost of the
mixture per kg is tk 0.05 less than that of the superior variety and tk 0.75
more than that of the inferior variety. What was the ratio of superior
variety to inferior variety in the mixture?
Ans with explanations:
Let,
Cost price of mixture = 1 tk
Superior Cost price = 1+0.05=1.05 tk
Inferior Cost price =1-0.75=0.25 tk
Let,
Mix the ratio in x:y
ATQ,
X*1.05+Y*0.25=1(x+y)
=> x:y=15:1( ans)
.........#SumonSahidul
314.A 4 liter oil can costs tk. 200. If the price of oil increases by a half
percent, how much will two 4 liters cans cost?
Ans with explanations:
Given that,
A 4 litre oil can cost 200 tk
At (1/2)% increase the present cost of a 4 litre can
= 200+200*(1/2)%=201 tk
Now, two 4 litre can cost = 201*2 = 402 tk (ans)
..........Rubel Hasan
315.A car uses one liter of petrol to travel 17 miles. After a tune-up, the car
travels 17% farther on one. How many liters of petrol (to the nearest tenth)
will it take for the car to travel 170 miles after a time?
Ans with explanations:
17 miles goes by 1 litre
At 17% tune up, the distance will =17 + 17*17/100 = 19.89 miles
Now, 19.89 miles goes by 1 litre
So, 170 miles goes by 170/19.89 = 8.55 litres (ans)
........Sumon Sahidul
316.The mile meter of a car misses every eleventh mile being traveled. After
a certain time, the meter shows that 1251 miles were travelled. How many
miles were actually traveled?
Ans with explanations:
The mile meter of a car misses every eleventh mile being traveled.
So, When the meter shows 11 miles it's actual distance
= 11+1= 12 miles
When it shows 1251 miles then it's actual distance is
= (1251*12)/11 = 1364.72 miles (Ans)
..........Giko Giko
317.X sells a product to Y and makes a profit of 25%. Y sells it to Z at a
loss of 20%. If Z buys it for tk. 140, what price X paid for it?
Ans with explanations:
According to question X sells the product to Y in 100+25=125 tk
& Y sells the product to Z in 100-20=80 tk
So,In case of Z,
His Cost price is 80 tk when it's selling price is 100 tk
Hence,................ 140 tk when it's selling price is 100*140/80=175 tk,
which is cost price for Y.
In case of Y,
His cost price is 125 tk when selling price is 100 tk
Thus,................ 175 tk when selling price is 100*175/125=140 tk,
which is cost price for X (ans)
318.A woman buys a pound of steak for $3.00. If the meat loses one-fourth
of its weight when cooked, what is the cost per pound when it is served at
the table?
Ans with explanations:
Amount of meat after loosing it's weight (1 - 1/4) = 3/4 pound
ATQ;
The cost of 3/4 pound of meat is $3.00
So,............. 1 pound of meat is $4*3/3 = $4.00 (ans)
.........#RubelHasan
319.A shopkeeper makes a loss of 12% by selling an item at 80% of its
marked price. What will be the percent profit made by the shopkeeper if he
sells the item at 95% of its marked price?
Ans with explanations:
At 12% loss, the selling price = 100-12 = 88 tk
ATQ;
While selling price is 80 tk, it's market price is 100 tk
..................................88 tk,................................. 88*100/80 = 110 tk
Again,
While market price is 100 tk, it's selling price 95 tk
................................. 110 tk,..........................110*95/100 = 104.5 tk
So, profit % = 104.5 - 100 = 4.5% (ans)
320.In a 100 m race, Jami beats Rony by 4 sec .If Jami allowed Rony to
start 16m ahead of him, and then Jami and Rony reach the finishing point
at the same time. How long does Jami take to run the 100 in race?
Ans with explanations:
Jami takes 4 sec less than Rony.
So, Rony goes 16m in 4 sec
Rony goes 100m in (4*100)/16 = 25 sec.
So, Jami takes (25-4) = 21 sec (Ans)
.........#GikoGiko
321.An article is sold at 20% profit. If its cost price is increased by tk. 50
and at the same time if its selling price is also increased by tk. 30, the
percentage of profit decreases by 10/3. Find the cost price.
Ans with explanations:
Let, the cost price be x tk
At 20% profit, the selling price = x + x*20/100 = 6x/5 tk
At 50 tk increased, new cost price = (x+50) tk
At 30 tk increased, new selling price = (6x/5 + 30) tk
ATQ;
(6x/5 + 30) - (x+50) = (20 - 10/3)% of (x+50) =(50/300=)1/6 of (x+50)
=>x = 850
So, the cost price is 850 tk (ans)
322.In a factory there are three types of Machines M1, M2 and M3 which
produces 25%, 35% and 40% of the total products respectively. M1, M2
and M3 produce 2%, 4% and 5% defective products, respectively. What is
the percentage of non-defective products?
Ans with explanations:
Total production = 100
M1 produces = 25
M2 produces = 35
M3 produces = 40
So, M1 produces non defective products = 25*98/100 = 24.5
M2 produces........................................... = 35*96/100 = 33.6
M3 produces............................................= 40*95/100 = 38
Hence, total percentage of non defective products
= (24.5+33.6+38) = 96.10% (ans)
323.In a class of 150 students, 90 students have taken F, 75 students have
taken B, and 55 students have taken E. 10 students have taken F and E but
not B, 15 students have taken F and B but not E, and 25 students have
taken B and E but not F. Find the number of students who have taken all
the three courses.
Ans with explanations:
Let,
As we have,
Total = G1 + G2 + G3 - (Exactly 2 groups) - 2*(3 groups) + none
=>150 = 90 + 75 + 55 - (10 + 15 + 25) - 2*(3 groups) + 0
=>3 groups = 10
So, the no. of students are 10 who have taken all the 3 courses (ans)
..........Sarker Mamun
324.A, B and C invite 17 men for drinking tea .three partner are bear
the cost equally. A gives 7 person’s tea bill and B also gives 10
person’s bill and C gives 34 tk as a his cost .now give this 34 tk
between A and B?
Ans with explanations:
Total Cost = (34*3)= Tk 102
per man cost= 102/17 = Tk 6
A gives = 7*6 = Tk 42
B gives = 10*6 = Tk 60
Now, A gets from C = 42-34 = Tk 8
And, B gets from C =60-34 = Tk 26
A= Tk 8 & B = Tk 26 (Ans)
............Giko Giko
325.An amount of loan allow for 4 years .4% and 5% interest rate for
ist 2 years and next two years respectively .after 4 years capital
plus interest would become 1416. Find initial capital.
Ans with explanations:
Let, Initial capital be x tk
ATQ;
x + x*2*4/100 + x*2*5/100 = 1416
=>x = 1200
So, Initial capital is 1200 tk (ans)
326.The Cost price of 20 articles is the same as the selling price of
X articles. It the profit is 25% then the value of X is:
Ans with explanations:
Let, Cost price of 20 articles be x tk
& Selling price of 20 articles = 20 tk
Profit % = {(20 - x)*100/x }tk
ATQ;
(20-x)*100/x = 75
=>x = 16 (ans)
327.What is the minimum number of tiles of size 16 by 24 required
to form a square by placing the tiles adjacent to one another?
Ans with explanations:
LCM of 16 & 24 is 48
Since,the tiles form a square
A side of the square must be a multiple of both 16 & 24,
So 48/16 = 3 means there are 3 column of length 16
& 48/24 = 2 means there are 2 rows of length 24.
Hence, the total no of tiles required 3*2 = 6 (ans)
328.Three workers, A, B, and C, can complete a certain task in 10, 5
and x hours respectively. A starts working alone and 2 hours later
B joins. After another 2 hours joins C. After that A, B, and C
together complete the task in 15 minutes. What is the value of x?
Ans with explanations:
A, B & C's 1 hour work = (1/10 +1/5 + 1/x) parts
As A works for 2 hrs alone,it completes 2*1/10 = 1/5 parts
After 1st 2 hrs A & B together work for another 2 hrs.So,
They complete 2*(1/10 + 1/5) = 3/5 parts
Thus, remaining work after 4 hrs = (1 - 1/5 - 3/5) = 1/5 parts
Given, A, B & C together complete 1/5 parts in 15 mins
So,...............................1 part in 5*15 = 75 mins = 75/60 hrs = 5/4 hrs
ATQ;
1/10 + 1/5 + 1/x = 4/5
=>x = 2
So, the value of x is 2 (ans)
329.Machine A can process 6000 envelopes in 3 hour. Machines B
and C working together but independently can process the same
number of envelopes in 2.5 hour. If Machines A and C working
together but independently process 3000 envelopes in 1 hour, then
how many hours would it take Machine B to process 12000
envelopes.
Ans with explanations:
Given that, Machine A can process in 3 hrs 6000 envelopes
So,Machine A's rate = 6000/3 =2000 envelopes per hour
Machine (B+C) rate =2400 envelopes per hour
Machine (A+C)rate =3000 envelopes per hour
Then
C 's rate=3000-2000=1000 envelopes per hour
B 's rate=(2400-1000)=1400 envelopes per hour
b rate =12000/1400=60/7 = 8(4/7) hours. (ans)
330.It takes 6 technicians a total of 10 hours to build and program a
new server from Direct Computer, with each working at the same
rate. If six technicians start to build the server at 11:00 AM, and one
technician per hour is added beginning at 5:00 PM, at what time will
the server be complete?
Ans with explanations:
in 10 hours 6 technicians can do 1 part of the job
So,11AM to 5PM = 6 hours 6 technicians can do (6/10)= 3/5 parts
At 6PM, total 7 technicians work for 1 hour
So, the job is done in 1 hr = 7/(6*10) = 7/60 parts
so, total job done at 6.00PM is = 6/10+7/60 = 43/60 parts
At 7PM, total 8 technicians work for 1 hour
So, the job is done = 8/(6*10) = 8/60 = 2/15 parts
total job done at 7.00PM is = 43/60 + 2/15 = 51/60 parts
At 8PM, total 9 technicians work for 1 hour
So, the job is done = 9/(6*10) = 9/60 = 3/20 parts
Total job done at 8.00PM is = 51/60 + 9/60 = 60/60=1 part
So, Server will be complete at 8PM (ans)
.........Asma Binte Salahuddin
331.A, B and C enter into partnership. A invests some money at the
beginning, B invests double the amount after 6 months and C
invests thrice the amount after 8 months. If the annual profit be Tk.
18000, c’s share is?
Ans with explanations:
Let, A invest=12x tk
B=2x*6=12x tk
C=3x*4=12x tk
ATQ;
12x+12x+12x=18000
=>36x=18000
=>x=500
So, A's invest=12*500=6000 tk (ans)
...............Joy Shaha
332.A rectangular carpet covers half of a rectangular floor that is 9
feet wide and 12 feet long. If the dimensions of the carpet are in the
same ratio as those of the floor, then what is the length of the
carpet?
Ans with explanations:
Area of floor=9*12=108 sqft
Area of carpet= 108*(1/2)=54 sqft
Ratio of width & length= 9:12=3:4.
Let, width=3x & length=4x.
So, 4x*3x=54
=> x=3/√2.
Length= 4*(3/√2)=6√2 ft [answer]
...........Joy Shaha
333.If x is not a negative number, what is the maximum possible
value of 22- 3x?
Ans with explanations:
if x can not be a negative number. So x=0,1,2.3...
f(0)=2-3.0=2
f(1)=2-3.1=-1
f(2)=2-3.2=-4
So f(x) is maximum at x=0
334.The longest side of an isosceles right triangle measures 16
meter. What is the perimeter of the triangle?
Ans with explanations:
Give that,
The longest side of an isosceles right triangle 16 meter
According to pythagorian law,
a^2 + a^2 = 16^2
=>a = 8sqrt2
So, perimeter of triangle = 8sqrt2 + 8sqrt2 + 16 = 16(1+sqrt2) (ans)
335.In a class of 40 students, the number of students who passed
the math exam is equal to half the number of students who passed
the science exam. Each students in the class passed at least one of
the two exams. 5 students passed both exams, then how many
students passed the math exam?
Ans with explanations:
Suppose math=x
Science=2x
ATQ;
40=2x+x-5
=>45=3x
=>x=15
So, 15 students are passed in maths (ans)
........Joy Shaha
336.In how many ways can Annie, Bushra, Camellia, Don, Elina and
Farzana be seated if Annie and Bushra cannot be seated next to
each other?
Ans with explanations:
Total person =6
they can be seated =6! =720 way
Let Annei and Bushra seated next to each other,so they can be considered as
one person
Here total person =5
So, they can be seated=5!=120
Annie & Bushra can be seated each other =2!=2 way
So number of ways in which Annie Bushra next to each other=2*120=240
Therefore, Not next to each other=720-240=480 (ans)
.........Joy Shaha
337.The diagonal of a square is 4√2 cm. The diagonal of another
square whose area is double that of the first square, is?
Ans with explanations:
Here diagonal=4√2 cm
So, One side, a =4 cm [as a*sqrt2 = 4*sqrt2]
Area=4²=16 sqcm
Double area=16*2=32 sqcm
One side=√32=4√2 cm
Now, Diagonal=4√2*√2=8 cm (ans)
......Sohel Rana
338.The ratio of the number of boys and girls in a school is 3:2.If
20% of the boys and 25% of the girls are scholarship holders, what
% of the students does not get the scholarship?
Ans with explanations:
Suppose,Total no. of Boys & Girls are 300 & 200 respectively (according to
given ratio)
No. of Boys who are scholarship holders=300*20%=60
No. of Girls who are scholarship holders=200*25%=50
Total scholarship holders=(60+50)=110
So,no of students who doesn't get scholarship = 500-110 = 390
Required %=390/500*100=78% (ans)
339.A dessert recipe calls for 50% melted chocolate and 50%
raspberry puree to make a particular sauce. A chef accidentally
makes 15 cups of the sauce with 40% melted chocolate and 60%
raspberry puree instead. How many cups of the sauce does he
need to remove and replace with pure melted chocolate to make the
sauce the proper 50% of each?
Ans with explanations:
Melted chocolate =15*40%=6 cups
Raspberry =15*60%=9
We need 1:1
Raspberry should =15/2 =7.5 cups
Raspberry need to remove=1.5 cups
60%=1.5 cups
100%=1.5*100/60=2.5 cups
So 2.5 cups sauce need to remove and replace with pure melted chocolate to
make 50% each (ans)
......Sohel Rana
340.Fanny and Alexander are 360 miles apart and are traveling in a
straight line toward each other at a constant rate of 25 mph and 65
mph respectively, how far apart will they be exactly 1.5 hours
before they meet?
Ans with explanations:
As Fanny and Alexander's combined rate is (25+65) mph
Then, 1.5 hours before they meet
They'll be (25+65)*1.5=135 miles apart. (ans)
341.A drink contains 20% cranberry juice, 20% raspberry juice and
the rest is apple juice. You needed 250 ml water in 750 ml of the
drink. Now what is the ratio of water to apple juice in the drink?
Ans with explanations:
apple juice = 100-20-20 =60%
apple juice in mixture = 750*60%= 450ml
ratio of water to apple juice = 250 : 450 = 5 : 9(ans)
342.Four friends R,G,S,N purchased a toy train .R contribute half
that of total contribution made by other three and G is one fourth of
other three and S is two third of the other threes. If total price of the
toy train is tk.9900. Find contribution made by R?
Ans with explanations:
ATQ;
R = (G+S+N)/2.....(i)
and, R+G+S+N=9900
=> (G+S+N)/2 = (9900-R)/2......(ii)
From (i),
R = (9900-R)/2
=> 3R = 9900
=> R = 3300 (Ans.)
343.A person travels from x to y at a speed of 40km/p and returns
by increasing his speed by 50%. What is his average speed for both
trips?
Ans with explanations:
going speed=40kmh
returns speed=40*150/100
=60kmh
Avg. speed=2*40*60/(40+60)=48kmh
344.50 men took a dip in a water tank 40 m long and 20 m broad on
a religious day. If the average displacement of water by a man is 4
m3, then the rise in the water level in the tank will be?
Ans with explanations:
Displacement of water by 1 man = 4 m3.
Total volume of water displaced by 50 men = (4 x 50) m3 = 200 m3.
Rise in water level in the tank is same as the height of the volume of cuboid.
Volume = Length x Breadth x Height.
Rise in water level = 200/(40 x 20) = 0.25 m or 25 cm.(ans)
345.A square office, 1000 feet by 1000 feet, is to be partitioned into
two offices by a single interior wall. The difference between the
perimeters of the resulting two officers is 400 feet. What are their
dimensions?
Ans with explanations:
Let, width of first portion after partition = x ft
So, width of second portion other side = (1000 - x) ft
According to the question,
2(1000 + x) - 2(1000 + 1000 - x) = 400
Or, 2000 + 2x – 4000 + 2x = 400
Or, 4x = 2000 + 400
Or, x = 600
So, width of first portion = 600 ft
Width of other side = 1000 – 600 = 400 ft
So, dimensions of 1st office = 1000 × 600 = 600000 sq. ft.
And, dimensions of 2nd office = 1000 × 400 = 400000 sq. ft. (Ans)
346.A rectangular park 60 m long and 40 m wide has two concrete
crossroads running in the middle of the park and rest of the park
has been used as a lawn. If the area of the lawn is 2109 sq. m, then
what is the width of the road?
Ans with explanations:
Area of the park = (60 x 40) m2 = 2400 sqm
Area of the lawn = 2109 sqm
Area of the crossroads = (2400 - 2109) m2 = 291 sqm
Let, the width of the road be x metres.
Then,ATQ;
60x + 40x - x2 = 291
=>x2 - 100x + 291 = 0
=>(x - 97)(x - 3) = 0
=>x = 3.
So, the width of the road 3 meter (Ans)
347.A rectangular field is to be fenced on three sides leaving a side
of 20 feet uncovered. If the area of the field is 680 sq. feet, how
many feet of fencing will be required?
Ans with explanations:
another side= x feet
we have, 20*x=680
=> x=34
as we have left 20 feet side.
other three side are to fancing, ic (34+34+20)=88 feet. (Ans)
348.A man traveled one-fourth of the total distance of his trip by
car. He traveled the remaining distance on foot. The ratio of his
walking time to driving time was 15:1. Calculate the ratio of his
driving speed to his walking speed.
Ans with explanations:
let, total distance x
time required by car & foot t & 15t respectively.
speed of car=(x/4)/t=
x/4t
speed in foot=(3x/4)/15t=
x/20t
ratio of speed=(x/4t):(x/20t)=5:1 (Ans)
349. For an item the profit is 40% when the discount is 30%. What is
the profit when the discount is 40%?
Ans with explanations:
Let, market price =100
selling price =100-30=70
cost price =70*100/140=50
when discount 40%, selling price=100-40=60
then profit=60-50=10
profit%=10*100/50=20% (ans)
350.From 7:00 AM to 11:00 AM it rained 2.25 inches. At 11:00 AM
the rain increased to fall at a rate of 1.25 inches every two hour.
How many inches of rain landed of the ground by 5:00 PM?
Ans with explanations:
from 11AM to 5PM, total 6hour
so rain increased = 6*1.25/2 [given, 2hour rate]
= 3.75 inches
rain landed of the ground by 5:00 PM = 2.25 + 3.75 inches
= 6 inches (ans)
351.Of the 50 students in a college, 32 are enrolled in Psychology
class and 37 are enrolled in Logic. If 6 of the students did not enroll
in either class, how many of the students are in only one of these
two classes?
Ans with explanations:
total=psychology + logic - both +none
=>50=32+37- both+ 6
=> both=15 (ans)
352.A man walking at 3 km/hr crosses a square field diagonally in 2
minuties. Area of the field is?
Ans with explanations:
Speed = 3 kmph = (3 * 5)/18 = 0.83 m/sec
Time = 2 minutes = 120 sec
Distance covered in 120 seconds = 120 * 0.83 = 100 m.
Length of the diagonal = 100
As,we know, Sqrt 2*side = 100
=>Side = 70.92 m
So,Area = Side * side = 70.92 * 70.92 = 5000 m^2 (ans)
353.A man can hit a target once in 4 shots. If he fires 4 shots in
succession what is the probability that he will hit his target?
Ans with explanations:
Probability of hitting the target in one shot out of four shots= 1/4
Probability of missing the target in one shot
= 1 - (1/4)
= 3/4
So the probability of missing in all four shots = (3/4)*(3/4)*(3/4)*(3/4)
= 3*3*3*3/4*4*4*4
= 81/256
Therefore, probability of hitting at least one target
= 1- (81/256)
= 175/256. (ans)
354.A container of 64 ltr was full of milk. 16 ltr milk was drawn off
and replace by water and this process is repeated 2 times more find
the quantity of water in the mixture now?
Ans with explanations:
Suppose, a container contains x units of a liquid from which y units are taken
out & replaced by water.After n operations, quantity of pure liquid = x(1 -
y/x)^n units
So,Milk contained by the container now = 64(1 - 16/64)^3 = 27 litres (ans)
355.A watch which gains uniformly is 2 minutes low at noon on
Monday and is 4 min. 48 sec fast at 2 p.m. on the following Monday.
When was it correct?
Ans with explanations:
Time from 12 pm to next Monday 2 pm
=7 days 2 h(12 pm -2 pm)=170 hours
Total gain=2+4(48/60)=2+4(4/5)=2+24/5=34/5 min.
34/5 min gain from 170 h.
So,2 min gain =(170*5*2)/34=50 h
That's mean it was 2 days 2 h after Monday at 12 pm.
So, correct time is 2 p.m. Wednesday.(ans)
356.How many different words can be formed taking three at a time
from the letters of the ‘AMERICA’?
Ans with explanations:
There are 7 words but 2 words (A) same so counting words 6.
Case1:3 from6=6!/3!=6*5*4*3!/3!=120
Case2:When 2 words same.
Another word from remaining 5 choice.
So,,5*3!/2!=5*3=15
Total=120+15 = 135 (ans)
357.If 3 men or 5 boys can do a work in 30 days, then 5 men and 4
boys together can do it in how many days?
Ans with explanations:
3 men=5 boys
1men=5/3 boys
ATQ,
5 men + 4 boys=(5*5/3+4)=37/3 boys
5 boys can do in 30 days
37/3 can do=30*5*3/37 =450/37=12.13 days (ans)
358.During Air-raid, sirens at four different places of a town are
rung at interval of 1, 1*1/4, 1*1/2, 1*3/4 minutes respectively. If they
ring together at the starting, when will they ring together again?
Ans with explanations:
Given that,
1, 1*1/4, 1*1/2, 1*3/4 minutes
Turn all the values into seconds we got 60,75,90,105 seconds
LCM of 60,75,90,105=6300 secs or 105 minutes or 1 hour 45 mins (Ans)
359.For making a cupboard it requires human labor three times the
labor required to make a bench. Six carpenters can make 36
benches and 5 cupboards in 12 days. How many days will 10
carpenters require to make 61 benches and 8 cupboards?
Ans with explanations:
ATQ;
1 cupboard = 3 benches
5 cupboards = 5*3 = 15 benches
8 cupboards = 8*3 = 24 benches
So, 36 benches & 5 cupboards = 36+15 = 51 benches
& 61 benches and 8 cupboards = 61+24 = 85 benches
So, 6 carpenters can make 51 benches in 12 days
Now,10 carpenters can make 85 benches 12*6*85/51*10 = 12 days (ans)
360.A Company employs 15 persons, each working 44 hours a
week. If 4 persons are absent, how many hours a week would the
rest of the persons have to work to make up the time lost?
Ans with explanations:
15 persons need to complete a work in 44 hours
So, (15-4)=11 persons need to complete a work in 15*44/11=60 hrs(ans)
361.A batsman makes a score of 87 runs in the 17th innings and
thus increase his average by 3.Find his average after 17th innings?
Ans with explanations:
Let, the average after 17th innings be x
16(x-3)+87 = 17x
or, x = 87 - 48 = 39 (ans)
362.Suppose you deposited Taka 10,000 on January 01, 2013 at
12.50% interest rate-for 1 year, on July 01, 2013 Taka 15000 at 12%
interest rate for 6 months and on October 01, 2013 Taka 20000 at
22.50% interest rate for 3 months (assume that the stated interest
rates are simple and annual). Suppose you withdrew all deposits
(include due interests) on December 31, 2013. Calculate the overall
annual rate of interest you have received.
Ans with explanations:
.‪#Investment_01:
•• Tk. 10000 from 01.01.2013 to 31.12.2013 @12.50%.
Total Interest Earned = Tk. 10000 × 12.50% × 12/12 = Tk. 1250.
Weighted average annual investment = Tk. 10000 × 12/12 = Tk. 10000.
‪#Investment_02:
•• Tk. 15000 from 01.07.2013 to 31.12.2013 @12%.
Total Interest Earned = Tk. 15000 × 12% × 6/12 = Tk. 900.
Weighted average annual investment = Tk. 15000 × 6/12 = Tk. 7500.
‪#Investment_03:
•• Tk. 20000 from 01.10.2013 to 31.12.2013 @11.50%.
Total Interest Earned = Tk. 20000 × 11.50% × 3/12 = Tk. 575.
Weighted average annual investment = Tk. 20000 × 3/12 = Tk. 5000.
‪#Now:
◘ Total Interest Earned = Tk. 1250 + 900 + 575 = Tk. 2725.
◘ Weighted Average Annual Investment To Earn The Interest = Tk. 10000 +
7500 + 5000 = Tk. 22500.
‪#Therefore:
Overall Annual Rate of Interest = 2725/22500 = 0.121111.... = 12.11%(ans)‪

.........Joy Shaha

363.An automobile financier claims to be lending money at simple


interest, but he includes the interest every six months for
calculating the principal. If he is charging an interest of 10%, the
effective rate of interest becomes?
Ans with explanations:
Let,the sum be 100 tk. Then,
S.I. for 1st 6 months = 100*10*1/100*2 = 5 tk
S.I. for last 6 months = 105*10*1/100*2 = 5.25 tk
So, total amount at the end of 1 year = (100 + 5 + 5.25)=110.25 tk
Now, effective rate = 110.25 - 100 = 10.25% (ans)
364.Three workers can do a job in 12 days. Two of the workers
work twice as fast as the third. How long would it take one of the
faster workers to do the job alone?
Ans with explanations:
Let,the 3rd worker complete the whole job in 2x days,then the other two's
complete the same in x days.
ATQ;
1/x+1/x/+1/2x=1/12
=>5/2x=1/12
=>x=30
So,It takes 30 days for one of the faster worker(ans)
365.A tank can be filled by a tap in 20 minutes and by another tap in
60 minutes. Both the taps are kept open for 10 minutes and then
the first tap is shut off. After this, the tank will be completely filled
in?
Ans with explanations:
Both the tanks are filled combinedly in 1 min (1/20+1/60)=1/15 parts of work.
............................................................10 mins (10/15) = 2/3 parts of work.
Remaining work = 1 - 2/3 = 1/3 parts
2nd tank completes 1/60 parts of work in 1 day
So,...........................1/3 parts of work in 60/3 = 20 days(ans)
366.Two pipes A and B can fill a tank in 6 hours & 4 hrs
respectively. If they are opened on alternate hours and if pipe A is
opened first, in how many hours the tank shall be full?
Ans with explanations:
A's & B's 2 hrs work (1/6 + 1/4) = 5/12 parts
A's & B's another 2 hrs work = 2*5/12 = 5/6 parts
Remaining work = 1 - 5/6 = 1/6 parts
ATQ;
A do 1/6 parts of work in 1 hr
So,the work totally complete in 2 + 2 + 1 = 5 hours (ans)
367.One pipe can fill a pool 1.25 times faster than a second pipe.
When both pipes are opened, they fill the pool in five houTK. How
long would it take to fill the pool if only the slower pipe is used?
Ans with explanations:
1.25 = 125/100 = 5/4
Let,Slower pipe can fill the pool in 5x hours & faster pipe can fill the pool in
4x hrs
ATQ;
1/5x + 1/4x = 1/5
=>x = 9/4
So,slower pipe takes 9*5/4 = 11.25hrs (ans)
368.Dipu works alone, he will take 20 more hours to complete a
task than if he worked together with Enam. If Enam works alone, he
will take 5 more hours to complete the task than if he worked
together with Dipu. If they work together, how many hours do they
need to complete the task?
Ans with explanations:
If Dipu finishes a job in x hours, and Eman finishes a job in y hours, and
together they finish the job in t hours, then
1/x + 1/y = 1/t................1
Here, x = t + 20, and y = t + 5. Solve the above equation
1/(t+20) + 1/(t+5) = 1/t or,t=10(ans)
369.The sum of two numbers is 56.If three times of the smaller
number is one-half of the larger number, what is the larger
number?
Ans with explanations:
Let, Larger number x
& Smaller number (56-x)
ATQ;
3(56-x)=x/2
=>336-6x=x
=>7x=336
=>x=48
So,larger number 48 (ans)
370.Jamil bought 25 kg of rice at the rate of Tk 6 per kg and 35 kg of
rice at the rate of Tk 7 per kg. He mixed the two and sold the
mixture at the rate of Tk 6.75 per kg. What was the gain or loss in
the transaction?
Ans with explanations:
price of 25 kg rice=6*25=150 tk
price of 35 kg=7*35=245 tk
total price of 60 kg=150+245=395 tk
selling price =6.75*60=405 tk
hence profit=405-395=10 tk
gain % =10*100/395=2.53% (ans)
371.By selling 8 dozen of pencils, a shopkeeper gains the selling
price of 1 dozen pencils. Find out his gain percent.
Ans with explanations:
Gain = Selling price of 8 dozen - Cost price of 8 dozen
=>Selling price of 1 dozen = Selling price of 8 dozen - Cost price of 8 dozen
=>Cost price of 8 dozen = Selling price of 7 dozen
Let, Cost price of each dozen be 1 tk
Thus,selling price of 7 dozen 7 tk
& cost price of 8 dozen 8 tk
Now, gain % = (8 - 7)*100/7 = 14(2/7)% (ans)
372.The marked price of a radio is Tk 480. The shopkeeper allows a
discount of 10% and gains 8%. If no discount is allowed, what will
be his gain percentage?
Ans with explanations:
Marked price =480 tk
Selling price = 480*90/100=432 tk
Cost price =432*100/108=400 tk
When, discount not given, selling price =480 tk
profit=480-400=80 tk
gain%=80*100/400=20%(ans)
373.Son's age is now one-third of father's age. In twelve years from
now son's age will be one half of the father's age. What is the son's
age in years now?
Ans with explanations:
Let, the age of father & son be 3x & x yrs
ATQ;
x+12 = (3x+12)/2
So, x = 12 years (ans)
374.936 tk will distribute among 13 women ,15 men and 12 boys .if 1
woman’s amount equal 3 boys amount and 1 man’s amount equal 2
women’s and 1boy’s amount .how much get every man?
Ans with explanations:
Given that,
1 woman's amount =3 boy's amount
So,13 women's amount =39 boys
Again, 1 man's amount= (2women+1boy)'s amount
=(2*3 boys+1 boy)'s amount =7boy's amount
Now,15 men's amount=15*7=105 boys
ATQ;
(39+105+12=)936 boys get 936 tk.
So,1 boy gets (936/156)=6tk
Thus, 7boy's amount = 1 man's amount =6*7=42 tk (ans)
375.Rahim can a work by 3 hours, Karim can the same work by 4
hours and Aman can the half work and Aman’s working speed is as
like as Karim’s working speed .85 tk wage given them. Find their
part separately.
Ans with explanations:
Given that,
Aman’s working speed is as like as Karim’s working speed
So,if Kamal takes 4 hrs then for the same work Aman takes 8 hrs (as he takes
4 hrs for 1/2 parts of work)
ATQ;
The ratio of their portion of work in 1 hr 1/3 : 1/4 : 1/8 = 8:6:3
Hence, Rahim's part = 8*85/17 = 40 tk
Kamal's part = 6*85/17 = 30 tk
Aman's part = 3*85/17 = 15 tk (ans)
376.In a partnership business A , B and C’s capital ratio is 1/2 : 1/3
:1/4 .after 4 months A withdraw his half capital and end of the year
profit is 2024 tk .Now findout A’s profit.
Ans with explanations:
The ratio of their initial investment = 1/2 : 1/3 : 1/4
= 6 : 4: 3
Let's take the initial investment of A , B and C’s as 6x, 4x and 3x respectively.
A:B:C = (6x * 2 + 3x * 10) : (4x*12) : (3x*12)
= (12+30) : 4*12 : 3*12
=(4+10) : 4*4 : 12
= 14 : 16 : 12
=7:8:6
So, B's share = 2024*(7/21) = 674.67 tk (ans)

You might also like